You are on page 1of 85

Case No.

1
Topic: Certificate of Title

Castillo v. Escutin

Facts. Petitioner is a judgment creditor of a certain Raquel K. Moratilla. Escutin, the


Register of Deeds of Lipa City.
In the course of her search for properties to satisfy the judgment in her favor, petitioner
discovered that Raquel, her mother Urbana Kalaw, and sister Perla K. Moratilla, co-
owned Lot 13713, a parcel of land consisting of 15,000 square meters, situated at Brgy.
Bugtongnapulo, Lipa City, Batangas, and covered by Tax Declaration No. 00449.
Upon verifying the ownership of the said lot, petitioner was able to secure an Order issued
by the Department of Agrarian Reform approving the application of Summit Point Golf & Country
Club, Inc. for conversion of several agricultural landholdings, including Lot 13713. She was also
able to get from the Office of the City Assessor, Lipa City, a Certification stating that Lot 13713,
covered by Tax Declaration No. 00554-A, was in the name of co-owners Raquel, Urbana, and
Perla; and a certified true copy of Tax Declaration No. 00554-A itself. Lastly, the Register of
Deeds of Lipa City issued a Certification attesting that Lot 13713 in the name of co-owners Raquel,
Urbana, and Perla, was not covered by a certificate of title, whether judicial or patent, or subject
to the issuance of a Certificate of Land Ownership Award or patent under the Comprehensive
Agrarian Reform Program.
Only thereafter did petitioner proceed to levy on execution Lot 13713, and the public
auction sale of the same was scheduled on 14 May 2002. Before the scheduled public auction sale,
petitioner learned that Lot 13713 was inside the Summit Point Golf and Country Club Subdivision
owned by Summit Point Realty and Development Corporation. The public auction still pushed
through and Petitioner bought Raquel’s 1/3 pro-indiviso share in Lot 13713.
Petitioner had the following documents, on her acquisition of Raquels 1/3 pro-
indiviso share in Lot 13713, recorded in the Primary Entry Book and Registration Book of the
Register of Deeds of Lipa City in accordance with Act No. 3344: (a) Notice of Levy; (b) Certificate
of Sale; (c) Affidavit of Publication; and (d) Writ of Execution.
Subsequently, petitioner was issued by the City Assessor of Lipa City Tax Declaration No.
00942-A, indicating that she owned 5,000 square meters of Lot 13713, while Urbana and Perla
owned the other 10,000 square meters.
When petitioner attempted to pay real estate taxes for her 5,000-square-meter share
in Lot 13713, she was shocked to find out that her Tax Declaration No. 00942-A was
cancelled. Lot 13713 was said to be encompassed in and overlapping with the 105,648 square
meter parcel of land known as Lot 1-B, covered by Transfer Certificate of Title (TCT) No. 129642
and Tax Declaration No. 00949-A, both in the name of Francisco Catigbac.
On 25 July 2002, TCT No. 129642 in the name of Catigbac was cancelled and TCT No. T-
134609 in the name of Summit Realty was issued in its place.
Petitioner then filed a complaint before the Office of the Ombudsman to charge the officials
privy on the disputed property.
Petitioner’s Contentions. (1) The supposed Deed of Absolute Sale in favor of Summit
Realty executed on 22 July 2002 by Leonardo Yagin, as Catigbac’s attorney-in-fact, did not
express the desire of Summit Realty, as vendee, to purchase Lot 1-B or indicate its consent and
conformity to the terms of the Deed.
(2) That being a corporation, Summit Realty could only act through its Board of Directors.
(3) That Catigbac had long been dead and buried. Thus, petitioner argued, Yagin no longer
had authority to execute on 22 July 2002 the Deed of Absolute Sale of Lot 1-B in favor of Summit
Realty, making the said Deed null and void ab initio.
(4) That the Special Power of Attorney dated 6 February 1976 granted Yagin the right to
sue on behalf of Catigbac, yet it was Summit Realty which instituted LRC Case No. 00-0376, and
Yagin had no participation at all in said case.
(5) Petitioner questioned why, despite the cancellation of TCT No. 129642 in the name of
Catigbac and the issuance in its place of TCT No. T-134609 in the name of Summit Realty, it was
the former cancelled title which was used as basis for canceling petitioner’s Tax Declaration No.
00942-A. Tax Declaration No. 00949-A was thus still issued in the name of Catigbac, instead of
Summit Realty.

Escutin, relying on the finding of Examiner Juanita H. Sta. Ana, refused to have the
Sheriff’s Deed of Final Sale/Conveyance registered. He likewise denied petitioner’s request to
have her Affidavit of Adverse Claim annotated on TCT No. T-134609.Neither did the Office of
the Deputy Ombudsman for Luzon find any probable cause to criminally charge private
individuals.

Respondent’s Contentions. 1. The claimant’s rights or interest is not adverse to the


registered owner. The registered owner is Summit Point Realty and Development Corporation
under Transfer Certificate of Title No. T-134609 of the Registry of Deeds for Lipa City.
2. The records of the Registry reveals that the source of the rights or interest of the adverse
claimant is by virtue of a Levy on Execution by the Regional Trial Court Fourth Judicial Region,
Branch 30, San Pablo City, in Civil Case No. SP-4489 (1996), [Dinah] C. Castillo vs. Raquel
Buenaventura. The registered owner, Summit Point Realty and Development Corporation nor its
predecessor-in-interest are not the judgment debtor or a party in the said case. Simply stated, there
is no privity of contract between them (Consulta No. 1044 and 1119). If ever, her adverse claim is
against Raquel Buenaventura, the judgment debtor who holds no title over the property.

Issue. Should the petitioner’s title evidenced by tax declaration be upheld rather than that
covered by certificate of title?

Ruling. NO. Between Catigbac’s title, covered by a certificate of title, and petitioner’s
title, evidenced only by a tax declaration, the former is evidently far superior and is, in the absence
of any other certificate of title to the same property, conclusive and indefeasible as to Catigbac’s
ownership of the subject lot.
Title is generally defined as the lawful cause or ground of possessing that which is ours. It
is that which is the foundation of ownership of property, real or personal. Title, therefore, may be
defined briefly as that which constitutes a just cause of exclusive possession, or which is the
foundation of ownership of property. Certificate of title, on the other hand, is a mere evidence of
ownership; it is not the title to the land itself. Under the Torrens system, a certificate of title may
be an Original Certificate of Title, which constitutes a true copy of the decree of registration; or a
Transfer Certificate of Title, issued subsequent to the original registration.
Catigbac’s certificate of title is binding upon the whole world, including respondent public
officers and even petitioner herself. Court has ruled that tax declarations and corresponding tax
receipts cannot be used to prove title to or ownership of a real property inasmuch as they are not
conclusive evidence of the same. Petitioner acquired her title to the 5,000 square meter property
from Raquel, her judgment debtor who, it is important to note, likewise only had a tax declaration
to evidence her title.
A certificate of title issued is an absolute and indefeasible evidence of ownership of the
property in favor of the person whose name appears therein. It is binding and conclusive upon the
whole world. Therefore, upon presentation of TCT No. 129642, the Office of the City Assessor
must recognize the ownership of Lot 1-B by Catigbac and issue in his name a tax declaration for
the said property. And since Lot 1-B is already covered by a tax declaration in the name of
Catigbac, accordingly, any other tax declaration for the same property or portion thereof in the
name of another person, not supported by any certificate of title, such that of petitioner, must be
cancelled.
Therefore, a certificate of title held by Summit Realty constitutes conclusive and
indefeasible evidence of its ownership of the said property and, thus, cannot be collaterally
attacked in the administrative and preliminary investigations.

Case No. 2
Topic: The Concept of Torrens System; Registration of Proceeding in rem

Acosta v. Salazar

FACTS. On November 19, 1985, respondents Trinidad and Aniceta Salazar (hereinafter,
Salazars), filed a petition for the cancellation of the entries annotated at the back of Original
Certificate of Title (OCT) No. 40287 registered in the names of spouses Juan Soriano and Vicenta
Macaraeg, who died without issue. The Salazars claim that two of the entries Entry Nos. 19756
and 20102 annotated at the back of the aforesaid title are void since no consolidation of rights
appear in the Registry of Deeds (RD) of Tarlac to support the entries; and that Transfer Certificate
of Title (TCT) No. 9297, which supposedly cancelled OCT No. 40287, is non-existent according
to a certification issued by the RD. RTC Branch 63 of Tarlac resolved to grant the petition and
ordered the cancellation of Entry No. 20102.

ISSUE. Whether or not claim of the respondent is valid?

RULING. The court ruled in favor of the petitioner.


It is true that the registration of land under the Torrens system is a proceeding in rem and
not in personam. Such a proceeding in rem, dealing with a tangible res, may be instituted and
carried to judgment without personal service upon the claimants within the state or notice by mail
to those outside of it. Jurisdiction is acquired by virtue of the power of the court over the res. Such
a proceeding would be impossible were this not so, for it would hardly do to make a distinction
between constitutional rights of claimants who were known and those who were not known to the
plaintiff, when the proceeding is to bar all. Once a title is registered under the Torrens system, the
owner may rest secure, without the necessity of waiting in the portals of the courts or sitting in the
mirador su casa to avoid the possibility of losing his land. Rarely will the court allow another
person to attack the validity and indefeasibility of a Torrens certificate, unless there is compelling
reason to do so and only upon a direct action filed in court proceeded in accordance with law.
Finally, this Court also takes note of the fact that for more than 30 years from the time
Entry No. 20102 was annotated at the back of OCT No. 40287 on February 17, 1950 until the time
of the filing of the ex parte petition for cancellation of entries on the said certificate of title on
November 19, 1985 the Salazars remained deafeningly quiet and never made any move to question
the issue of ownership over the said land before the proper forum. They also failed to ventilate
their claim during the intestate proceeding filed by the heirs of Juan Soriano sometime in 1939.
Likewise, they miserably failed to stop the transfer of portions of the property to petitioners who,
for themselves, were able to secure TCTs in their own names.

Case No. 3
Topic: Purpose of Torrens System

Ingusan v. Heirs of Aureliano Reyes


Facts:This case involves a 1,254 sq. m. residential land located in Poblacion, San
Leonardo, Nueva Ecija originally owned by Leocadio Ingusan who was unmarried and childless
when he died in 1932. His heirs were his two brothers and a sister, namely, Antonio, Macaria and
Juan. Antonio died and was succeeded by his son Ignacio who also later died and was succeeded
by his son, petitioner Miguel Ingusan. Macaria also died and was succeeded by her child, Aureliano
I. Reyes, Sr. (father of respondents Artemio Reyes, Corazon Reyes-Reguyal, Elsa Reyes, Estrella
Reyes-Razon, Aureliano Reyes, Jr., Ester Reyes, Reynaldo Reyes and Leonardo Reyes). Thus,
petitioner is the grandnephew of Leocadio and Aureliano, Sr. was the latter's nephew. After the
death of Leocadio, Aureliano, Sr. was designated by the heirs as administrator of the land. In 1972,
while in possession of the land and in breach of trust, he applied for and was granted a free patent
over it. As a result, he was issued OCT No. P-6176 in 1973. In 1976, petitioner filed an accion
reivindicatoria against Aureliano, Sr. and his wife Jacoba Solomon seeking the recovery of Lot
120-A with an area of 502 sq. m. which was part of the land at issue here. But the case was
dismissed because petitioner did not pursue it.Also in 1976, Aureliano, Sr. executed a special
power of attorney (SPA) in favor of his son Artemio authorizing him to mortgage the land in
question to any bank. Using that SPA, Artemio mortgaged the land to secure a loan of P10,000
from the Philippine National Bank (PNB). In 1983, Aureliano, Sr. died intestate. He was survived
by his children, the respondents. In 1986, petitioner paid the PNB loan. The mortgage over the
land was released and the owners duplicate copy of OCT No. P-6176 was given to him.
On June 19, 1988, respondents and petitioner entered into a Kasulatan ng Paghahati-hati
Na May Bilihan wherein they adjudicated unto themselves the land in question and then sold it to
their co-heirs, as follows: (a) to petitioner, 1,171 sq. m. and (b) to respondent Estrella, 83 sq. m.
This deed was notarized but not registered.
On January 8, 1990, respondent Corazon, despite signing the Kasulatan, executed an
affidavit of loss, stating that she could not find the owners duplicate copy of OCT No. P-6176.
This was registered and annotated on the original copy of said title.
Subsequently, the following documents appeared purportedly with the following dates:
a) April 23, 1994 - notarized deed of donation of titled property supposedly executed by the
spouses Aureliano, Sr. and Jacoba, whereby said spouses donated 297 sq. m. of the subject land to
respondent Artemio and the remaining 957 sq. m. to petitioner;
b) September 5, 1994 - cancellation of affidavit of loss supposedly executed by respondent
Corazon stating that the annotation of the affidavit of loss on the title should be canceled and the
petition for a new title was no longer necessary because she had already found the missing owners
duplicate copy of OCT No. P-6176;
c) September 27, 1994 agreement of subdivision with sale purportedly executed by respondent
Artemio and petitioner, with the consent of their wives. Pursuant to this document, the land was
subdivided into Lot 120-A with an area of 297 sq. m. corresponding to the share of Artemio and
Lot 120-B with an area of 957 sq. m. which was the share of petitioner. The document also
indicated that Artemio sold Lot 120-A to one Florentina Fernandez.
When respondent Corazon learned about the cancellation of the annotation of her affidavit
of loss, she executed an affidavit of adverse claim on January 17, 1995 stating that the cancellation
of affidavit of loss and the agreement of subdivision with sale were both spurious and the
signatures appearing thereon were forgeries. This affidavit of adverse claim was not registered.
On April 17, 1995, petitioner brought the owners duplicate copy of OCT No. P-6176, the
cancellation of affidavit of loss, deed of donation of titled property and agreement of subdivision
with sale to the Registry of Deeds for registration. Consequently, the following took place on that
same day:
1. Corazons annotated affidavit of loss was canceled;
2. by virtue of Aureliano, Sr. and Jacobas deed of donation of titled property to Artemio and
petitioner, OCT No. P-6176 was canceled and in lieu thereof, TCT No. NT-241155 in the name of
petitioner and TCT No. NT-241156 in the name of respondent Artemio were issued and
3. by virtue of the agreement of subdivision with sale, TCT Nos. NT-241155 and NT-241156
were canceled and TCT Nos. NT-239747 and NT-239748 were issued in the names of petitioner
and Florentina Fernandez, respectively.
On June 27, 1995, petitioner took possession of his portion and built his house thereon. On
July 4, 1995, respondents filed an action for cancellation, annulment and surrender of titles with
damages against petitioner and Florentina Fernandez in the RTC of Cabanatuan City, Nueva Ecija,
Branch 25.In a decision dated April 17, 1997, the RTC dismissed the case and declared OCT No.
P-6176 as well as the subsequent certificates of title (TCT Nos. NT-239747 and NT-239748), the
deed of donation of titled property, agreement of subdivision with sale and cancellation of affidavit
of loss as null and void. It held that the aforementioned documents were spurious since the
signatures were falsified by respondent Artemio. On appeal, the CA modified the RTC decision.
It ruled that only TCT Nos. NT-241155, NT-241156, NT-239747 and NT-239748 were null and
void. Their source, OCT No. P-6176, remained valid because it had already become indefeasible
and could no longer be attacked collaterally.
Issue: Whether or not the OCT No. P-6176 was valid.
Held: There is no doubt that the deed of donation of titled property, cancellation of
affidavit of loss and agreement of subdivision with sale, being falsified documents, were null and
void. It follows that TCT Nos. NT-241155, NT-241156, NT-239747 and NT-239748 which were
issued by virtue of these spurious documents were likewise null and void. Neither side disputes
these findings and conclusions.
The question is whether the source of these titles, OCT No. P-6176, was valid. Petitioner
argues that it should be invalidated because it was issued based on a fictitious affidavit purportedly
executed in 1970 by Leocadio (who died in 1932) wherein the latter supposedly sold the land to
Aureliano, Sr. According to petitioner, Aureliano, Sr. used this to fraudulently and in breach of
trust secure a free patent over the land in his name.
We agree with the CA that OCT No. P-6176 remains valid. The issue of the validity of title
(e.g. whether or not it was issued fraudulently or in breach of trust) can only be assailed in an
action expressly instituted for that purpose. A certificate of title cannot be attacked collaterally.
Section 48 of PD 1529[32] states:
SEC. 48. Certificate not subject to collateral attack. ― A certificate of title shall not be subject to
collateral attack. It cannot be altered, modified, or canceled except in a direct proceeding in
accordance with law.
The rationale behind the Torrens System is that the public should be able to rely on a
registered title. The Torrens System was adopted in this country because it was believed to be the
most effective measure to guarantee the integrity of land titles and to protect their indefeasibility
once the claim of ownership is established and recognized. In Fil-estate Management, Inc. v.
Trono, we explained:
It has been invariably stated that the real purpose of the Torrens System is to quiet title to
land and to stop forever any question as to its legality. Once a title is registered, the owner may
rest secure, without the necessity of waiting in the portals of the court, or sitting on the "mirador
su casa" to avoid the possibility of losing his land.
Case No. 4
Topic: Registration in proceeding in rem; Notice of Initial Hearing
Republic v. Herbieto
FACTS: Respondents in the present Petition are the Herbieto brothers, Jeremias and
David, who filed with the MTC, on 23 September 1998, a single application for registration of two
parcels of land located in Cabangahan, Consolacion, Cebu (Subject Lots). They claimed to be
owners in fee simple of the Subject Lots, which they purchased from their parents, spouses
Gregorio Herbieto and Isabel Owatan, on 25 June 1976.
On 11 December 1998, the petitioner Republic of the Philippines (Republic) filed an
Opposition to the respondents' application for registration of the Subject Lots. The MTC set the
initial hearing on 03 September 1999 at 8:30 a.m. All owners of the land adjoining the Subject
Lots were sent copies of the Notice of Initial Hearing. A copy of the Notice was also posted on 27
July 1999 in a conspicuous place on the Subject Lots, as well as on the bulletin board of the
municipal building of Consolacion, Cebu, where the Subject Lots were located. Finally, the Notice
was also published in the Official Gazette on 02 August 1999 and The Freeman Banat News on
19 December 1999. On 21 December 1999, the MTC promulgated its Judgment ordering the
registration and confirmation of the title of respondents. It subsequently issued an Order on 02
February 2000 declaring its Judgment final and executory, and directing the Administrator of the
Land Registration Authority (LRA) to issue a decree of registration for the Subject Lots.
Petitioner Republic appealed the MTC Judgment to the Court of Appeals, which affirmed
the appealed MTC Judgment.
The Republic filed the present Petition for the review and reversal of the Decision of the
Court of Appeals on the basis that the respondents failed to comply with the procedure for land
registration under the Property Registration Decree, hence, the proceedings held before the MTC
is void, as the latter did not acquire jurisdiction over it.
ISSUE: Did the respondents comply with the publication requirements mandated by the
Property Registration Decree, thus, vesting the MTC with jurisdiction as a land registration court?
RULING: No. A land registration case is a proceeding in rem, and jurisdiction in rem
cannot be acquired unless there be constructive seizure of the land through publication and service
of notice. Section 23 of the Property Registration Decree requires that the public be given Notice
of the Initial Hearing of the application for land registration by means of (1) publication; (2)
mailing; and (3) posting. Also, the Court declared that publication in a newspaper of general
circulation is mandatory for the land registration court to validly confirm and register the title of
the applicant or applicants. In the instant Petition, the initial hearing was set by the MTC, and was
in fact held, on 03 September 1999 at 8:30 a.m. While the Notice thereof was printed in the issue
of the Official Gazette, dated 02 August 1999, and officially released on 10 August 1999, it was
published in The Freeman Banat News, a daily newspaper printed in Cebu City and circulated in
the province and cities of Cebu and in the rest of Visayas and Mindanao, only on 19 December
1999, more than three months after the initial hearing. Indubitably, such publication of the Notice,
way after the date of the initial hearing, would already be worthless and ineffective. Whoever read
the Notice as it was published in The Freeman Banat News and had a claim to the Subject Lots
was deprived of due process for it was already too late for him to appear before the MTC on the
day of the initial hearing to oppose respondents' application for registration, and to present his
claim and evidence in support of such claim.
The late publication of the Notice of Initial Hearing in the newspaper of general circulation
is tantamount to no publication at all. Owing to such defect in the publication of the Notice, the
MTC failed to constructively seize the Subject Lots and to acquire jurisdiction over respondents'
application for registration thereof. Therefore, the MTC Judgment is null and void for having been
issued by the MTC without jurisdiction.
Case No. 5
Topic: Registration in proceeding in rem
Munoz vs Yabut
Facts: Yee Ching is married to Emilia, Munoz sister. Munoz is living with spouses Ching
in their house and lot. As consideration for the valuable services rendered by Munoz to the spouses
Chings family, Yee L. Ching agreed to have the house and lot transferred to Munoz. After a Deed
of Absolute Sale was executed by Ching in favor of Munoz, TCT was issued in her name.
Allegedly, on December 28, 1972 Munoz executed a Deed of Absolute Sale selling the subject
property to Emilia who later sold the property to spouses Go. Munoz, however, filed an adverse
claim to the property and complaint for annulment of the Deed of Absolute Sale dated December
28, 1972 and the TCT in spouses Go’s name. Included in the complaint is the restoration of TCT
issued in Munoz name. The RTC granted the spouses Go’s motion for the issuance of a writ of
preliminary mandatory injunction and ordered the sheriff to put the spouses Go in possession of
the subject property. The writ was implemented by the sheriff on March 26, 1980, driving Munoz
and her housemates away from the subject property. As a result, the notice of lis pendens caused
by Munoz in TCT issued in her name was cancelled. Munoz filed a petition for certiorari and
prohibition before CA.
Spouses Go obtained a loan from BPI and to secure the loan the property which they bought
was mortgage. However, when spouses Go defaulted in paying the BPI foreclosed the mortgage
and was able to obtained a TCT in BPI’s name. The BPI sold the property to spouses Chan and a
TCT was issued in their name. Meanwhile, the complaint made by Munoz in CA found out that
the signature of Munoz was forged; hence the sale was declared null and void. The RTC issued
writ of execution but the Spouses Chan filed an Urgent Motion to Stop the Execution issued by
RTC. The spouses Chan further contended that the final judgment could not be executed against
them since they were not parties to the said case and they purchased the subject property from BPI
Family without any notice of defect in the latters title. However, the petition of spouses Go was
denied because the property was in dispute as there was a notice of lis pendens.
Issue: Whether or not the TCT issued to Spouses Go is null and void
Ruling: The complaint made by Munoz is an action for reconveyance of real property. The
Supreme Court declared that an action for reconveyance is an action in personam available to a
person whose property has been wrongfully registered under the Torrens system in anothers name.
Although the decree is recognized as incontrovertible and no longer open to review, the registered
owner is not necessarily held free from liens. As a remedy, an action for reconveyance is filed as
an ordinary action in the ordinary courts of justice and not with the land registration court.
Reconveyance is always available as long as the property has not passed to an innocent third person
for value. A notice of lis pendens may thus be annotated on the certificate of title immediately
upon the institution of the action in court. The notice of lis pendens will avoid transfer to an
innocent third person for value and preserve the claim of the real owner. An action for declaration
of nullity of title and recovery of ownership of real property, or re-conveyance, is a real action but
it is an action in personam, for it binds a particular individual only although it concerns the right
to a tangible thing. Any judgment therein is binding only upon the parties properly impleaded.
Since they were not impleaded as parties and given the opportunity to participate in the complaint
of Munoz the final judgment in said case cannot bind BPI Family and the spouses Chan. The effect
of the said judgment cannot be extended to BPI Family and the spouses Chan by simply issuing
an alias writ of execution against them. No man shall be affected by any proceeding to which he
is a stranger, and strangers to a case are not bound by any judgment rendered by the court. In the
same manner, a writ of execution can be issued only against a party and not against one who did
not have his day in court. Only real parties in interest in an action are bound by the judgment
therein and by writs of execution issued pursuant thereto.

Case No. 6
Topic: The Regalian Doctrine; Alienable and Disposable Land of the Public Domain
Republic v. Medida
FACTS: On October 22, 2004, herein respondent Marlon Medida filed with the RTC,
Argao, Cebu a petition for registration of title over two parcels of land situated in Poblacion,
Boljoon, Cebu, Medida testified that he purchased the subject properties in February 1997 from
one Eufemia Romero, who had previously obtained the lots from Nabor Derama. At the time of
the lots’ purchase by Medida, the properties were covered by Tax Declaration No. 08774 under
the name of Romero. Medida started occupying the properties in 1997, and had since then declared
the properties for tax purposes under his name.
Binagatan, daughter of Derama, testified that her father had inherited the subject properties
from his uncle, one Florencio Villareal, who possessed the lots even prior to the Second World
War. She presented the old Tax Declaration No. 08590 under the name of her father and covering
the subject properties.
The respondent argues that the Advance Survey Plans that were prepared by Engr.
Dumaguing and approved by the DENR-Land Management Bureau, providing notations that the
lots indicated therein are within the alienable and disposable properties of the State, should suffice.
RTC finds the petitioner to have sufficient title proper for registration, the petition is hereby
GRANTED and judgment is hereby rendered confirming the title of petitioner Marlon D. Medida,
married to Patricia F. Medida. The CA affirmed the decision of the lower court.
The Republic argues that the alienable and disposable character of the subject parcels of
land has not been sufficiently proved by the mere presentation of the surveyor’s notations on the
Advance Survey Plans for Lot Nos. 817 and 597. Petitioner Republic claims that such requirement
must be established by the existence of a positive act of the government, such as a presidential
proclamation or an executive order, an administrative action, investigation reports of Bureau of
Lands investigators, and a legislative act or statute.
ISSUE: whether or not the land is alienable and disposable based solely on the survey
plan
RULING: Under the Regalian Doctrine, which is embodied in our Constitution, all lands
of the public domain belong to the State, which is the source of any asserted right to any ownership
of land. All lands not appearing to be clearly within private ownership are presumed to belong to
the State. Accordingly, public lands not shown to have been reclassified or released as alienable
agricultural land, or alienated to a private person by the State, remain part of the inalienable public
domain. As the rule now stands, an applicant must prove that the land subject of an application for
registration is alienable and disposable by establishing the existence of a positive act of the
government such as a presidential proclamation or an executive order. In a line of cases, we have
ruled that mere notations appearing in survey plans are inadequate proof of the covered properties’
alienable and disposable character.
This Court also holds that the alienability and disposability of land are not among the matters that
can be established by mere admissions, or even the agreement of parties. The law and
jurisprudence provide stringent requirements to prove such fact. Our Constitution,no less,
embodies the Regalian doctrine that all lands of the public domain belong to the State, which is
the source of any asserted right to ownership of land. The courts are then empowered, as we are
duty-bound, to ensure that such ownership of the State is duly protected by the proper observance
by parties of the rules and requirements on land registration.
Case No. 7
Topic: Alienable and Disposable Land of the Public Domain

Republic of the Philippines v. Celestina Naguiat


Facts: Celestina Naguiat filed an application for registration of title to four parcels of land
located in Panan, Botolan, Zambales.
Petitioner’s Contention:Petitioner Republic opposed on the ground that neither the
applicant nor her predecessors-in interest have been in open, continuous, exclusive and notorious
possession and occupation of the lands in question since 12 June 1945 or prior thereto, considering
the fact that she has not established that the lands in question have been declassified from forest
or timber zone to alienable and disposable property.
Respondent’s Contention: She alleged that she is the owner of the said parcels of land
having acquired them by purchase from the LID Corporation which likewise acquired the same
from Demetria Calderon, Josefina Moraga and Fausto Monje and their predecessors-in-interest
who have been in possession thereof for more than thirty (30) years; and that to the best of her
knowledge, said lots suffer no mortgage or encumbrance of whatever kind nor is there any person
having any interest, legal or equitable, or in possession thereof.
Issue: Did the areas in question cease to have the status of forest or other inalienable lands
of the public domain?
Held: No, the said areas are still classified as forest land. The issue of whether or not
respondent and her predecessors-in-interest have been in open, exclusive and continuous
possession of the parcels of land in question is of little moment. For, unclassified land cannot be
acquired by adverse occupation or possession; occupation thereof in the concept of owner,
however long, cannot ripen into private ownership and be registered as title.
A forested area classified as forest land of the public domain does not lose such
classification simply because loggers or settlers have stripped it of its forest cover. Parcels of land
classified as forest land may actually be covered with grass or planted to crops by kaingin
cultivators or other farmers. "Forest lands" do not have to be on mountains or in out of the way
places. The classification is merely descriptive of its legal nature or status and does not have to be
descriptive of what the land actually looks like.
Case No. 8
Topic: Alienable and Disposable Land of the Public Domain

Aranda v. Republic
FACTS: Subject of a petition for original registration before the RTC is a parcel of land
situated in San Andres, Malvar, Batangas with an area of 9,103 square meters and designated as
Lot 3730, Psc 47, Malvar Cadastre. The petition was originally filed by ICTSI Warehousing, Inc.
(ICTSI-WI) represented by its Chairman, Enrique K. Razon, Jr. ICTSI-WI sought leave of court
to amend the application citing the following reasons: (1) the petition was not accompanied by a
certification of non-forum shopping; (2) the statement of technical description was based merely
on the boundaries set forth in the tax declaration; and (3) due to a technicality, the sale between
the vendor and applicant corporation cannot push through and consequently the tax declaration is
still in the name of vendor Ramon Aranda and the land cannot be transferred and declared in the
name of ICTSI-WI. The trial court admitted the Amended Application for Registration of Title,
this time filed in the name of Ramon Aranda, herein petitioner. Petitioner prayed that should the
Land Registration Act be not applicable to this case, he invokes the liberal provisions of Section
48 of Commonwealth Act No. 141, as amended, having been in continuous possession of the
subject land in the concept of owner, publicly, openly and adversely for more than thirty (30) years
prior to the filing of the application.
ISSUE: Whether or not the land applied for is part of the public domain and the applicant
has not acquired a registrable title thereto under the provisions of Commonwealth Act No. 141 as
amended by Republic Act No. 6940.

RULING: Petition is denied.


The Property Registration Decree (P.D. No. 1529) provides for original registration of land
in an ordinary registration proceeding. Under Section 14(1) thereof, a petition may be granted upon
compliance with the following requisites: (a) that the property in question is alienable and
disposable land of the public domain; (b) that the applicants by themselves or through their
predecessors-in-interest have been in open, continuous, exclusive and notorious possession and
occupation; and (c) that such possession is under a bona fide claim of ownership since June 12,
1945 or earlier.
Under the Regalian doctrine which is embodied in Section 2, Article XII of the 1987
Constitution, all lands of the public domain belong to the State, which is the source of any asserted
right to ownership of land. All lands not appearing to be clearly within private ownership are
presumed to belong to the State. Unless public land is shown to have been reclassified or alienated
to a private person by the State, it remains part of the inalienable public domain. To overcome this
presumption, incontrovertible evidence must be established that the land subject of the application
is alienable or disposable.
To prove that the land subject of an application for registration is alienable, an applicant
must establish the existence of a positive act of the government such as a presidential proclamation
or an executive order; an administrative action; investigation reports of Bureau of Lands
investigators; and a legislative act or a statute.

Case No. 9
Topic: Alienable and Disposable Land of the Public Domain; Notice of Initial Hearing

Republic v. Bantigue Point Development Corporation


Facts: On 17 July 1997, respondent Bantigue Point Development Corporation filed with
the Regional Trial Court (RTC) of Rosario, Batangas an application for original registration of title
over a parcel of land with an assessed value of ₱4,330, ₱1,920 and ₱8,670, or a total assessed value
of ₱14,920 for the entire property, more particularly described as Lot 8060 of Cad 453-D, San
Juan Cadastre, with an area of more or less 10,732 square meters, located at Barangay Barualte,
San Juan, Batangas. On 18 July 1997, the RTC issued an Order setting the case for initial hearing
on 22 October 1997. On 7 August 1997, it issued a second Order setting the initial hearing on 4
November 1997. Petitioner Republic filed its Opposition to the application for registration on 8
January 1998 while the records were still with the RTC. On 31 March 1998, the RTC Clerk of
Court transmitted motu proprio the records of the case to the MTC of San Juan, because the
assessed value of the property was allegedly less than ₱100,000.
Thereafter, the MTC entered an Order of General Default and commenced with the
reception of evidence. Among the documents presented by respondent in support of its application
are Tax Declarations, a Deed of Absolute Sale in its favor, and a Certification from the Department
of Environment and Natural Resources (DENR) Community Environment and Natural Resources
Office (CENRO) of Batangas City that the lot in question is within the alienable and disposable
zone. Thereafter, it awarded the land to respondent Corporation.
Acting on an appeal filed by the Republic, the CA ruled that since the former had actively
participated in the proceedings before the lower court, but failed to raise the jurisdictional
challenge therein, petitioner is thereby estopped from questioning the jurisdiction of the lower
court on appeal. The CA further found that respondent Corporation had sufficiently established
the latters registrable title over the subject property after having proven open, continuous,
exclusive and notorious possession and occupation of the subject land by itself and its
predecessors-in-interest even before the outbreak of World War II.
Issue: Whether or not the Municipal Trial Court acquired jurisdiction over the case.
Held: We uphold the jurisdiction of the MTC, but remand the case to the court a quo for
further proceedings in order to determine if the property in question forms part of the alienable
and disposable land of the public domain.
The Property Registration Decree provides: Sec. 23. Notice of initial hearing, publication,
etc. - The court shall, within five days from filing of the application, issue an order setting the date
and hour of the initial hearing which shall not be earlier than forty-five days nor later than ninety
days from the date of the order. x x x.
In this case, the application for original registration was filed on 17 July 1997. On 18 July
1997, or a day after the filing of the application, the RTC immediately issued an Order setting the
case for initial hearing on 22 October 1997, which was 96 days from the Order. While the date set
by the RTC was beyond the 90-day period provided for in Section 23, this fact did not affect the
jurisdiction of the trial court. In Republic v. Manna Properties, Inc., petitioner Republic therein
contended that there was failure to comply with the jurisdictional requirements for original
registration, because there were 125 days between the Order setting the date of the initial hearing
and the initial hearing itself. We ruled that the lapse of time between the issuance of the Order
setting the date of initial hearing and the date of the initial hearing itself was not fatal to the
application. Thus, we held:
x x x [A] party to an action has no control over the Administrator or the Clerk of Court
acting as a land court; he has no right to meddle unduly with the business of such official
in the performance of his duties. A party cannot intervene in matters within the exclusive
power of the trial court. No fault is attributable to such party if the trial court errs on matters
within its sole power. It is unfair to punish an applicant for an act or omission over which
the applicant has neither responsibility nor control, especially if the applicant has complied
with all the requirements of the law.
Case No. 10
Topic: Inalienable Property

Ituralde v. Falcasantos
FACTS: On October 17, 1986, petitioner acquired by purchase from the heirs of Pedro
Mana-ay a parcel of land located at Baas, Lantawan, Basilan Province. However, on November 3,
1986, respondent applied with the Bureau of Lands in Isabela, Basilan province, for the award to
him of the same parcel of land under free patent. On November 17, 1986, petitioner filed a protest
to such application.
On February 7, 1989, the Regional Director of Lands rendered a decision giving respondent
a period of 120 days to exercise the right to repurchase the land by reimbursing petitioner of all
expenses he incurred in the purchase of the property in question, and held in abeyance respondent's
application for free patent. On October 11, 1989, the Regional Director issued an order declaring
that respondent had waived his right of repurchase, and rejected his application for free patent for
lack of interest, and allowed petitioner to file a public land application for the subject land.
On May 8, 1990, the Regional Director ordered respondent to vacate the land in question,
but respondent refused. So on July 24, 1990, petitioner filed with the Regional Trial Court, Basilan
province, a complaint for recovery of ownership and possession with preliminary injunction of the
subject parcel of land.
In answer to the complaint, respondent alleged that the land occupied by him belonged to
the Republic of the Philippines, and that he had introduced improvements thereon such as coconut
and other fruit trees.
After trial on the merits, the trial court rendered decision declaring petitioner the owner
and possessor of the subject parcel of land with all the improvements existing thereon. The decision
was appealed to the Court of Appeals, which rendered a decision reversing the appealed decision,
and entering a new judgment dismissing petitioner's complaint.
Hence, the present recourse. Petitioner submits that the Court of Appeals erred in setting
aside the trial court's decision in his favor and dismissing the complaint because when the Director
of Lands allowed petitioner to file a public land application for said property, it was equivalent to
a declaration that said land was no longer part of the public domain.
ISSUE: Is the subject land declassified from forest land to alienable and disposable land
of the public domain by the action of the Director of Lands in allowing the filing of a public land
application for said property?
RULING: No. The Court of Appeals correctly held that "the evidence is unrebutted that
the subject land is within the Forest Reserve Area and, hence, not capable of private appropriation
and occupation. In Republic v. Register of Deeds of Quezon, the Court held that "Forest lands,
like mineral or timber lands which are public lands, are not subject to private ownership unless
they under the Constitution, become private properties. In the absence of such classification, the
land remains unclassified public land until released therefrom and rendered open to disposition.
Also, in Sunbeam Convenience Foods Inc. v. Court of Appeals, the Court said: “Thus, before any
land may be declassified from the forest group and converted into alienable or disposable land for
agricultural or other purposes, there must be a positive act from the government. Even rules on the
confirmation of imperfect titles do not apply unless and until the land classified as forest land is
released in an official proclamation to that effect so that it may form part of the disposable
agricultural lands of the public domain."
Hence, a positive act of the government is needed to declassify a forest land into alienable
or disposable land for agricultural or other purposes. And the rule is Possession of forest lands,
however long, cannot ripen into private ownership. What is more, there is yet no award or grant to
petitioner of the land in question by free patent or other ways of acquisition of public land.
Consequently, he cannot lawfully claim to be the owner of the land in question.
Case No. 11
Topic: Inalienable Property

Gordula vs. Court of Appeals


Facts: On June 26, 1969, former President Ferdinand E. Marcos issued Proclamation No.
573 withdrawing from sale and settlement and setting aside as permanent forest reserves, subject
to private rights, certain parcels of the public domain which were primarily for use as watershed
area. Their development was to be undertaken by the Bureau of Forestry, with the cooperation of,
among other government agencies, the National Power Corporation (Napocor). However, Parcel
No.9 the Caliraya-Lumot River Forest Reserve has been subjected for Application for a Free Patent
filed by Gordula with the Bureau of Lands on January 9, 1973.The application has been approved
by Bureau of Land declaring that applicants have sufficient ground to establish `priority rights'
over the areas claimed and that agricultural improvements introduced thereon are not detrimental
to the watershed and it was issued in his name. The OCT also was issued in his name.
On June 22, 1979, Gordula sold the land to Fernandez. Fernandez subdivided the lots and
later sold to Estrellado. On July 16, 1987, former President Corazon Aquino issued Executive
Order (E.O.) No. 224 vesting in the Napocor "complete jurisdiction, control and regulation" over
the "Caliraya-Lumot Watershed Reservation as covered by Proclamation No. 573". In July, some
fences and roads were being built in the lots sold to Estrellado. The Manager of the Cavinti
reservoir complex asked petitioner Fernandez to remove all the improvements made in the
Estrellado lots. In reply, petitioner Fernandez claimed that the roads being constructed would not
adversely affect the reservoir area in case of heavy floods because the Estrellado lots were elevated.
As a result, the Napocor assigned guards over the lot. On November 18, 1987, respondent
Republic, through the Napocor, filed against petitioners a Complaint for Annulment of Free Patent
and Cancellation of Titles and Reversion with Writ of Preliminary Injunction in the RTC of Sta.
Cruz, Laguna. The RTC ruled in favour of the petitioners hence it was elevated to CA. The CA set
aside and reversed the decision of RTC and ruled that Free Patent No. IV-5-693 and Original
Certificate of Title No. P1405 issued under the name of Edubigis Gordula and all derivative titles
issued to the Appellees are hereby declared null and void.
Issue: Whether or not the Free Patent and OCT issued to Gordula is valid.
Ruling: The Supreme Court affirmed the decision of CA stating that no public land can be
acquired by private persons without any grant, express or implied from the government; it is
indispensable that there be a showing of a title from the state. The facts show that petitioner
Gordula, did not acquire title to the subject land prior to its reservation under Proclamation No.
573. He filed his application for free patent only in January, 1973, more than three (3) years after
the issuance of Proclamation No. 573 in June, 1969. At that time, the land, as part of the Caliraya-
Lumot River Forest Reserve, was no longer open to private ownership as it has been classified as
public forest reserve for the public good. The Supreme Court also declared that one who is
claiming“private rights” must prove that he has complied with C. A. No. 141 which requires at
least thirty (30) years of open, continuous, exclusive and notorious possession and occupation of
agricultural lands of the public domain, under a bona fide claim of acquisition, immediately
preceding the filing of the application for free patent. The rationale for the 30-year period lies in
the presumption that the land applied for pertains to the State, and that the occupants and/or
possessors claim an interest therein only by virtue of their imperfect title or continuous, open and
notorious possession. Indeed, the possession of public agricultural land, however long the period
may have extended, never confers title thereto upon the possessor.
Case No. 12
Topic: Inalienable Property

Director of Forestry v. Villareal


Facts: Petitioner, Director of Forestry was one of the several persons who opposed the
application for registration of a parcel land classified as mangrove swamps in the municipality of
Sapian, Capiz with an area of 178,113 square meters of mangrove swamps, to the applicant
Ruperto Villareal. He alleged that he and his predecessors-in-interests had been in possession of
the said parcel of land for more than forty years (40). Both parties agreed in one point that the
disputed land was a mangrove swamp. The respondent argued that mangrove swamp are
agricultural land but the petitioner contended that it is a forestall land therefore not disposable.The
Court of the First Instance of Capiz however grants the application of the respondent. The decision
of the lower court was later affirmed by the Court of Appeals. Hence the Director of Forestry
elevated the case to the Supreme Court for review on certiorari.
Issue: Whether or not, mangrove swamps are agricultural land or forest land.
Held: The Supreme Court held that mangrove swamps as forest lands is descriptive of its
legal nature or status and does not have to be descriptive of what the land actually looks like.
Furthermore, the legislative definition embodied in section 1820 of the Revised Administrative
Code of 1917 which declares that mangrove swamps or manglares form part of the public forests
of the Philippines hence they are not alienable. The evidence presented by the respondent in its
claim were not sufficient to prove its possession and ownership of the land, he only presented tax
declaration. Wherefore the decision of the Court of Appeals was set aside and the application for
registration of title by the respondent is dismissed by the Supreme Court.
Case No. 13
Topic: Inalienable Property

De Ocampo v. Arlos
Facts:Federico S. Arlos and Teofilo D. Ojerio filed an application for registration,
docketed as Land Registration Case No. N-340, wherein they seek judicial confirmation of their
titles to three parcels of land. Petitioner’s Spouses Ocampo, Spouses Santos, Spouses Manalao and
Spouses Manalo and Baron, including the Republic opposed to the said registration.
Petitioner’s Contentions: Spouses Geminiano de Ocampo and Amparo De Ocampo and
spouses Pedro Santos and Crisanta Santos opposed the application for registration, alleging that
they are the co-owners of Lots 1 and 2 of Plan SGS 3062, situated at Cabcaben, Mariveles, Bataan,
and their ownership is evidenced by Transfer Certificate of Title Nos. T-43298 and T-44205, and
that they became owners of said lots by purchase from the government through sales patents.
The Republic of the Philippines also opposed the application, contending that neither the
applicants nor their predecessors-in-interests have been in open, continuous, exclusive and
notorious possession and occupation of the lands in question for at least 30 years immediately
preceding the filing of the application; and that the parcels of land applied for are portions of the
public domain belonging to the Republic of the Philippines not subject to private appropriation.
Respondent’s Contentions: They purchased the subject lots in 1967 from Bernardo and
Arsenio Obdin, who in turn had been in possession of the property since 1947. Hence, when the
former filed their application for registration in 1977, they and their predecessors-in-interest had
been occupying and cultivating, in the concept of owners, the said parcels of land for at least 30
years, as required by the Public Land Act.
Issue: Is the registration of the respondent’s title under the Public Land Act proper?
Held: Respondent’s application for registration of title to the three parcels of land that were
once part of the public domain is governed by the Public Land Act, a title may be judicially
confirmed under Section 48 of the Public LandAct only if it pertains to alienable lands of the public
domain. Unless such assets are reclassified and considered disposable and alienable, occupation
thereof in the concept of owner, no matter how long cannot ripen into ownership and be registered
as a title. Verily, Presidential Decree No. 1073 clarified Section 48 (b) of the Public Land Act by
specifically declaring that the latter applied only to alienable and disposable lands of the public
domain.
In the present case, the disputed land which was formerly a part of a US military reservation
that had been turned over to the Philippine government in 1965, was declared disposable and
alienable only in1971. Second, respondents and their predecessors-in-interest could not have
occupied the subject property from 1947 until 1971 when the land was declared alienable and
disposable, because it was a military reservation at the time. Hence, it was not subject to
occupation, entry or settlement.
The land was declared alienable only in 1971; hence, respondents have not satisfied the
thirty-year requirement under the Public Land Act. Moreover, they could not have occupied the
property for thirty years, because it formed part of a military reservation. Clearly then, their
application for the registration of their titles was erroneously granted by the appellate and the trial
courts.
Case No. 14
Topic: Inalienable Property

Republic v. Court of Appeals


FACTS: Private respondent, Isabel Lastimado, filed in the Court of First Instance of
Bataan a Petition for the reopening of cadastral proceedings over a portion of Lot on Mariveles
Cadastre, pursuant to Republic Act No. 931, as amended by Republic Act No. 2061. In the absence
of any opposition, whether from the Government or from private individuals, private respondent
was allowed to present her evidence ex-parte. On October 14, 1967, the trial Court rendered a
Decision granting the Petition and adjudicating the land in favor of private respondent. The trial
Court issued an order for the issuance of a decree of registration on November 20, 1967, and on
November 21, 1967, the Land Registration Commission issued Decree No. N-117573 in favor of
private respondent. Eventually, Original Certificate of Title No. N-144 was also issued in her
favor. Private respondent thereafter subdivided the land into ten lots, and the corresponding titles.
Transfer Certificates of Title were issued by the Register of Deeds. On June 3, 1968, or within one
year from the entry of the decree of registration, petitioner filed a Petition for Review on the ground
of fraud alleging that during the period of alleged adverse possession by private respondent, said
parcel of land was part of the U.S. Military Reservation in Bataan. which was formally turned over
to the Republic of the Philippines only on December 22, 1965, and that the same is inside the
public forest of Mariveles, Bataan and, therefore, not subject to disposition or acquisition under
the Public Land Law.
ISSUE: Whether or not such parcel of land was subject to disposition or acquisition under
Public Land Law.
RULING: The Supreme Court reverse the decision of CA. Although there was an
agreement by the parties to submit for resolution the Opposition to the Petition for Review, which
was treated as a motion to dismiss, the trial Court, in the exercise of sound judicial discretion,
should not have dismissed the Petition outright but should have afforded petitioner an opportunity
to present evidence in support of the facts alleged to constitute actual and extrinsic fraud committed
by private respondent. If the allegation of petitioner that the land in question was inside the military
reservation at the time it was claimed is true, then, it cannot be the object of any cadastral p nor
can it be the object of reopening under Republic Act No. 931. Similarly, if the land in question,
indeed forms part of the public forest, then, possession thereof, however long, cannot convert it
into private property as it is within the exclusive jurisdiction of the Bureau of Forestry and beyond
the power and jurisdiction of the Cadastral Court to register under the Torrens System. Even
assuming that the government agencies can be faulted for inaction and neglect (although the
Solicitor General claims that it received no notice), yet, the same cannot operate to bar action by
the State as it cannot be estopped by the mistake or error of its officials or agents. 11 Further, we
cannot lose sight of the cardinal consideration that "the State as persona in law is the juridical
entity, which is the source of any asserted right to ownership in land" under basic Constitutional
Precepts, and that it is moreover charged with the conservation of such patrimony.
Case No. 15
Topic: Inalienable Property

Apex Mining v. Southeast Mindanao Gold Mining


Facts: The case involves the “Diwalwal Gold Rush Area” (Diwalwal), a rich tract of
mineral land located inside the Agusan-Davao-Surigao Forest Reserve in Davao del Norte and
Davao Oriental. Since the early 1980s, Diwalwal has been stormed by conflicts brought about by
numerous mining claims over it. On March 10, 1986, Marcopper Mining Corporation (MMC) was
granted an Exploration Permit (EP 133) by the Bureau of Mines and Geo-Sciences (BMG). A long
battle ensued between Apex and MMC with the latter seeking the cancellation of the mining claims
of Apex on the ground that such mining claims were within a forest reservation (Agusan-Davao-
Surigao Forest Reserve) and thus the acquisition on mining rights should have been through an
application for a permit to prospect with the BFD and not through registration of a DOL with the
BMG. When it reached the SC in 1991, the Court ruled against Apex holding that the area is a
forest reserve and thus it should have applied for a permit to prospect with the BFD.
On February 16 1994, MMC assigned all its rights to EP 133 to Southeast Mindanao Gold
Mining Corporation (SEM), a domestic corporation which is alleged to be a 100%-owned
subsidiary of MMC. Subsequently, BMG registered SEM’s Mineral Production Sharing
Agreement (MPSA) application and the Deed of Assignment. Several oppositions were filed. The
Panel of Arbitrators created by the DENR upheld the validity of EP 133.
During the pendency of the case, DENR AO No. 2002-18 was issued declaring an
emergency situation in the Diwalwal Gold Rush Area and ordering the stoppage of all mining
operations therein.
Issue: Whether or not EP 133 and its subsequent transfer to SEM is valid
Held: Invalid. One of the terms and conditions of EP 133 is: “That this permit shall be for
the exclusive use and benefit of the permittee or his duly authorized agents and shall be used for
mineral exploration purposes only and for no other purpose.” While it may be true that SEM is a
100% subsidiary corporation of MMC, there is no showing that the former is the duly authorized
agent of the latter. As such, the assignment is null and void as it directly contravenes the terms and
conditions of the grant of EP 133.
a. The Deed of Assignment was a total abdication of MMC’s rights over the permit. It is not a
mere grant of authority to SEM as agent.
b. Reason for the stipulation. Exploration permits are strictly granted to entities or individuals
possessing the resources and capability to undertake mining operations. Without such a condition,
non-qualified entities or individuals could circumvent the strict requirements under the law by the
simple expediency of acquiring the permit from the original permittee.
c. Separate personality. The fact that SEM is a 100% subsidiary of MMC does not automatically
make it an agent of MMC. A corporation is an artificial being invested by law with a personality
separate and distinct from persons composing it as well as from that of any other legal entity to
which it may be related. Absent any clear proof to the contrary, SEM is a separate and distinct
entity from MMC.
d. Doctrine of piercing the corporate veil inapplicable. Only in cases where the corporate fiction
was used as a shield for fraud, illegality or inequity may the veil be pierced and removed. The
doctrine of piercing the corporate veil cannot therefore be used as a vehicle to commit prohibited
acts. The assignment of the permit in favor of SEM is utilized to circumvent the condition of
nontransferability of the exploration permit. To allow SEM to avail itself of this doctrine and to
approve the validity of the assignment is tantamount to sanctioning an illegal act which is what the
doctrine precisely seeks to forestall.
e. PD 463 requires approval of Secretary of DENR. Also, PD 463 (Mineral Resources
Development Decree), which is the governing law when the assignment was executed, explicitly
requires that the transfer or assignment of mining rights, including the right to explore a mining
area, must be with the prior approval of the Secretary of DENR. Such is not present in this case.
f. EP 133 expired by non-renewal. Although EP 133 was extended for 12 months until July 6,
1994, MMC never renewed its permit prior and after its expiration. With the expiration of EP 133
on July 6, 1994, MMC lost any right to the Diwalwal Gold Rush Area. SEM, on the other hand,
has not acquired any right to the said area because the transfer of EP 133 in its favor is invalid.
Hence, both MMC and SEM have not acquired any vested right over the area covered by EP 133.
Case No. 16
Topic: Inalienable Property

Republic v. AFP Retirement and Separation Benefits System


FACTS: Three lots which are of the public domain located in General Santos are the
disputed lands in this case were reserved for recreation and health purposes by Proc. No. 168. A
subsequent Proclamation declared that 2 of the 3 lots were open for disposition to qualified
applicants leaving the remaining lot which is Lot X to be remained part of the reservation now
known as Magsaysay Park.
In 1997, DENR approved respondents-intervenor’s application for issuance of
miscellaneous patents over Lot X and later that year those titles were conveyed to AFP-RSBS.
After a year, petitioner Republic, instituted a civil case for reversion and cancellation against AFP-
RSBS of those titles, on the thesis that they were issued over a public park which is classified as
inalienable and non-disposable public land.
Respondent-intervenors together with AFP-RSBS argued that their predecessors-in-
interest Kusop had acquired vested rights having occupied the same for more than 30 years. They
claimed that these vested rights were alienable and disposable land of public domain, as well as
the subsequent issuance of sales patents in their names. RTC rendered judgment nullifying the
AFP-RSBS titles and ordering the return of Lot X to the Republic. Their private rights, which were
guaranteed under Proc. 168, have already been recognized and respected through the subsequently
issued Proc. 2273; as a consequence, the succeeding sales patents and OCTs in the names of the
respondents-intervenors should be declared null and void not only for being in violation of law,
but also because respondents-intervenors did not deserve to acquire more land.
The CA reversed the RTC decision based on that as a consequence of their predecessor’s
possession of Lot X since time immemorial, respondents-intervenors have acquired title without
need of judicial or other action, and the property ceased to be public land and thus became private
property. Hence, this petition.
ISSUE: Is Lot X alienable and disposable land of the public domain to which respondents-
intervenors have a vested right as a consequence of their predecessor’s possession of it since time
immemorial?
RULING: No. Evidently, the sales patents over Lot X are null and void, for at the time the
sales patents were applied for and granted, the land had lost its alienable and disposable character.
It was set aside and was being utilized for a public purpose, that is, as a recreational park. Under
Section 83 of CA 141, "the President may designate by proclamation any tract or tracts of land of
the public domain as reservations for the use of the Commonwealth of the Philippines or of any of
its branches, or of the inhabitants thereof, in accordance with regulations prescribed for this
purpose, or for quasi-public uses or purposes, when the public interest requires it and under the
present Constitution, national parks are declared part of the public domain, and shall be conserved
and may not be increased nor diminished, except by law.”
Respondents-intervenors no longer had any right to Lot X, not by acquisitive prescription,
and certainly not by sales patent. In fact, their act of applying for the issuance of miscellaneous
sales patents operates as an express acknowledgment that the State, and not respondents-
intervenors, is the owner of Lot X. It is erroneous to suppose that respondents-intervenors
possessed title to Lot X when they applied for miscellaneous sales patents, for the premise of such
grant or privilege is precisely that the State is the owner of the land, and that the applicant
acknowledges this and surrenders to State ownership. The government, as the agent of the State,
is possessed of the plenary power as the persona in law to determine who shall be the favored
recipients of public lands, as well as under what terms they may be granted such privilege, not
excluding the placing of obstacles in the way of their exercise of what otherwise would be ordinary
acts of ownership.
Case No. 17
Topic: Inalienable Property

Celestial vs. Cachopero


Facts: Respondent Cachopero filed a Miscellaneous Sales Application (MSA) with the
Bureau of Lands alleging that he had been occupying the land since 1968 where he built a house
and other improvements. Petitioner Celestial filed a protest against his MSA claiming a
preferential right over the land since it is adjacent to her house. The Bureau of Lands made an
ocular inspection and found out that the land covered by the MSA is outside the commerce of man
since it was part of the creek that became dry as a result of irrigation made by NIA. Therefore, the
Bureau of Land rejected the MSA filed by Cachopero but he is still allowed to occupy the land
until the Government need the land for expansion of the road.
Petitioner filed an ejectment case against the respondent. Subsequently, respondent filed
another MSA with the DENR Regional Office of Cotabato involving a portion of the same lot
subject of his first MSA but this time it is supported by a certification issued by the Office of the
Mayor of Midsayap and an Indorsement by the District Engineer of the Department of Public
Works and Highways stating that the subject land is suitable for residential purposes and no longer
needed by the municipal government. Again, the Petitioner filed a protest claiming a preferential
right over the land. The DENR declared that the lot was suitable for residential purposes but that,
in light of the conflicting interest of the parties, it be sold at public auction. Hence, the DENR
dismissed the MSA of respondent. Cachopero then filed a petition for certiorari, prohibition and
mandamus against the DENR with the Regional Trial Court (RTC) but was denied. On appeal, the
Court of Appeals reversed and set aside the decision of the RTC.
Issue: Whether or not petitioner’s claim under Art. 370 of the Spanish Civil Code of 1889
and/or Article 461 of the Civil Code is enough for her ownership of the land
Ruling: A creek, like the Salunayan Creek, is a recess or arm extending from a river and
participating in the ebb and flow of the sea. As such, under Articles 420(1) and 502(1) 45 of the
Civil Code, the Salunayan Creek, including its natural bed, is property of the public domain which
is not susceptible to private appropriation and acquisitive prescription. And, absent any declaration
by the government, that a portion of the creek has dried-up does not, by itself, alter its inalienable
character. Under Article 370 of the Spanish Civil Code of 1889 which took effect in the Philippines
on December 7, 1889, the beds of rivers which remain abandoned because the course of the water
has naturally changed belong to the owners of the riparian lands throughout their respective
lengths. If the abandoned bed divided estates belonging to different owners, the new dividing line
shall run at equal distance therefrom. Article 461 which abandoned Article 370 provides that
“River beds which are abandoned through the natural change in the course of the waters ipso facto
belong to the owners whose lands are occupied by the new course in proportion to the area lost.
However, the owners of the lands adjoining the old bed shall have the right to acquire the same by
paying the value thereof, which value shall not exceed the value of the area occupied by the new
bed.” The Court declared that both Article 370 of the Old Code and Article 461 of the present Civil
Code are applicable only when" river beds are abandoned through the natural change in the course
of the waters." It is not applicable in this case because the land in dispute is a result of irrigation
made by NIA.
Case No. 18
Topic: Inalienable Property

Republic v. Enciso
FACTS: On April 24, 2000, the respondent, alleging to be the owner in fee simple of a
parcel of residential land located in Barangay South Poblacion, Masinloc, Zambales, filed a
petition for land registration before the RTC of Iba, Zambales.
The respondent averred, inter alia, that he acquired title to the said lot by virtue of an
extrajudicial settlement of estate and quitclaim on March 15, 1999; the said property is not tenanted
or occupied by any person other than the respondent and his family who are in actual physical
possession of the same; and the respondent and his predecessors-in-interest have been in
continuous, peaceful, open, notorious, uninterrupted and adverse possession of the land in the
concept of an owner for not less than 30 years immediately preceding the filing of the application.
The respondent presented tax receipts to show that the property was declared for taxation
purposes in his name. He also testified that he acquired the property by inheritance from his
deceased father, Vicente Enciso, who died on May 18, 1991. He then immediately took possession
of the property and constructed a house thereon in 1991. On March 15, 1999, he and his siblings
executed an extrajudicial settlement of estate where the land was adjudicated in his favor.
The respondent further narrated that the property was originally owned by the Municipality
of Masinloc, Zambales. On October 5, 1968, the municipality passed Resolution No. 71,
undertaking to construct a road along the shoreline of the poblacion, but requiring landowners
adjoining the roads to share in the expenses for an inner wall adjacent to their lots. In view of this,
the same resolution provided that:
The trial court ruled that the respondent satisfactorily proved his ownership in fee simple,
as well as the identity of the land sought to be titled. Likewise, the trial court found that the
respondent, as well as his predecessors-in-interest, had been in open, peaceful, continuous, public,
adverse, and under a bona fide claim of ownership. According to the trial court, there was no
evidence that the subject parcel of land was within any government reservation, or that the
applicant was disqualified from owning real property under the Constitution. The CA disposed of
the appeal on September 26, 2003 and affirmed the decision of the trial court.
The petitioner contends that the first and primordial element in order to warrant the
registration of title is to show that the land must be an alienable and disposable land of the public
domain. On this note, the petitioner believes that the respondent failed to adduce any evidence to
show that the subject land was already previously declared part of such alienable and disposable
land of the public domain. Furthermore, the petitioner adds that under the Regalian doctrine, all
lands of the public domain belong to the State, and those not otherwise appearing to be clearly
within private ownership are presumed to belong to it.
ISSUE: Whether or not the land is alienable and disposable.
RULING: No. The respondent’s possession and that of his "predecessors-in-interest" will
not suffice for purposes of judicial confirmation of title. What is categorically required by law is
open, continuous, exclusive, and notorious possession and occupation under a bona fide claim of
ownership since June 12, 1945 or earlier.
Well-entrenched is the rule that the burden of proof in land registration cases rests on the
applicant who must show clear, positive and convincing evidence that his alleged possession and
occupation were of the nature and duration required by law. Bare allegations, without more, do
not amount to preponderant evidence that would shift the burden to the oppositor.
Evidently, the respondent failed to prove that (1) Lot No. 2278-A was classified as part of
the disposable and alienable land of the public domain; and (2) he and his predecessors-in-interest
have been in open, continuous, exclusive, and notorious possession and occupation thereof in the
concept of owners since time immemorial, or from June 12, 1945.
SEC. 14. Who may apply. –The following persons may file in the proper Court of First
Instance an application for registration of title to land, whether personally or through their duly
authorized representatives:
(1) Those who by themselves or through their predecessors-in-interest have been in open,
continuous, exclusive and notorious possession and occupation of alienable and disposable lands
of the public domain under a bona fideclaim of ownership since June 12, 1945, or earlier.
Applicants for registration of title must therefore prove the following: (a) that the land
forms part of the disposable and alienable lands of the public domain; and (b) that they have been
in open, continuous, exclusive, and notorious possession and occupation of the same under a bona
fide claim of ownership either since time immemorial, or since June 12, 1945. It is not disputed
that the land sought to be registered was originally part of the reclamation project undertaken by
the Municipality of Masinloc, Zambales. The prevailing rule is that reclaimed disposable lands of
the public domain may only be leased and not sold to private parties. These lands remained sui
generis, as the only alienable or disposable lands of the public domain which the government could
not sell to private parties except if the legislature passes a law authorizing such sale. Reclaimed
lands retain their inherent potential as areas for public use or public service. The ownership of
lands reclaimed from foreshore areas is rooted in the Regalian doctrine, which declares that all
lands and waters of the public domain belong to the State. On November 7, 1936, the National
Assembly approved Commonwealth Act No. 141, also known as the Public Land Act, compiling
all the existing laws on lands of the public domain. This remains to this day the existing and
applicable general law governing the classification and disposition of lands of the public domain.
The State policy prohibiting the sale of government reclaimed, foreshore and marshy alienable
lands of the public domain to private individuals continued under the 1935 Constitution.
Case No. 19
Topic: Inalienable property; Who may apply for original registration proceedings? – Those
who acquired ownership of land by accession or accretion

Vda. De Nazareno v. Court of Appeals


Facts: The subject of this controversy is a parcel of land situated in Telegrapo, Puntod,
Cagayan de Oro City. Said land was formed as a result of sawdust dumped into the dried-up
Balacanas Creek and along the banks of the Cagayan river.
Sometime in 1979, private respondents Jose Salasalan and Leo Rabaya leased the subject
lots on which their houses stood from one Antonio Nazareno, petitioners' predecessor-in-interest.
In the latter part of 1982, private respondents allegedly stopped paying rentals. As a result, Antonio
Nazareno and petitioners filed a case for ejectment with the Municipal Trial Court of Cagayan de
Oro City. The decision of the lower court was enforced with the private respondents being ejected
from portions of the subject lots they occupied.
Before he died, Antonio Nazareno caused the approval by the Bureau of Lands of the
survey plan designated as Plan Csd-106-00571 with a view to perfecting his title over the accretion
area being claimed by him. Before the approved survey plan could be released to the applicant,
however, it was protested by private respondents before the Bureau of Lands.
Petitioner’s Contentions:
The subject land is private land being an accretion to his titled property, applying Article
457 of the Civil Code which provides:
To the owners of lands adjoining the banks of rivers belong the accretion which they
gradually receive from the effects of the current of the waters.
Issues: (1) Is the subject land a public land?
(2) Who has the authority to exercise executive control over any form of concession,
disposition and management of the lands of public domain?
Held: (1) Yes.
In Meneses v. CA, the Court held that accretion, as a mode of acquiring property under
Art. 457 of the Civil Code, requires the concurrence of these requisites : (1) that the deposition of
soil or sediment be gradual and imperceptible; (2) that it be the result of the action of the waters
of the river (or sea); and (3) that the land where accretion takes place is adjacent to the banks of
rivers (or the sea coast). These are called the rules on alluvion which if present in a case, give to
the owners of lands adjoining the banks of rivers or streams any accretion gradually received from
the effects of the current of waters.
However, they admit that the accretion was formed by the dumping of boulders, soil and
other filling materials on portions of the Balacanas Creek and the Cagayan River bounding their
land. It cannot be claimed, therefore, that the accumulation of such boulders, soil and other filling
materials was gradual and imperceptible, resulting from the action of the waters or the current of
the Balacanas Creek and the Cagayan River.
Petitioners' submission not having met the first and second requirements of the rules on
alluvion, they cannot claim the rights of a riparian owner.
(2) The Director of Lands.
As borne out by the administrative findings, the controverted land is public land, being an
artificial accretion of sawdust. As such, the Director of Lands has jurisdiction, authority and
control over the same, as mandated under Sections 3 and 4 of the Public Land Law (C.A. No. 141)
which states, thus:
Sec. 3. The Secretary of Agriculture and Natural Resources shall be the exclusive officer charged
with carrying out the provisions of this Act through the Director of Lands who shall act under his
immediate control.
Sec. 4. Subject to said control, the Director of Lands shall have direct executive control of the
survey, classification, lease, sale or any other form of concession or disposition and management
of the lands of the public domain, and his decisions as to questions of fact shall be conclusive when
approved by the Secretary of Agriculture and Natural Resources.
In connection with the second issue, petitioners ascribe whim, arbitrariness or
capriciousness in the execution order of public respondent Abelardo G. Palad, the Director of
Lands. This Court finds otherwise since said decision was based on the conclusive finding that the
subject land was public land. Thus, this Court agrees with the Court of Appeals that the Director
of Lands acted within his rights when he issued the assailed execution order, as mandated by the
aforecited provisions.
Case No. 20
Topic: Who may apply for original registration proceedings? - Those who have been in
OCEN of A & D land under bona fide claim of ownership since June 12, 1945

Republic v. Naguit
FACTS: On January 5, 1993, Naguit filed a petition for registration of title of a parcel of
land. The application sought a judicial confirmation of imperfect title over the land.
The public prosecutor, appearing for the government, and Angeles opposed the petition.
The evidence revealed that the subject parcel of land was originally declared for taxation purposes
in the name of Urbano in 1945. Urbano executed a Deed of Quitclaim in favor of the heirs of
Maming, wherein he renounced all his rights to the subject property and confirmed the sale made
by his father to Maming sometime in 1955 or 1956. Subsequently, the heirs of Maming executed
a deed of absolute sale in favor of respondent Naguit who thereupon started occupying the same.
Naguit constituted Blanco, Jr. as her attorney-in-fact and administrator. The administrator
introduced improvements, planted trees in addition to existing coconut trees which were then 50
to 60 years old, and paid the corresponding taxes due on the subject land. Furthermore, respondent
and her predecessors-in-interest had occupied the land openly and in the concept of owner without
any objection from any private person or even the government until she filed her application for
registration.
The OSG argued that the property which is in open, continuous and exclusive possession
must first be alienable. Since the subject land was declared alienable only on October 15, 1980,
Naguit could not have maintained a bona fide claim of ownership since June 12, 1945, as required
by Section 14 of the Property Registration Decree, since prior to 1980, the land was not alienable
or disposable. In addition, the OSG suggested an interpretation that all lands of the public domain
which were not declared alienable or disposable before June 12, 1945 would not be susceptible to
original registration, no matter the length of unchallenged possession by the occupant.
ISSUE: Is it necessary under Section 14(1) of the Property Registration Decree that the
subject land be first classified as alienable and disposable before the applicant’s possession under
a bona fide claim of ownership could even start?
RULING: Section 14 of the Property Registration Decree, governing original registration
proceedings, provides:
SECTION 14. Who may apply— The following persons may file in the proper Court of First
Instance an application for registration of title to land, whether personally or through their duly
authorized representatives:
(1) those who by themselves or through their predecessors-in-interest have been in open,
continuous, exclusive and notorious possession and occupation of alienable and disposable lands
of the public domain under a bona fide claim of ownership since June 12, 1945, or earlier.
(2) Those who have acquired ownership over private lands by prescription under the provisions of
existing laws.
There are three obvious requisites for the filing of an application for registration of title
under Section 14(1) – that the property in question is alienable and disposable land of the public
domain; that the applicants by themselves or through their predecessors-in-interest have been in
open, continuous, exclusive and notorious possession and occupation, and; that such possession is
under a bona fide claim of ownership since June 12, 1945 or earlier. The more reasonable
interpretation of Section 14(1) is that it merely requires the property sought to be registered as
already alienable and disposable at the time the application for registration of title is filed. If the
State, at the time the application is made, has not yet deemed it proper to release the property for
alienation or disposition, the presumption is that the government is still reserving the right to utilize
the property; hence, the need to preserve its ownership in the State irrespective of the length of
adverse possession even if in good faith. However, if the property has already been classified as
alienable and disposable, as it is in this case, then there is already an intention on the part of the
State to abdicate its exclusive prerogative over the property.
In this case, the 3 requisites for the filing of registration of title under Section 14(1) had
been met by Naguit. The parcel of land had been declared alienable; Naguit and her predecessors-
in-interest had been in open, continuous, exclusive and notorious possession and occupation of the
land evidenced by the 50 to 60-year old trees at the time she purchased the property; as well as the
tax declarations executed by the original owner Urbano in 1954, which strengthened one's bona
fide claim of ownership.
Case No. 21
Topic: Who may apply for original registration proceedings? - Those who have been in
OCEN of A & D land under bona fide claim of ownership since June 12, 1945

Heirs of Mario Mabanan v. Republic


FACTS: The property subject of the application for registration is a parcel of land situated
in Barangay Tibig, Silang Cavite, more particularly identified as Lot 9864-A, Cad-452-D, with an
area of 71,324-square meters. On February 20, 1998, applicant Mario Malabanan, who had
purchased the property from Eduardo Velazco, filed an application for land registration covering
the property in the Regional Trial Court (RTC) in Tagaytay City, Cavite, claiming that the property
formed part of the alienable and disposable land of the public domain, and that he and his
predecessors-in-interest had been in open, continuous, uninterrupted, public and adverse
possession and occupation of the land for more than 30 years, thereby entitling him to the judicial
confirmation of his title.
To prove that the property was an alienable and disposable land of the public domain,
Malabanan presented during trial a certification dated June 11, 2001 issued by the Community
Environment and Natural Resources Office (CENRO) of the Department of Environment and
Natural Resources (DENR), which reads: “This is to certify that the parcel of land designated as
Lot No. 9864 Cad 452-D, Silang Cadastre as surveyed for Mr. Virgilio Velasco located at
Barangay Tibig, Silang, Cavite containing an area of 249,734 sq. meters as shown and described
on the Plan Ap-04-00952 is verified to be within the Alienable or Disposable land per Land
Classification Map No. 3013 established under Project No. 20-A and approved as such under FAO
4-1656 on March 15, 1982. “
After trial, on December 3, 2002, the RTC rendered judgment granting Malabanan’s application
for land registration.
The Office of the Solicitor General (OSG) appealed the judgment to the CA, arguing that
Malabanan had failed to prove that the property belonged to the alienable and disposable land of
the public domain, and that the RTC erred in finding that he had been in possession of the property
in the manner and for the length of time required by law for confirmation of imperfect title.
On February 23, 2007, the CA promulgated its decision reversing the RTC and dismissing
the application for registration of Malabanan. Citing the ruling in Republic v. Herbieto (Herbieto),
the CA declared that under Section 14(1) of the Property Registration Decree, any period of
possession prior to the classification of the land as alienable and disposable was inconsequential
and should be excluded from the computation of the period of possession. Noting that the CENRO-
DENR certification stated that the property had been declared alienable and disposable only on
March 15, 1982, Velazco’s possession prior to March 15, 1982 could not be tacked for purposes
of computing Malabanan’s period of possession.
ISSUE: Is the petitioner has the right to register the land in accordance with Section 14 of
Property Registration Decree?
Held: No. The applicant must satisfy the following requirements in order for his
application to come under Section 14(1) of the Property Registration Decree, to wit:
1. The applicant, by himself or through his predecessor-in-interest, has been in possession and
occupation of the property subject of the application;
2. The possession and occupation must be open, continuous, exclusive, and notorious;
3. The possession and occupation must be under a bona fide claim of acquisition of ownership;
4. The possession and occupation must have taken place since June 12, 1945, or earlier; and
5. The property subject of the application must be an agricultural land of the public domain.
The agricultural land subject of the application needs only to be classified as alienable and
disposable as of the time of the application, provided the applicant’s possession and occupation of
the land dated back to June 12, 1945, or earlier. The petitioners failed to present sufficient evidence
to establish that they and their predecessors-in-interest had been in possession of the land since
June 12, 1945. Without satisfying the requisite character and period of possession - possession and
occupation that is open, continuous, exclusive, and notorious since June 12, 1945, or earlier - the
land cannot be considered ipso jure converted to private property even upon the subsequent
declaration of it as alienable and disposable. Prescription never began to run against the State, such
that the land has remained ineligible for registration under Section 14(1) of the Property
Registration Decree. Likewise, the land continues to be ineligible for land registration under
Section 14(2) of the Property Registration Decree unless Congress enacts a law or the President
issues a proclamation declaring the land as no longer intended for public service or for the
development of the national wealth.
Case No. 22
Topic: Who may apply for original registration proceedings? - Those who have been in
OCEN of A & D land under bona fide claim of ownership since June 12, 1945; Those who
acquired ownership of private land by prescription

Lim v. Republic
FACTS: Joyce Lim, petitioner, filed on September 7, 1998 before the RTC of Tagaytay
City an Application for Registration of Title over two parcels of land in Cavite, declaring that she
purchased both lots on April 30, 1997 from Spouses Edgardo and Jorgina Pagkalinawan (Spouses
Pagkalinawan) as evidenced by a Kasulatan ng Bilihang Lubusan ng Lupa. Petitioner sought the
application of the Property Registration Decree for both applications, claiming that she and her
predecessors-in-interest Trinidad Mercado, Fernanda Belardo, Victoria Abueg and the Spouses
Pagkalinawan have been in open, continuous, exclusive and notorious possession and occupancy
of the lots under a bona fide claim of ownership for more than thirty (30) years since 1941.
The RTC granted petitioners application, but the Solicitor General, respondent, appealed
the decisions to the CA on the ground that petitioner failed to comply with the provisions of the
Property Registration Decree and Article 1137 of the Civil Code both laws of which require at
least 30 years of adverse possession. Respondent emphasizes that the lots were classified to be
alienable and disposable only on March 15, 1982, citing the CENRO February 3, 1999
Certifications, hence, petitioner’s possession or occupancy of the lots could only be reckoned from
said date onwards.
The CA reversed and set aside the RTC decision, hence, this petition.
ISSUE: Can the petitioner acquire ownership over the subject parcels of land by
prescription under a bona fide claim of ownership as provided in the Property Registration Decree?
RULING: No. Under Section 14 (1) of Property Registration Decree, the requisites for the
filing of an application for registration of title are: that the property in question is alienable and
disposable land of the public domain; that the applicants by themselves or through their
predecessors-in-interest have been in open, continuous, exclusive and notorious possession and
occupation; and that such possession is under a bona fide claim of ownership since June 12, 1945
or earlier.
As the Solicitor General proffers, the alienable and disposable character of the lots should
have already been established on June 12, 1945 or earlier; and given that they were declared
alienable only on March 15, 1982, as reflected in the CENRO Certifications, petitioner could not
have maintained a bona fide claim of ownership since June 12, 1945 or earlier.
While a property classified as alienable and disposable public land may be converted into
private property by reason of open, continuous, exclusive and notorious possession of at least 30
years, public dominion lands become patrimonial property not only with a declaration that these
are alienable or disposable but also with an express government manifestation that the property is
already patrimonial or no longer retained for public use, public service or the development of
national wealth. And only when the property has become patrimonial can the prescriptive period
for the acquisition of property of the public dominion begin to run.
While the subject lots were declared alienable or disposable on March 15, 1982, there is
no competent evidence that they are no longer intended for public use or for public service. The
classification of the lots as alienable and disposable lands of the public domain does not change
its status as properties of the public dominion. Petitioner cannot thus acquire title to them by
prescription.
Case No. 23
Topic: Who may apply for original registration proceedings? -Those who acquired
ownership of private land by prescription
Tan v. Republic
FACTS: On June 14, 2001, the petitioners filed with the RTC of Naic, Cavite, an
application for land registration covering a parcel of land situated in Barangay Bancod, Indang,
Cavite and with an area of 6,920 square meters. The petitioners alleged that they acquired the
subject property from Gregonio Gatdula pursuant to a Deed of Absolute Sale dated April 25, 1996;
and they and their predecessors-in-interest have been in open, continuous and exclusive possession
of the subject property in the concept of an owner for more than 30 years.
The RTC granted the petitioner’s application which the respondent Republic appealed to
the CA. The CA ruled in favor of the respondent stating that the appellees or their predecessors-
in-interest possession over the subject property can be reckoned only from 21 June 1983, the date
when according to evidence, the subject property became alienable and disposable. From said date
up to the filing of the application for registration of title over the subject property on 14 June 2001,
only eighteen (18) years had lapsed. Thus, possession of the subject property fell short of the
requirement of open, continuous and exclusive possession, in the concept of an owner, of at least
30 years. Hence, this petition.
ISSUE: Did the petitioners acquire ownership of the subject property by prescription as
provider under Section 14(2) of the Property Registration Decree?
RULING: No. For one to invoke the provisions of Section 14(2) and set up acquisitive
prescription against the State, it is primordial that the status of the property as patrimonial be first
established because the private property contemplated in Section 14(2) is patrimonial property as
defined in Article 421 in relation to Articles 420 and 422 of the Civil Code.
In Heirs of Malabanan, this Court ruled that possession and occupation of an alienable and
disposable public land for the periods provided under the Civil Code do not automatically convert
said property into private property or release it from the public domain. There must be an express
declaration that the property is no longer intended for public service or development of national
wealth. Without such express declaration, the property, even if classified as alienable or
disposable, remains property of the State, and thus, may not be acquired by prescription.
Case No. 24
Topic: Who may apply for original registration proceedings? -Those who acquired land
ownership by accession or accretion; The survey Plan (Tracing Cloth Plan)

Republic v. Santos
Facts: Alleging continuous and adverse possession of more than ten years, respondent
Arcadio Ivan A. Santos III (ArcadioIvan) applied on March 7, 1997 for the registration of Lot
4998-B (the property) in the Regional Trial Court (RTC) inParafiaque City. The property, which
had an area of 1,045 square meters, more or less, was located in BarangaySan Dionisio, Parañaque
City, and was bounded in the Northeast by Lot 4079 belonging to respondent Arcadio C.Santos,
Jr. (Arcadio, Jr.), in the Southeast by the Parañaque River, in the Southwest by an abandoned road,
and inthe Northwest by Lot 4998-A also owned by Arcadio Ivan.
On May 21, 1998, Arcadio Ivan amended his application for land registration to include
Arcadio, Jr. as his co-applicant because of the latter’s co-ownership of the property. He alleged
that the property had been formedthrough accretion and had been in their joint open, notorious,
public, continuous and adverse possession for morethan 30 years.
The City of Parañaque (the City) opposed the application for land registration, stating that
it needed the property for its flood control program; that the property was within the legal
easement of 20 meters from the river bank; and that assuming that the property was not covered
by the legal easement, title to the property could not be registered in favor of the applicants for the
reason that the property was an orchard that had dried up and had not resulted from accretion .
On May 10, 2000 the RTC granted the application for land registration,
disposing:WHEREFORE, the Court hereby declares the applicants, ARCADIO IVAN A.
SANTOS, III and ARCADIO C. SANTOS, JR.,both Filipinos and of legal age, as the TRUE and
ABSOLUTE OWNERS of the land being applied for which is situatedin the Barangay of San
Dionisio, City of Parañaque with an area of one thousand forty five (1045) square metersmore or
less and covered by Subdivision Plan Csd-00-000343, being a portion of Lot 4998, Cad. 299, Case
4,Parañaque Cadastre, LRC Rec. No. and orders the registration of Lot 4998-B
With this, the Republic, through the Office of the Solicitor General (OSG), appealed. The
CA grossly erred in applying Article 457 of the Civil Code to respondents’ benefit.
Article 457 of the Civil Code provides that "(t)o the owners of lands adjoining the banks
of rivers belong the accretion which they gradually receive from the effects of the currents of the
waters.
"In ruling for respondents, the RTC pronounced that on the basis of the evidence presented
by the applicants, the Court finds that Arcadio Ivan A. Santos III and Arcadio C. Santos, Jr., are
the owners of the land subject of this application which was previously a part of the Parañaque
River which became an orchard after it dried up and further considering that Lot 4 which adjoins
the same property is owned by applicant, Arcadio C. Santos, Jr., after itwas obtained by him
through inheritance from his mother, Concepcion Cruz, now deceased.
The CA upheld the RTC’s pronouncement, and stated that it could not be denied that "to
the owners of the landsadjoining the banks of rivers belong the accretion which they gradually
receive from the effects of the current of thewaters" (Article 457 New Civil Code) as in this case,
Arcadio Ivan Santos III and Arcadio Santos, Jr., are the ownersof the land which was previously
part of the Parañaque River which became an orchard after it dried up and considering that Lot 4
which adjoins the same property is owned by the applicant which was obtained by the latter from
his mother.
The Republic submits, however, that the application by both lower courts of Article 457 of
the Civil Code waserroneous in the face of the fact that respondents’ evidence did not establish
accretion, but instead the drying up of the Parañaque River.
Issue: Whether or not respondents could claim the property by virtue of acquisitive
prescription (section 14(1) of PD 1529)

Held: NO. (By law, accretion - the gradual and imperceptible deposit made through the
effects of the current of the water-belongs to the owner of the land adjacent to the banks of rivers
where it forms. The drying up of the river is notaccretion. Hence, the dried-up river bed belongs
to the State as property of public dominion, not to the riparian owner, unless a law vests the
ownership in some other person.)Respondents as the applicants for land registration carried the
burden of proof to establish the merits of their application by a preponderance of evidence, by
which is meant such evidence that is of greater weight, or more
Case No. 25
Topic: Who may apply for original registration proceedings? -Those who acquired land
ownership by accession or accretion

Bagaipo v. Court of Appeals


Facts: Petitioner Dionisia P. Bagaipo is the registered owner of Lot No. 415, a 146,900
square meter agricultural land situated in Ma-a, Davao City under Transfer Certificate of Title No.
T-1575.
Respondent Leonor Lozano is the owner of a registered parcel of land located across and
opposite the southeast portion of petitioner’s lot facing the Davao River.
Bagaipo filed a complaint for Recovery of Possession with Mandatory Writ of Preliminary
Injunction and Damages against Lozano for:
(1) the surrender of possession by Lozano of a certain portion of land measuring 29,162 square
meters which is supposedly included in the area belonging to Bagaipo under TCT No. T-15757;
and
(2) the recovery of a land area measuring 37,901 square meters which Bagaipo allegedly lost when
the Davao River traversed her property.
Petitioner’s Contention: As a result of a change in course of the said river, her property
became divided into three lots, namely: Lots 415-A, 415-B and 415-C.
Private Respondent’s Contention: The land claimed by Bagaipo is actually an accretion
to their titled property. He asserted that the Davao River did not change its course and that the
reduction in Bagaipos domain was caused by gradual erosion due to the current of the Davao River.
Lozano added that it is also because of the rivers natural action that silt slowly deposited and added
to his land over a long period of time. He further averred that this accretion continues up to the
present and that registration proceedings instituted by him over the alluvial formation could not be
concluded precisely because it continued to increase in size.
Issues: (1) Was there change in course of the Davao River such that petitioner owns the
abandoned river bed pursuant to Article 461 of the Civil Code?
(2) Did private respondent, Leonor Lozano, own Lot 415-C in accordance with the principle of
accretion under Article 457?
Held: (1) No. The decrease in petitioners land area and the corresponding expansion of
respondent’s property were the combined effect of erosion and accretion respectively. Art. 461 of
the Civil Code is inapplicable. Petitioner cannot claim ownership over the old abandoned riverbed
because the same is inexistent. The riverbeds former location cannot even be pinpointed with
particularity since the movement of the Davao River took place gradually over an unspecified
period of time, up to the present.
The rule is well-settled that accretion benefits a riparian owner when the following
requisites are present: 1) That the deposit be gradual and imperceptible; 2) That it resulted from
the effects of the current of the water; and 3) That the land where accretion takes place is adjacent
to the bank of the river. These requisites were sufficiently proven in favor of respondents. In the
absence of evidence that the change in the course of the river was sudden or that it occurred through
avulsion, the presumption is that the change was gradual and was caused by alluvium and erosion.
(2) NO. In C.N. Hodges vs. Garcia, 109 Phil. 133, 135:
The fact that the accretion to his land used to pertain to plaintiffs estate, which is covered
by a Torrens certificate of title, cannot preclude him (defendant) from being the owner thereof.
Registration does not protect the riparian owner against the diminution of the area of his land
through gradual changes in the course of the adjoining stream. Accretions which the banks of rivers
may gradually receive from the effect of the current become the property of the owners of the
banks (Art. 366 of the old Civil Code; Art. 457 of the new). Such accretions are natural incidents
to land bordering on running streams and the provisions of the Civil Code in that respect are not
affected by the Land Registration Act.
Case No. 26
Topic: Who may apply for original registration proceedings? -Those who acquired land
ownership by accession or accretion

Heirs of Emiliano Navarro v. Intermediate Appellate Court


FACTS: Sometime in the early part of 1960, Sinforoso Pascual filed an application to
register and confirm his title to a parcel of land, situated in Sibocon, Balanga, Bataan. Pascual
claimed that this land is an accretion to his property, situated in Barrio Puerto Rivas, Balanga,
Bataan, and covered by Original Certificate of Title No. 6830. It is bounded on the eastern side by
the Talisay River, on the western side by the Bulacan River, and on the northern side by the Manila
Bay. The Talisay River as well as the Bulacan River flow downstream and meet at the Manila Bay
thereby depositing sand and silt on Pascual's property resulting in an accretion thereon. Sinforoso
Pascual claimed the accretion as the riparian owner.
On March 25, 1960, the Director of Lands, represented by the Assistant Solicitor General,
filed an opposition thereto stating that neither Pascual nor his predecessors-in-interest possessed
sufficient title to the subject property, the same being a portion of the public domain and, therefore,
it belongs to the Republic of the Philippines. The Director of Forestry, through the Provincial
Fiscal, similarly opposed Pascual's application for the same reason as that advanced by the Director
of Lands. Later on, however, the Director of Lands withdrew his opposition. The Director of
Forestry become the sole oppositor.
Then a new party surfaced. Mr Emiliano Navarro jumped into the fray opposing the same
application, stating the he leased part of the property sought to be registered. He sought to protect
his fishpond that rested on the same property. Sinforoso was not amused and filed ejectment
against Mr. Navarro, claiming that Navarro used stealth force and strategy to occupy a portion of
his land. Pascual lost the case against Navarro so he appealed. During the appeal, his original land
registration case was consolidated and tried jointly. The heirs of Pascual took over the case.
Lower courts ruling: On 1975, the court decided that the property was foreshore land and
therefore part of public domain. The RTC dismissed the complaint of Pascual for ejectment against
Navarro and also denied his land registration request. Pascual’s heirs appealed and the RTC was
reversed by the IAC. The Apellate court granted petition for registration. The accretion was caused
by the two rivers, not manila bay. Hence it wasn’t foreshore land. Aggrieved, the Director of
Forestry moved for reconsideration. The Apellate court denied all motions of the Director and the
Government.
ISSUE: May the land sought to be registered be deemed an accretion?
RULING: No. It cannot be registered. This is land of Public domain. Pascual claimed
ownership under Article 457 of the Civil Code saying that the disputed 14-hectare land is an
accretion caused by the joint action of the Talisay and Bulacan Rivers Art 457: Accretion as a
mode of acquiring property and requires the concurrence of the following requisites: (1) that the
accumulation of soil or sediment be gradual and imperceptible; (2) that it be the result of the action
of the waters of the river; and (3) that the land where the accretion takes place is adjacent to the
bank of the river.
Unfortunately, Pasucal and Heirs claim of ownership based on Art 457 is misplaced. If
there’s any land to be claimed, it should be land ADJACENT to the rivers Talisay and Bulacan.
The law is clear on this. Accretion of land along the river bank may be registered. This is not the
case of accretion of land on the property adjacent to Manila Bay. Furthermore, Manila Bay is a
sea. Accretion on a sea bank is foreshore land and the applicable law is not Art 457 but Art 4 of
the Spanish Law of Waters of 1866. This law, while old, holds that accretion along sea shore
cannot be registered as it remains public domain unless abandoned by government for public use
and declared as private property capable of alienation.
Case No. 27
Topic: Who may apply for original registration proceedings? -Those who acquired land
ownership by accession or accretion

MAXIMO JAGUALING, ANUNCITA JAGUALING and MISAMIS ORIENTAL


CONCRETE PRODUCTS, INC. v. COURT OF APPEALS (FIFTEENTH DIVISION),
JANITA F. EDUAVE and RUDYGONDO EDUAVE
FACTS: The parties to this case dispute the ownership of a certain parcel of land located
in Sta. Cruz, Tagoloan, Misamis Oriental with an area of 16,452 square meters, more or less,
forming part of an island in a non-navigable river.
The appellant [private respondent Janita Eduave] claims that she inherited the land from
his father, Felomino Factura, together with his co-heirs, Reneiro Factura and Aldenora Factura,
and acquired sole ownership of the property by virtue of a Deed of Extra Judicial Partition with
sale.The land is declared for tax purposes under Tax Decl. No. 26137 with an area of 16,452 square
meters more or less. Since the death of her father on May 5, 1949, the appellant had been in
possession of the property although the tax declaration remains in the name of the deceased
father.The appellants further state that the entire land had an area of 16,452 square meters
appearing in the deed of extrajudicial partition, while in [the] tax declaration the area is only 4,937
square meters, and she reasoned out that she included the land that was under water. The land was
eroded sometime in November 1964 due to typhoon Ineng, destroying the bigger portion and the
improvements leaving only a coconut tree. In 1966 due to the movement of the river deposits on
the land that was not eroded increased the area to almost half a hectare and in 1970 the appellant
started to plant bananas.
In 1973 the defendants-appellees [petitioners herein] asked her permission to plant corn
and bananas provided that they prevent squatters to come to the area.The appellant engaged the
services of a surveyor who conducted a survey and placed concrete monuments over the land. The
appellant also paid taxes on the land in litigation, and mortgaged the land to the Luzon Surety and
Co., for a consideration of P6,000.00.
The land was the subject of a reconveyance case, in the Court of First Instance of Misamis
Oriental, Branch V, at Cagayan de Oro City, Civil Case No. 5892, between the appellant Janita
Eduave vs. Heirs of Antonio Factura which was the subject of judgment by compromise in view
of the amicable settlement of the parties, dated May 31, 1979.
That the heirs of Antonio Factura, who are presently the defendants-appellees in this case
had ceded a portion of the land with an area of 1,289 square meters more or less, to the appellant,
Janita Eduave, in a notarial document of conveyance, pursuant to the decision of the Court of First
Instance, after a subdivision of the lot No. 62 Pls-799, and containing 1,289 square meters more
or less was designated as Lot No. 62-A and the subdivision plan was approved as Pls-799-Psd-10-
001782.
Appellant also applied for concession with the Bureau of Mines to extract 200 cubic meters
of gravel; and after an ocular inspection the permit was granted.That the appellant after permit was
granted entered into an agreement with Tagoloan Aggregates to extract sand and gravel, which
agreement was registered in the office of the Register of Deeds;
The defendants-appellees [petitioners herein] denied the claim of ownership of the
appellant, and asserted that they are the real owners of the land in litigation containing an area of
18,000 square meters more or less. During the typhoon Ineng in 1964 the river control was washed
away causing the formation of an island, which is now the land in litigation. The defendants started
occupying the land in 1969, paid land taxes as evidenced by tax declaration No. 26380 and tax
receipts, and tax clearances. Photographs showing the actual occupation of the land by the
defendants including improvements and the house were presented as evidence. The report of the
Commissioner who conducted the ocular inspection was offered as evidence of the defendants .
The sketch plan prepared by Eng. Romeo Escalderon shows that the plaintiffs' [private
respondents'] land was across the land in and in going to the land of the plaintiff, one has to cross
a distance of about 68 meters of the Tagoloan river to reach the land in litigation.
On 17 July 1987 the trial court dismissed the complaint for failure of private respondents
as plaintiffs therein to establish by preponderance of evidence their claim of ownership over the
land in litigation. On appeal to the Court of Appeals, respondent court found that the island was
formed by the branching off of the Tagoloan River and subsequent thereto the accumulation of
alluvial deposits. Basing its ruling on Articles 463 and 465 of the Civil Code the Court of Appeals
reversed the decision of the trial court, declared private respondents as the lawful and true owners
of the land subject of this case and ordered petitioners to vacate the premises and deliver possession
of the land to private respondents.
ISSUE: Between the one who has actual possession of an island that forms in a non-
navigable and non-flotable river and the owner of the land along the margin nearest the island,
who has the better right thereto?
HELD: The parcel of land in question is part of an island that formed in a non-navigable
and non-flotable river; from a small mass of eroded or segregated outcrop of land, it increased to
its present size due to the gradual and successive accumulation of alluvial deposits. In this regard
the Court of Appeals also did not err in applying Article 465 of the Civil Code. Under this
provision, the island belongs to the owner of the land along the nearer margin as sole owner
thereof; or more accurately, because the island is longer than the property of private respondents,
they are deemed ipso jure to be the owners of that portion which corresponds to the length of their
property along the margin of the river.
What then, about the adverse possession established by petitioners? Are their rights as such
not going to be recognized? It is well-settled that lands formed by accretion belong to the riparian
owner. This preferential right is, under Article 465, also granted the owners of the land located in
the margin nearest the formed island for the reason that they are in the best position to cultivate
and attend to the exploitation of the same. In fact, no specific act of possession over the accretion
is required. If, however, the riparian owner fails to assert his claim thereof, the same may yield to
the adverse possession of third parties, as indeed even accretion to land titled under the torrens
system must itself still be registered.
Petitioners may therefore, acquire said property by adverse possession for the required
plumber of years under the doctrine of acquisitive prescription. Their possession cannot be
considered in good faith, however, because they are presumed to have notice of the status of private
respondents as riparian owners who have the preferential right to the island as recognized and
accorded by law; they may claim ignorance of the law, specifically Article 465 of the Civil Code,
but such is not, under Articles 3 and 526 of the same code, an adequate and valid defense to support
their claim of good faith. Hence, not qualifying as possessors in good faith, they may acquire
ownership over the island only through uninterrupted adverse possession for a period of thirty
years. By their own admission, petitioners have been in possession of the property for only about
fifteen years. Thus, by this token and under the theory adopted by petitioners, the island cannot be
adjudicated in their favor.
Case No. 28
Topic: Judicial confirmation of imperfect or incomplete title

Del Rosario-Igtiben v. Republic


FACTS: On 08 January 1998, petitioners filed with the trial court an application for
registration of a parcel of land in Cavite. Petitioners alleged that they acquired the Subject Property
by purchase, and that they, by themselves and through their predecessors-in-interest, had been in
actual, continuous, uninterrupted, open, public, and adverse possession of the Subject Property in
the concept of owner for more than 30 years. Based on the testimonial and documentary evidence
presented, the trial court traced the history of possession of the Subject Property back to 1958 up
to 1998.
The trial court rendered a decision approving the petitioners’ application for registration,
but the Republic of the Philippines, represented by the Office of the Solicitor General, appealed
the decision to the Court of Appeals, alleging that the trial court erred in approving the application
for registration despite petitioners’ failure to prove open, continuous, exclusive and notorious
possession and occupation of the Subject Property since 12 June 1945, or earlier, as required by
Section 48(b) of Commonwealth Act No. 141, otherwise known as the Public Land Act, as
amended by PD No. 1073. Also, it was alleged that petitioners also failed to produce muniments
of title to tack their possession to those of their predecessors-in-interest in compliance with the
prescriptive period required by law.
The CA set aside the trial court’s decision, and dismissed the petitioner’s application,
hence, this petition.
ISSUE: Are the petitioners entitled to confirmation and registration of their incomplete or
imperfect title to the Subject Property?
RULING: No. An applicant for judicial confirmation of an imperfect or incomplete title
under the Public Land Act must be able to prove that: (1) the land is alienable public land; and (2)
his open, continuous, exclusive and notorious possession and occupation of the same must either
be since time immemorial or for the period prescribed in the Public Land Act.
The confirmation of imperfect or incomplete titles may be done two ways, either by: (a)
administrative legalization or free patents under Chapter VII of the Public Land Act; or (b) judicial
legalization or judicial confirmation of imperfect or incomplete titles under Chapter VIII of the
same Act.
Section 44 of the Public Land Act, as amended by RA No. 6940, which provides for a
prescriptive period of thirty (30) years possession, applies only to applications for free patents; but
the case at bar is a judicial application for confirmation of an imperfect or incomplete title over
the Subject Property covered by Section 48(b) of the Public Land Act; and Section 48(b) of the
Public Land Act requires for judicial confirmation of an imperfect or incomplete title the
continuous possession of the land since 12 June 1945, or earlier, which petitioners herein failed to
comply with.
Case No. 29
Topic: Judicial confirmation of imperfect or incomplete title; The Survey Plan (Tracing
Cloth Plan)

Republic vs Espinosa
Facts: On March 3, 1999, respondent Espinosa filed with the MTC an application for land
registration covering a parcel of land with an area of 5,525 square meters situated in Barangay
Cabangahan, Consolacion, Cebu. Espinosa alleged that the property is alienable and disposable
that he purchased the property from his mother Isabel which the other heirs waived their rights
and he and his predecessor-in-interest had been in possession of the property in the concept of an
owner for more than 30 years. Moreover, Espinosa presented an Advance Survey Plan to prove
the identity of the land and to identify the land as alienable and disposable he marked as evidence
the annotation thereof made by the chief of map projection section. He also presented 2 tax
declaration for 1965 and 1974 in Isabel’s name proving the that she had been in possession of the
land since 1965. Petitioner opposed Espinosa’s application, claiming that Section 48(b) of
Commonwealth Act No. 141 otherwise known as the "Public Land Act" had not been complied
with as Espinosa’s predecessor-in-interest possessed the property only after June 12, 1945; and the
tax declarations do not prove that his possession and that of his predecessor-in-interest are in the
character and for the length of time required by law. The MTC granted the petition of Espinosa for
satisfying all the requirements such as the advance survey plan for the possession of the land.
Petitioner appealed to the CA and pointed Espinosa’s failure to prove that his possession and that
of his predecessor-in-interest were for the period required by law. The CA dismissed petitioner’s
appeal and affirmed the MTC decision.
Issue: (1) Whether or not the advanced survey plan substantially complies with
Section 17 of P.D. No. 1529
(2) Whether or not notation on the blueprint copy of the plan made by the geodetic engineer
who conducted the survey sufficed to prove that the land applied for is alienable and disposable.
Ruling: All lands of the public domain belong to the State, which is the source of any
asserted right to any ownership of land. All lands not appearing to be clearly within private
ownership are presumed to belong to the State. Accordingly, public lands not shown to have been
reclassified or released as alienable agricultural land, or alienated to a private person by the State,
remain part of the inalienable public domain. To prove that the land in question formed part of the
alienable and disposable lands of the public domain, petitioners relied on the printed words which
was written by a surveyor geodetic engineer which read: "This survey plan is inside Alienable and
Disposable Land Area” which is not sufficient. Such notation does not constitute a positive
government act validly changing the classification of the land in question. a mere surveyor has no
authority to reclassify lands of the public domain. By relying solely on the said surveyor’s
assertion, petitioners have not sufficiently proven that the land in question has been declared
alienable. The blueprint copy of the advanced survey plan may be admitted as evidence of the
identity and location of the subject property if: (a) it was duly executed by a licensed geodetic
engineer; (b) it proceeded officially from the Land Management Services (LMS) of the DENR;
and (c) it is accompanied by a technical description of the property which is certified as correct by
the geodetic surveyor who conducted the survey and the LMS of the DENR. However, while such
blueprint copy of the survey plan may be offered as evidence of the identity, location and the
boundaries of the property applied for, the notation therein may not be admitted as evidence of
alienability and disposability.
Case No. 30
Topic: Judicial persons or private corporations as applicants for land registration

Republic v. T.A.N. Properties


Facts: In 1999, T.A.N. Properties filed in the RTC of Batangas an application for the
registration of a land, located at Sto. Tomas, Batangas and with an area of 56.4007 hectares. To
support its application, it submitted two certificates, issued by CENRO and FMS-DENR and both
certifying that the land applied for was alienable and disposable.
The Republic of the Philippines, represented by the Director of Lands, opposed the
application on the ground that T.A.N. Properties did not prove that the land was alienable and
disposable.
Issue: Whether or not the applicant proved that, the land is alienable and disposable.
Ruling: No. It is the burden of the applicant to prove that the land subject to registration is
alienable and disposable and for such the applicant must prove that the DENR Secretary had
approved the land classification and released the land of the public domain as alienable and
disposable.
In the present case, T.A.N. Properties did not provide the needed proof. For the documents
provided by the company, the Court cited DENR Administrative Order No. 20 (DAO No. 20) and
DAO No. 38; DAO No. 20 proves that FMS-DENR has no authority to issue certificates,
classifying lands to be alienable and disposable; and DAO No. 38 provides that CENRO can issue
certificates of land classification for lands having a maximum area of 50 hectares. The land applied
for in the case has an area of 56.4007 hectares, thus CENRO has no jurisdiction over it. It is clear
from the aforementioned DAO’s that the documents submitted by T.A.N. Properties did not prove
that the land is alienable and disposable.
Case No. 31
Topic: Judicial persons or private corporations as applicants for land registration

Chavez v. PEA and AMARI


Facts: In 1973, the Comissioner on Public Highways entered into a contract to reclaim
areas of Manila Bay with the Construction and Development Corportion of the Philippines
(CDCP).
PEA (Public Estates Authority) was created by President Marcos under P.D. 1084, tasked
with developing and leasing reclaimed lands. These lands were transferred to the care of PEA
under P.D. 1085 as part of the Manila Cavite Road and Reclamation Project (MCRRP). CDCP and
PEA entered into an agreement that all future projects under the MCRRP would be funded and
owned by PEA.
By 1988, President Aquino issued Special Patent No. 3517 transferring lands to PEA. It
was followed by the transfer of three Titles (7309, 7311 and 7312) by the Register of Deeds of
Paranaque to PEA covering the three reclaimed islands known as the FREEDOM ISLANDS.
Subsequently, PEA entered into a joint venture agreement (JVA) with AMARI, a Thai-
Philippine corporation to develop the Freedom Islands. Along with another 250 hectares, PEA and
AMARI entered the JVA which would later transfer said lands to AMARI. This caused a stir
especially when Sen. Maceda assailed the agreement, claiming that such lands were part of public
domain.
Petitioner Frank J. Chavez filed case as a taxpayer praying for mandamus, a writ of
preliminary injunction and a TRO against the sale of reclaimed lands by PEA to AMARI and from
implementing the JVA. Following these events, under President Estrada’s admin, PEA and
AMARI entered into an Amended JVA and Mr. Chaves claim that the contract is null and void.
Issue: Is the transfer to AMARI lands reclaimed or to be reclaimed as part of the
stipulations in the JVA between AMARI and PEA violate Sec. 3 Art. XII of the 1987 Constitution?
Held: The 157.84 hectares of reclaimed lands comprising the Freedom Islands, now
covered by certificates of title in the name of PEA, are alienable lands of the public domain. PEA
may lease these lands to private corporations but may not sell or transfer ownership of these lands
to private corporations. PEA may only sell these lands to Philippine citizens, subject to the
ownership limitations in the 1987 Constitution and existing laws.
The 592.15 hectares of submerged areas of Manila Bay remain inalienable natural
resources of the public domain until classified as alienable or disposable lands open to disposition
and declared no longer needed for public service. The government can make such classification
and declaration only after PEA has reclaimed these submerged areas. Only then can these lands
qualify as agricultural lands of the public domain, which are the only natural resources the
government can alienate. In their present state, the 592.15 hectares of submerged areas are
inalienable and outside the commerce of man.
Since the Amended JVA seeks to transfer to AMARI, a private corporation, ownership of
77.34 hectares110 of the Freedom Islands, such transfer is void for being contrary to Section 3,
Article XII of the 1987 Constitution which prohibits private corporations from acquiring any kind
of alienable land of the public domain.
Since the Amended JVA also seeks to transfer to AMARI ownership of 290.156
hectares111 of still submerged areas of Manila Bay, such transfer is void for being contrary to
Section 2, Article XII of the 1987 Constitution which prohibits the alienation of natural resources
other than agricultural lands of the public domain.
PEA may reclaim these submerged areas. Thereafter, the government can classify the
reclaimed lands as alienable or disposable, and further declare them no longer needed for public
service. Still, the transfer of such reclaimed alienable lands of the public domain to AMARI will
be void in view of Section 3, Article XII of the 1987Constitution which prohibits private
corporations from acquiring any kind of alienable land of the public domain.
Case No. 32
Topic: Judicial persons or private corporations as applicants for land registration

Chavez v. PEA and AMARI


FACTS: On February 4, 1977, then President Ferdinand E. Marcos issued Presidential
Decree No. 1084 creating PEA. PD No. 1084 tasked PEA "to reclaim land, including foreshore
and submerged areas," and "to develop, improve, acquire, lease and sell any and all kinds of lands."
On the same date, then President Marcos issued Presidential Decree No. 1085 transferring to PEA
the "lands reclaimed in the foreshore and offshore of the Manila Bay" under the Manila-Cavite
Coastal Road and Reclamation Project (MCCRRP).
On January 19, 1988, then President Corazon C. Aquino issued Special Patent No. 3517,
granting and transferring to PEA "the parcels of land so reclaimed under the Manila-Cavite Coastal
Road and Reclamation Project (MCCRRP). Subsequently, on April 9, 1988, the Register of Deeds
of the Municipality of Parañaque issued Transfer Certificates in the name of PEA, covering the
three reclaimed islands known as the “Freedom Islands.”
PEA and AMARI entered into the JVA through negotiation without public bidding. On
April 28, 1995, the Board of Directors of PEA, in its Resolution No. 1245, confirmed the JVA. On
June 8, 1995, then President Fidel V. Ramos, through then Executive Secretary Ruben Torres,
approved the JVA. The Senate Committees reported the results of their investigation in Senate
Committee. Among the conclusions of their report are: (1) the reclaimed lands PEA seeks to
transfer to AMARI under the JVA are lands of the public domain which the government has not
classified as alienable lands and therefore PEA cannot alienate these lands; (2) the certificates of
title covering the Freedom Islands are thus void, and (3) the JVA itself is illegal.
On December 5, 1997, then President Fidel V. Ramos issued Presidential Administrative
Order No. 365 creating a Legal Task Force to conduct a study on the legality of the JVA in view
of Senate Committee Report. The Legal Task Force upheld the legality of the JVA, contrary to the
conclusions reached by the Senate Committees.

ISSUE: Can private persons apply for original land registration proceedings?
RULING: No. Act No. 2874 of the Philippine Legislature Sec. 55. Any tract of land of
the public domain which, being neither timber nor mineral land, shall be classified as suitable for
residential purposes or for commercial, industrial, or other productive purposes other than
agricultural purposes, and shall be open to disposition or concession, shall be disposed of under
the provisions of this chapter, and not otherwise.
The rationale behind this State policy is obvious. Government reclaimed, foreshore and
marshy public lands for non-agricultural purposes retain their inherent potential as areas for public
service. This is the reason the government prohibited the sale, and only allowed the lease, of these
lands to private parties. The State always reserved these lands for some future public service.
However, government reclaimed and marshy lands, although subject to classification as disposable
public agricultural lands, could only be leased and not sold to private parties because of Act No.
2874.
The 1987 Constitution continues the State policy in the 1973 Constitution banning private
corporations from acquiring any kind of alienable land of the public domain. Like the 1973
Constitution, the 1987 Constitution allows private corporations to hold alienable lands of the
public domain only through lease. As in the 1935 and 1973 Constitutions, the general law
governing the lease to private corporations of reclaimed, foreshore and marshy alienable lands of
the public domain is still CA No. 141. Without the constitutional ban, individuals who already
acquired the maximum area of alienable lands of the public domain could easily set up corporations
to acquire more alienable public lands. An individual could own as many corporations as his means
would allow him. An individual could even hide his ownership of a corporation by putting his
nominees as stockholders of the corporation. The corporation is a convenient vehicle to circumvent
the constitutional limitation on acquisition by individuals of alienable lands of the public domain.
Case No. 33
Topic: Judicial persons or private corporations as applicants for land registration

Director of Forestry v. Villareal


FACTS. In 1949, respondent Ruperto Villareal applied for the registration of 178,113
square meters of mangrove swamps situated in Sapian, Capiz alleging that he and his predecessors-
in-interest had been in possession of the land for more than forty (40) years. The CFI of Capiz
approved the application, which was further affirmed by the CA.
Petitioner, the Director of Forestry, then filed a petition for review on certiorari claiming
that the subject land was forestal in nature and not subject to private appropriation and asking for
the reversal of the registration.

The respondent invoked the survey plan of the mangrove swamps approved by the Director
of Lands as a proof that the land is registrable. He also sustained his claim of possession over the
subject land by providing the tax declarations as evidence for such.
ISSUE. What is the legal classification of mangrove swamps, commonly known as
manglares?
RULING. The Supreme Court concluded that mangrove swamps or manglares should be
understood as comprised within the public forest of the Philippines as defined in the Section 1820
of the Administrative Code of 1917 which provides that public forest includes, except as otherwise
specifically indicated, all unreserved public land, including nipa and mangrove swamps, and all
forest reserves of whatever character.
The Court maintained the view in the Amunategui case that the classification of mangrove
swamps as forest lands is descriptive of its legal nature or status and does not have to be descriptive
of what the land actually looks like. Unless and until the land classified as ‘forest’ is released in
an official proclamation to that effect so that it may form part of the disposable agricultural lands
of the public domain, the rules on confirmation of imperfect titles do not apply.
Such description of mangrove swamps as pertaining to our agricultural lands should be
understood as covering only those lands over which ownership had already vested before October
1, 1917 when the Administrative Code of 1917 became effective. Such lands could not be
retroactively legislated as forest lands for this would prejudice a duly acquired property right
protected by the due process clause.
Thus, the subject land being admittedly a part of the mangrove swamps of Sapian, for
which a minor forest license had in fact been issued by the Bureau of Forestry from 1920 to 1950,
must be considered forest land and cannot be the subject of the adverse possession and consequent
ownership claimed by the respondent in support of his application for registration.
The mere existence of a survey plan would not have the effect of converting the mangrove
swamps, which are forest lands, into agricultural land. And the approval by the Director of Lands
is ineffectual, since it is clearly officious, because his office was not authorized to act in the
premises. It is the Director of Forestry who has the authority to determine whether forest land is
more valuable for agricultural rather than forestry uses, as a basis for its declaration as agricultural
land and release for private ownership.
Tax declarations are not sufficient to prove possession and much less vest ownership over
a property as the Higher Court have held in several cases.
Thus, the Supreme Court SET ASIDE the decision of the CA and DISMISSED the
application for registration of title of respondent Villareal, with costs against him. This was
immediately executory.

Case No. 34
Topic: Venue of original land registration proceedings

City of Dumaguete v. Philippine Ports Authority


FACTS: On October 14, 1998, petitioner, City of Dumaguete, filed before the RTC an
Application for Original Registration of Title over a parcel of land with improvements, located in
the City of Dumaguete under the Property Registration Decree.
The respondent filed oppositions and posited that the subject property was foreshore land
belonging to the State and not subject to private appropriation, unless the same had already been
declared by the executive or legislative department of the national government as no longer needed
for coast guard service, public use, or special industries, and classified as alienable and disposable.
Hence, full-blown trial in the case was no longer necessary as the evidence so far presented by
petitioner had already established that the RTC lacked jurisdiction over the subject matter of the
case.
Petitioner insisted that the RTC should continue with the hearing of the case and allow
petitioner to present evidence that the subject property is reclaimed land. Petitioner sufficiently
alleged in its application for registration that it has been in open, continuous, exclusive, and
notorious possession of the subject property for more than thirty (30) years under a bona fide claim
of ownership. In support of such allegation, petitioner must necessarily prove that the subject
property was previously a swampy area, which had to be filled or reclaimed before the construction
of the City Engineers Office building thereon.
The RTC initially agreed with respondent, but in the interest of justice, it issued an order
to have a full-blown proceeding to determine factual issues. The respondent then sought recourse
to the CA which found merit in the petition of respondent and set aside the RTC Orders.
ISSUE: Does the RTC have jurisdiction over the land registration proceedings?
RULING: Yes. What determines the jurisdiction of the court is the nature of the action
pleaded as appearing from the allegations in the complaint. The averments therein and the
character of the relief sought are the ones to be consulted.
Under Section 17 the Property Registration Decree, jurisdiction over an application for
land registration was vested on the CFI of the province or city where the land was situated. The
applicant shall file together with the application all original muniments of titles or copies thereof
and a survey plan of the land approved by the Bureau of Lands.
Batas Pambansa Blg. 129, otherwise known as The Judiciary Reorganization Act of 1980,
created the RTC in place of the CFI. Presently, jurisdiction over an application for land registration
remains with the RTC where the land is situated, except when such jurisdiction is delegated by the
Supreme Court to the Metropolitan Trial Court, Municipal Trial Courts, and Municipal Circuit
Trial Courts under certain circumstances.

It is not disputed that the Application for Original Registration of Title filed by petitioner
before the RTC of the City of Dumaguete conformed with the Property Registration Decree, which
prescribes the form and contents of such applications. In its Application, petitioner prayed that its
title to the subject property, which it repeatedly alleged to have acquired through continuous and
adverse possession and occupation of the said property for more than 30 years or since 1960, be
placed under the land registration laws. The allegations and prayer in the Application of petitioner
were sufficient to vest jurisdiction on the RTC over the said Application upon the filing thereof.
Case No. 35
Topic: The Survey Plan (Tracing Cloth Plan)

UP vs. Rosario
Facts. On September 7, 1971, Datu Ditingke Ramos filed with CFI Quezon City, an
application for registration of title covering a parcel of land situated in Quezon City, with an area
of 100,000 square meters. On August 31, 1972, petitioner U.P filed with the trial court a motion
for intervention in the case, claiming that the land covered by the application is within its property
described in Transfer Certificate of Title No. 9462. On March 15, 1973, U.P. filed with the trial
court an opposition and motion to dismiss Datu Ditingke Ramos application for registration. The
trial court ruled that the application does not encroach the property of the petitioner; hence their
motion to dismiss is denied.
The decision rendered has become final and the Commissioner of the Land Registration
Commission issued Decree No. N-150604 in favor of Rosario Alcovendas Vda. de
Ramos,surviving spouseof the original applicant who was substituted as party applicant in the
order of April 24, 1973, pursuant to which the Register of Deeds of Quezon City issued OCT No.
17 in her name. On February 23, 1988, Rosario Alcovendas Vda. de Ramos executed a deed of
absolute sale in favor of Segundina Rosario covering the parcel of land embraced in Transfer
Certificate of Title No. 223619. On June 11, 1988, fire razed the Quezon City Hall Building which
housed the Office of the Register of Deeds of Quezon City. Transfer Certificate of Title No.
223619 was one of the titles destroyed by the fire. Segundina, however, requested the Register of
Deeds to reconstitute Transfer Certificate of Title No. 223619 resulting in the issuance of Transfer
Certificate of Title No. RT-78195 (223619). On March 11, 1993, U.P. filed with the Regional Trial
Court, Branch 21, Quezon City a petition for the cancellation of Transfer Certificate of Title No.
(N-126671) 367316 naming Segundina and others as respondents. Nevertheless, Segundina was
able to cause the registration of the land in a new Transfer of Certificate of Title. On November
19, 1996, after the parties had presented their respective evidence, U.P. filed an amended petition
alleging that it is the true, absolute and registered owner of a parcel of land covered by Transfer
Certificate of Title No. 9462 of the Register of Deeds of Quezon City and that the unlawful acts
of ownership being exercised by Segundina and Bugnay Construction and Development
Corporation as well as the existence of their spurious certificates of title, create a cloud of doubt
on the title of (U.P.).On May 15, 1997, Segundina filed with the trial court an omnibus motion for
the dismissal of U. P.s third cause of action in the amended petition as well as the cancellation of
the notice of lis penden sannotated on TCT No. 121042. The trial court denied the motion of
Segundina. The CA, however, ruled in favour of Segundina which ruled that the third cause of
action is barred by res judicata and that the trial court committed grave abuse of discretion in
denying Segundinas motion to dismiss. U.P contended that that Survey Plan presented by
Segundina is not approved by the Bureau of Lands.
Issues: Whether or not the survey plan by Segundina is valid and acceptable as evidence
to a title.
Ruling: No. P. D. No. 1529 requires the Director of Lands to sign and approve the survey
plan for the land applied for, otherwise, the title is void. Under Sec. 17 of P.D 1529 provides that
The application for land registration shall be filed with the Court of First Instance of the province
or city where the land is situated. The applicant shall file together with the application all original
muniments of titles or copies thereof and a survey plan approved by the Bureau of Lands. The
Court found out that OCT. No. 17 is void and Segundina traces her rights to OCT No. 17, her
claim would have no basis as a spring cannot rise higher than its source. Hence, the Court denied
Segundinas motion to cancel the notice of lis pendens and grants the petition of U.P. In lieu thereof,
the Court orders the case REMANDED to the trial court for trial on the merits.
Case No. 36
Topic: The Survey Plan (Tracing Cloth Plan)

Republic v. Guinto-Aldana
FACTS: Respondents filed an application for registration of title over 2 pieces of land,
professing themselves to be co-owners of these lots having acquired them by succession from their
predecessors. That until the time of the application, they and their predecessors-in-interest have
been in actual, open, peaceful, adverse, exclusive and continuous possession of these lots in the
concept of an owner and that they had consistently declared the property in their name for purposes
of real estate taxation. In support of their application, respondents submitted to the court the
pertinent tax declarations, together with the receipts of payment thereof. Petitioner opposed the
application for the reason that the tax declaration submitted to the court did not constitute
competent and sufficient evidence of bona fide acquisition in good faith or of prior possession in
the concept of an owner.
ISSUE: WON respondents have occupied and possessed the property openly,
continuously, exclusively and notoriously under a bona fide claim of ownership.
HELD: Respondents’ possession through their predecessors-in-interest dates back to as
early as 1937 when the property had already been declared for taxation by respondent’s father.
Respondents could have produced more proof of this kind had it not been for the fact that, the
relevant portions of the tax records on file with the Provincial Assessor had been burned when its
office was razed by fire in 1997. With the tax assessments therecame next tax payments.
Respondents’ receipts for tax expenditures were likewise in therecords and in these documents the
predecessors of respondents were the named owners of the property. Tax declarations and realty
tax payment are not conclusive evidence ofownership, nevertheless, they are a good indication of
possession in the concept of an owner. No one in his right mind would be paying taxes for a
property that is not in his actual or at least constructive possession. Indeed, respondents herein
have been in possession of the land in the concept of an owner, open, continuous, peaceful and
without interference and opposition from the government or from any private individual. Itself
makes their right thereto unquestionably settled and hence, deserving of protection under the law.
Case No. 37
Topic: Notice of Initial Hearing

Fieldman Agricultural Trading Corp. (FATCO) v. Republic


Facts: On October 19, 1993, FATCO filed with the Regional Trial Court (RTC) of La
Union an application for confirmation of title to parcels of land, described as Lots No. 1505, No.
1234 and No. 47030,with an aggregate area of 8,463 square meters, situated in Barrio Poblacion,
Bacnotan, La Union.
On November 11, 1994, the RTC issued an Order setting the application for initial hearing
on February 28, 1995. The Order was published in the January 23, 1995 issue of the Official
Gazette,and the February 18-24, 1995 issue of the Guardian. The notice of hearing was, likewise,
posted in a conspicuous place in each parcel of land included in the application, and on the bulletin
board of the municipal building of Bacnotan, La Union. The Provincial Prosecutor of La Union
was furnished with a copy of notice of hearing on November 18, 1994.
At the scheduled initial hearing on February 28, 1995, Atty. Marita Balloguing entered her
appearance as collaborating counsel for FATCO, and requested the resetting of the marking of
exhibits. The RTC granted the request and issued an Order resetting the hearing to April 19, 1995.
The Republic, through the Provincial Prosecutor, was duly informed of the resetting.
On March 2, 1995, the OSG again entered its appearance as counsel for the Republic and
once more deputized the Provincial Fiscal of San Fernando, La Union to appear in the case. On
the same date, the Republic filed its Opposition to FATCOs application for registration.
During the hearing on April 19, 1995, Prosecutor Gloria D. Catbagan appeared for the
Republic. FATCO, through counsel, offered in evidence documents to establish jurisdictional
facts.
The RTC then issued an Order setting the case for the reception of evidence on May 25,
1995 at 8:30 in the morning.
Petitioner’s Contention: It complied with all the jurisdictional requirements, specifically
the publication of the notice of initial hearing. It, therefore, faulted the CA for reversing the RTC
and, accordingly, dismissing its application for registration.
Respondent’s Contention:
RTC never acquired jurisdiction over FATCOs application because the publication of
initial hearing was fatally defective. It points out that the initial hearing set on February 28, 1995
was reset to April 19, 1995. The actual initial hearing, therefore, took place on a date different
from what was stated in the published notice of initial hearing. Hence, re-publication of the new
notice of hearing was necessary, but FATCO failed to publish the notice of hearing set on April
19, 1995, thus, preventing the RTC from acquiring jurisdiction over the application.
CA’s Ruling: CA reversed the RTC Decision. The CA agreed with the Republic that the
RTC did not acquire jurisdiction over FATCOs application because the publication of initial
hearing was fatally defective. The notice that was published in the Official Gazette and in the
Guardian was the hearing set on February 28, 1995, but no hearing was conducted on the said date.
The actual initial hearing was held on April 19, 1995, a date different from what was stated in the
notice, thereby defeating the very purpose of the publication requirement.
Issue: Did the court acquire jurisdiction to hear petitioners application for registration due
to petitioners failure to publish the notice of actual hearing set on April 19, 1995 and to post said
notice in conspicuous places and to serve the same to adjoining owners?
Held: Yes. Section 23 of P.D. No. 1529, or the Property Registration Decree, explicitly
provides that before the court can act on the application for land registration, the public shall be
given notice of the initial hearing thereof by means of publication, mailing, and posting.
However, even if, at the February 28, 1995 hearing, FATCOs counsel requested a resetting,
and the RTC granted said request, the Republic and all interested parties were already fully
apprised of the pendency of the application. When the hearing was reset to April 19, 1995,
interested parties, the Republic included, may be deemed to have been given notice thereof. There
was, thus, no need for the re-publication of notice of hearing, for clearly, the avowed purpose of
Section 23 had already been accomplished. We, therefore, find that the application for registration
was rightfully given due course by the RTC. The CA, thus, committed reversible error in holding
otherwise.
Case No. 38
Topic: Effect of non- publication or defective publication

Republic v. Court of Appeals and Ribaya


FACTS: On 27 July 1920, a parcel of land located in the barrio of Magragondong,
Municipality of Ligao, Province of Albay, was surveyed for the spouses Luis Ribaya and
Agustina Revatoris by Telesforo Untalan, a Bureau of Lands surveyor. The parcel of land was
found to comprise an area of 25,542,603 square meters. The survey plan was denominated as
Plan II-13961 and allegedly approved by the Acting Director of Lands on 3 January 1922.
However, as noted by the Court of Appeals in its 9 January 1991 decision, these exhibits do not
at all show the surveyor's signature. Moreover, as per Land Classification Map No. 871 of the
Bureau of Forestry, the above parcel of land was considered part of the public forest and released
for disposition only on 31 December 1930.
In 1925, the spouses Ribaya applied for registration and confirmation of title of the lot
before the then Court of First Instance (CFI) of Albay. Notice of the application, and hearing
thereof were published in the 17 March 1925 issue of the Official Gazette,and in its decision of 18
September 1925,the CFI granted the said application. Sometime later, or on 18-21 November and
23-30 November 1925, a resurvey of the parcel of land covered by Plan II-13961 was conducted
at the instance of the spouses Ribaya. This gave rise to four different parcels of land with an
aggregate area of only 10,975,022 square meters, instead of the original 25,542,603 square meters.
Plan II-13961-Amd, appeared to have been approved by the Director of Lands on 26 February
1926. The application was not amended to reflect the resurvey and the amended plan was not
published.
On 31 July 1926, the corresponding decree of registration was issued, while on 19 August
1926, Original Certificate of Title (OCT) No. 3947 covering the four lots embraced by Plan II-
13961-Amd. was issued in the names of the spouses Ribaya.
On 11 September 1958, OCT No. 3947 was administratively reconstituted from the owner's
duplicate copy thereof and the reconstituted title was denominated as OCT No. PO-10848 (3947).
In 1964, the heirs of Luis Ribaya received compensation from the Foreign Claims Settlement
Commission of the United States for damages sustained by the land during the war.
In 1968, pursuant to a deed of partition executed by the private respondents herein, the land
covered by OCT No. RO-10848 (3947) was subdivided per Subdivision Plan LRC Psd-96075,
approved on 16 December 1968. Then, OCT No. RO-10848 (3947) was cancelled and separate
Transfer Certificates of Title (TCT) were issued to the private respondents.
ISSUE: Is the registration valid having only published once?
RULING: The Court hold that the land registration court in LRC Case No. 52, G.L.R.O.
Record No. 26050 never acquired jurisdiction over the land covered by either the original plan or
the amended plan for lack of sufficient publication of the first and total want of publication of the
second.
As found by both the trial court in Civil Case No. 6198 and the Court of Appeals, the notice
of the hearing of application of the spouses Ribaya for the registration of the land covered by the
original plan was published in the 17 March 1925 issue of the Official Gazette. In short, there was
only one publication thereof. Section 31 of Act No. 496, the governing law then, required two
publications. Hence, the decision of 18 September 1925 of the land registration court was void for
want of the required publications. The requirement of dual publication is one of the essential bases
of the jurisdiction of the registration court; it is a jurisdictional requisite. Land registration is a
proceeding in rem and jurisdiction in rem cannot be acquired unless there be constructive seizure
of the Land through publication and service of notice.
Worse, the decision of 18 September 1925 was entirely based on an alleged original survey
plan. The fact remains, however, that in November of that year that original plan was amended
and the amended plan was not published at all. There is no evidence that the court amended its
decision to conform to the amended plan, neither is there a showing that the parties even attempted
publication thereof. However, the decree that was subsequently issued was based on the amended
plan insofar as the four lots were concerned.
A decree of registration is required to recite the description of the land. It follows then that
the land registration court may have amended its decision to conform to the amended plan for the
four lots which ultimately found their way into the decree issued by the General Land Registration
Office, and finally, into OCT No. 3947. Whether it did so or not and the General Land Registration
Office merely adjusted the decree to conform to the amended plan, such aims were fatally flawed
due to the absence of publication of the amended plan.
The Court of Appeals in its challenged resolution of 24 January 1994 and the private
respondents, however, maintain that the publication of the amended plan was unnecessary under
our pronouncements in Benin vs. Tuazon. This case reiterates our rulings in Philippine
Manufacturing Co. vs. Imperial, Juan and Chuongco vs. Ortiz, Bank of the Philippine Islands vs.
Acua, Lichauco vs. Herederos de Corpus, and Director of Lands vs. Benitez, that only where the
original survey plan is amended during the registration proceedings, by the addition of land not
previously included in the original plan, should publication be made in order to confer jurisdiction
on the court to order the registration of the area added after the publication of the original plan.
Conversely, if the amendment does not involve an addition, but on the contrary, a reduction of the
original area that was published, no new publication is required.
Case No. 39
Topic: Effect of non- publication or defective publication

VICTOR BENIN, ET AL. v. MARIANO SEVERO TUASON y DE LA PAZ, ET AL.,


defendants. J. M. TUASON & CO., INC.
FACTS: The plaintiffs alleged that they were the owners and possessors of the three
parcels of agricultural lands, described in paragraph V of the complaint, located in the barrio of La
Loma (now barrio of San Jose) in the municipality (now city) of Caloocan, province of Rizal,
having an aggregate area of approximately 278,928 square meters; that they inherited said parcels
of land from their ancestor Sixto Benin, who in turn inherited the same from his father, Eugenio
Benin; that they and their predecessors in interest had possessed these three parcels of land openly,
adversely, and peacefully, cultivated the same and exclusively enjoyed the fruits harvested
therefrom; that Eugenio Benin, plaintiff's grandfather, had said parcels of land surveyed on March
4 and 6, 1894, that during the cadastral survey by the Bureau of Lands of the lands in Barrio San
Jose in 1933 Sixto Benin and herein plaintiffs claim the ownership over said parcels of land; that
they declared said lands for taxation purposes in 1940 under Tax Declaration No. 2429; that after
the outbreak of the last World War, or sometime in 1942 and subsequently thereafter, evacuees
from Manila and other places, after having secured the permission of the plaintiffs, constructed
their houses thereon and paid monthly rentals to plaintiffs.
Only defendant J.M. Tuason & Co., Inc. was actually served with summons. The other
defendants were ordered summoned by publication in accordance with Sections 16 and 17 of the
Rules of Court. Only defendant J.M. Tuason & Co., Inc. appeared. The other defendants were all
declared in default. The rendered a decision in favor of the Plaintiffs and against the Defendants.
ISSUE: Is there a need to for a new publication of the amended plan in order to vest the
Court of Land Registration with jurisdiction?
HELD: No. We believe that the lower court erred when it held that the Land Registration
Court was without jurisdiction to render the decision in LRC No. 7681. Under Section 23 of Act
496, the registration court may allow, or order, an amendment of the application for registration
when it appears to the court that the amendment is necessary and proper. Under Section 24 of the
same act the court may at any time order an application to be amended by striking out one or more
parcels or by severance of the application. The amendment may be made in the application or in
the survey plan, or in both, since the application and the survey plan go together. If the amendment
consists in the inclusion in the application for registration of an area or parcel of land not previously
included in the original application, as published, a new publication of the amended application
must be made. The purpose of the new publication is to give notice to all persons concerned
regarding the amended application. Without a new publication the registration court can not
acquire jurisdiction over the area or parcel of land that is added to the area covered by the original
application, and the decision of the registration court would be a nullity insofar as the decision
concerns the newly included land. The reason is because without a new publication, the law is
infringed with respect to the publicity that is required in registration proceedings, and third parties
who have not had the opportunity to present their claim might be prejudiced in their rights because
of failure of notice. But if the amendment consists in the exclusion of a portion of the area covered
by the original application and the original plan as previously published, a new publication is not
necessary. In the latter case, the jurisdiction of the court over the remaining area is not affected
by the failure of a new publication.
In the case at bar We find that the original plan covering Parcel 1 and Parcel 2 that
accompanied the application for registration in LRC No. 7681 was amended in order to exclude
certain areas that were the subject of opposition, or which were the subject of another registration
case; and the Chief of the Survey Division of the Court of Land Registration was ordered to
determine whether the amended plan included lands or areas not included in the original plan. In
compliance with the order of the registration court said Chief of the Survey Division informed the
court that no new parcels were included in the new (or amended) plan. It is thus shown that the
amended plan in LRC No. 7681 did not cover parcels, or areas, that were not previously included
in the original plan which accompanied the application that had been published in the Official
Gazette. There was, therefore, no necessity for a new publication of the amended plan in order to
vest the Court of Land Registration with jurisdiction to hear and decide the application for
registration in LRC No. 7681 and to order the issuance of Decree of Registration No. 17431 upon
which Original Certificate of Title No. 735 was based.
Case No. 40
Topic: Effect of non- publication or defective publication

De Luzuriaga v. Republic
FACTS: On May 16, 1997, petitioners filed an Application for the Registration of Title
over Lot No. 1524 of the Bacolod Cadastre. In it, the subject lot was specifically identified as Lot
No. 1524, and the survey plan and the technical description of the subject lot were submitted to
the RTC.
On May 12, 1998, the application was amended to state, thus: "that the parcel of land in
question be ordered registered and that an original Certificate of Title be issued in the name of the
late Jose R. De Luzuriaga, Sr. pursuant to Decree No. 22752 covering Lot No. 1524 of Bacolod
Cadastre.”
By Decision dated May 24, 1999, the trial court judicially confirmed the incomplete title
of the late De Luzuriaga, Sr. over Lot No. 1524 pursuant to Decree No. 22752, but the Republic
alleged that the RTC did not acquire jurisdiction over the case inasmuch as the corresponding
amended application for registration was not published and a copy of which the Republic was not
served.
ISSUE: Did the petitioners adequately satisfy the requirement of publication for land
registration proceedings?
RULING: Yes. The Republic, after participating in the proceedings, has raised the issue
of jurisdiction, drawing attention to the non-publication of the amended application for registration
during the trial of the case. The Court cannot see its way clear to the jurisdictional challenge posed
by the Republic. As it were, the Republic entered its appearance in the case represented by
prosecutor Bayona. The petitioners in that case appeared to have complied with the essential
jurisdictional requirement of publication. The required survey plan, technical description, and
original tracing cloth have been duly presented and submitted as evidence. Prosecutor Bayona
obviously found the cadastral proceedings to have been in order, else, he would have duly protested
and assailed the same.
We hardly can subscribe to the Republic’s argument that the publication of the amendment
in petitioners’ application is a condition sine qua non for the RTC, acting as cadastral court, to
acquire jurisdiction. Sec. 7 of Act No. 2259, otherwise known as the Cadastral Act, and Sec. 35 of
PD 1529, otherwise known as the Land Registration Decree, provide for the publication of the
application for registration and the schedule of the initial hearing. This is so since judicial cadastral
proceedings, like ordinary administrative registration, are in rem, and are governed by the usual
rules of practice, procedure, and evidence. Due publication is required to give notice to all
interested parties of the claim and identity of the property that will be surveyed. And any additional
territory or change in the area of the claim cannot be included by amendment of the plan or
application without new publication, otherwise the cadastral court does not acquire jurisdiction
over the additional or amended claim. But where the identity and area of the claimed property are
not the subjects of amendment but other collateral matters, a new publication is not needed.
In the case at bar, there is no dispute that due publication was made for Lot No. 1524, its
identity and area. The amendment in petitioners’ application in the relief portion neither altered
the area and identity of the subject lot nor added any territory. Thus, no new publication is required.
Besides, the Republic, through Prosecutor Bayona, has been duly notified of such amendment.
Consequently, the Republic could not plausibly argue that it was deprived of its day in court.
Case No. 41
Topic: Effect of failure of the Republic to file any opposition or answer to the application

Republic vs. Heirs of Evaristo Tiotio-en


Facts: This case involves the the second application filed by Evaristo Tiotioen on
September 6, 1993 for judicial confirmation and registration under the Torrens System of two
parcels of land situated in Pico, La Trinidad, Benguet, with an aggregate area of 180,488 square
meters. Evaristo Tiotioen was substituted by his heirs in the case when he died on June 21, 1997.
Santiago A. Santiago, the Municipality of La Trinidad, Benguet, and the petitioner opposed the
aforesaid application.
In a Notice of Appearance the OSG formally requested that its appearance be entered as
counsel for the petitioner and that all notices of hearings, orders, resolutions and decision be served
to the OSG at its given address. The said notice of appearance informed the court that the OSG
authorized the Provincial Prosecutor of Benguet to appear in the case. The petitioner filed its
Opposition and Supplemental Opposition on the ground that the parcels of land, applied for
registration by the respondents, belong to the communal forest of La Trinidad, Benguet, and are
therefore inalienable land of the public domain, which have not been classified and considered as
disposable and alienable. The land registration court granted the granted the application of the
Evaristo.
The petitioner and the municipality received their respective notices of the decision on
September 6 and 7, 2001. The municipality filed its Motion for Reconsideration thereto on
September 20, 2001. The petitioner, on the other hand, filed a Motion and Manifestationon October
5, 2001 adopting the said motion of the municipality. The land registration court denied the said
motion for reconsideration of the municipality in its Resolution dated December 6, 2001. The
OSGwas not furnished with a notice of such resolution. The OSG was informed by the provincial
prosecutor of such denial on January 4, 2002 when it received the Letter dated December 19, 2001
of the Provincial Prosecutor. The OSG filed the subject notice of appeal for the petitioner only on
January 11, 2002 which the land registration court denied for having been filed way beyond the
fifteen-day reglementary period to appeal which the said court reckoned from September 6, 2001.
The CA affirmed the land registration courts denial of the subject notice of appeal of the petitioner
but gave due course to the appeal of the municipality. The CA pointed out that the petitioner filed
its motion and manifestation adopting the adverted motion for reconsideration of the municipality
beyond the reglementary period to file an appeal and, thus, the decision of the land registration
court already attained finality insofar as the petitioner was concerned.
Issue: Whether or not the petitioner has still the right to appeal.
Ruling: Yes. The court is guided by the settled doctrine that the belated filing of an appeal
by the State, or even its failure to file an opposition, in a land registration case because of the
mistake or error on the part of its officials or agents does not deprive the government of its right
to appeal from a judgment of the court. The Court has allowed some meritorious cases to proceed
despite inherent procedural defects and lapses. This is in keeping with the principle that rules of
procedure are mere tools designed to facilitate the attainment of justice and that strict and rigid
application of rules which would result in technicalities that tend to frustrate rather than promote
substantial justice must always be avoided. It is a far better and more prudent cause of action for
the court to excuse a technical lapse and afford the parties a review of the case to attain the ends
of justice, rather than dispose of the case on technicality and cause grave injustice to the parties,
giving a false impression of speedy disposal of cases while actually resulting in more delay, if not
a miscarriage of justice. The vast tracts of land involved in this case are claimed by the petitioner
to be a protected watershed area, which allegedly preserves the main source of water of the
Municipality of La Trinidad. Relative thereto, the petitioner raises substantial factual and legal
issues which should be decided on their merit instead of being summarily disposed of based on a
technicality.
Case No. 42
Topic: Issuance of Order of Default; Remedies of an aggrieved party- Appaeal

Martinez v. Republic
FACTS: Martinez filed a PETITION FOR REGISTRATION in his name of three parcels
of land located in Cortes, Surigao del Sur with an area of 3700sqm. He alleged that:
a) He had acquired the property in 1952 through purchase from his uncle whose predecessors-
in-interest were traceable upto 1870s;
b) He had remained in continuous possession of the lots;
c) The lots remained unencumbered;
d) They became private property through prescription;
e) He had to initiate the proceedings since the Director of Land Management Services failed
to do so despite the completion of the cadastral survey.
RTC Surigao del Sur set the case for initial hearing and ordered the publication of the notice.
Republic opposed the application on the grounds that:
a) Martinez’s possession was not in accordance with Sec48(b) of CA141;
b) His muniments of title were insufficient to prove bona-fide acquisition and possession of
the property;
c) The lots formed part of the public domain.
RTC issued an ORDER OF GENERAL DEFAULT because no party appeared to oppose the
application during the hearing, and subsequently, decreed the registration of the lots in the name
of Martinez. RTC concluded that Martinez and his predecessors have been in the open, continuous,
public possession of the lots for over 100 years.
LRA informed RTC that only 2 lots were referred to in the Notice published since the other lot
(LOT 370) was omitted due to the lack of an approved survey plan.
CA reversed RTC and ordered the dismissal of the application on the ground that the evidence
presented by Martinez is insufficient to support his application.
Hence, this petition by Martinez arguing that Republic has no right to oppose the petition or appeal
following the issuance of the order of general default.
ISSUE: WON REPUBLIC, THRU OSG, CAN STILL APPEAL THE RTC’S DECISION
AFTER IT HAD BEEN DELARED IN DEFAULT
RULING: YES. A defendant party declared in default retains the right to appeal from the
judgment by default on the ground that the plaintiff failed to prove the material allegations of the
complaint, or that the decision is contrary to law, even without need of the prior filing of a motion
to set aside the order of default.
SEC 26 of PD1529 provides that the order of default may be issued if no person appears and
answers within the time allowed. RTC issued the order of general default simply because no
oppositor appeared on the date of the hearing, despite the fact that the Republic had already duly
filed its opposition.RTC erred in declaring oppositor in default simply because he failed to appear
on the day of the initial hearing. RTC should have accorded the oppositor ample opportunity to
establish its claim. (Dir of Lands vs Santiago). HOWEVER, the SC cannot decide on the validity
of the default order since Republic did not challenge such.
Case No. 43
Topic: Issuance of Order of Default

Vergel v. Court of Appeals


Facts: Digna Vergel, Eduardo Salvacruz, Beatriz Mañacop, Felicisima Flores, Generoso
and Blandino Salvacruz, Milagros Evangelista and the heirs of Corazon Santiago, namely:
Leocadio, Jr. and Concepcion Santiago (petitioners herein) filed with the Regional Trial Court,
Calamba, Laguna an application for registration of a parcel of land. The Republic opposed.
The trial court issued "an order of general default against the whole world with the
exception of Republic of the Philippines."
Respondent Dorotea Tamisin Gonzales filed with the trial court an "Urgent Motion to Set
Aside the Order of General Default" alleging, inter alia, in her affidavit that she "is claiming the
land in question subject of this petition as an owner" which motion was opposed by the petitioners
herein. Such was denied because of lack of merit.
Respondent filed with the Court of Appeals6 a petition for certiorari alleging that the trial
court judge "acted capriciously and without or in excess of his jurisdiction and gravely abused the
exercise of his discretion”.
The Court of Appeals promulgated a decision annulling the trial court’s orders.

Issue: Did the Court of Appeals erred in setting aside the trial court’s order of general
default in the land registration case involved without making a specific finding of fraud,
negligence, accident or excusable mistake but relying on its view that substantial justice and
speedy determination of the controversy would be better attained in lifting the order of general
default, to enable a claimant to oppose and to establish a case of ownership in herself.
Held: Yes. The Court of Appeals arbitrarily set aside the trial court’s order of general
default without factual basis save for its own gut feeling.
Respondent’s failure to file timely opposition to the application for land registration
because she missed reading the publication of the notice in the Official Gazette or in the newspaper
"Malaya" issue of August 8, 1994, in itself may not be considered excusable negligence.
In respondent’s motion to set aside order of general default, she alleged that petitioners
were aware of her claim of ownership over the subject property, but did not give her personal
notice of the filing of the application. She learned about the application by accident. In the petition
for certiorari she filed with the Court of Appeals, respondent alleged that petitioners filed the
application in bad faith, surreptitiously and without notice to her. The Court of Appeals did not
make a finding on this.
The Court is not a trier of facts. Consequently, the Court have to remand the case to the
Court of Appeals for it to make findings of fact constituting fraud, accident or excusable neglect
sufficient for the court to lift the order of general default in the land registration case involved.
Case No. 44
Topic: Attributes and Limitations of Certificate of Title

Cusi v. Domingo
FACTS: The property in dispute was a vacant unfenced lot situated in White Plains,
Quezon City and covered by Transfer Certificate of Title (TCT) No. N-165606 issued in the name
of respondent Lilia V. Domingo by the Registry of Deeds of Quezon City. It had an area of 658
square meters. In July 1999, Domingo learned that construction activities were being undertaken
on her property without her consent. She soon unearthed the series of anomalous transactions
affecting her property.
Radelia Sy (Sy) petitioned before the RTC for reissuance of new owner’s copy and, as
proof, presented a deed of sale dated July 14, 1997 executed by Domingo in her favor, and an
affidavit of loss dated July 17, 1997, stating that her bag containing the owner’s copy of TCT No.
N-165606 had been snatched while she was at the SM City, North EDSA. After the RTC granted
the petition, the Register of Deeds cancelled the TCT No. N-165606 and issued a new TCT No.
186142 in favor of Syby virtue of the deed of absolute sale date July 14, 1997. Sy immediately
subdivided the property and sold each half to Spouses De Vera and Spouses Cusi, and were issued
TCT Nos. 189568 and 189569 respectively, annotatedon the TCT a consideration of onlyPhp 1M
each but the entire lot had an actual valueof not less than Php 14M.
It was only on July 1999 when the respondent learned the situation.She filed an action
against Spouses Sy, Spouses De Vera, and the Spouses Cusi seeking annulment of titles, injuction,
and damages. She also applied for the issuance of writ of preliminary prohibition and mandatory
injunction, and a temporary restraining order (TRO).
ID., LOWER COURT’S RULING: The RTC granted her application, however, the title
of Spouses De Vera and Spouses Cusiremain valid as they were held purchasers in good faith.
Dissatisfied with the decision, Domingo filed a motion for reconsideration. The RTC set aside its
first decision and declaring the sale between the respondent and Sy void; the buyers were not
purchasers in good faith; cancellation of TCT Nos. 189568 and 189569; the TCT No. 165606 shall
be revalidated in the name of Domingo. This decision was brought up to the CA filed by the
petitioners but was denied. A motion for reconsideration was also filed but the same was
denied.Hence, this petition.
ISSUE: Is the petitioners considered petitioners in good faith having in possession a title
of the land in question?
RULING: Under the Torrens System of land registration, “a person dealing in the
registered land has the right to rely on the Torrens certificate title and to dispense with the need of
inquiring further, exceptwhen the party has actual knowledge of facts and circumstances that
would impel a reasonably cautious man to make such inquiry.”
Their observance of a certain degree of diligence within the context of the principles
underlying the Torrens System was not the only barometer for them to verify the acquisition of
title. Under the law and jurisprudence, it was not enough for them to show that the property was
unfenced and vacant nor it was safe for them to rely on the face of Sy’s TCT No. 186142 because
they were aware that the TCT was derived only from a duplicate owner’s copy reissued by virtue
of the loss of the original duplicate owner’s copy. That circumstance should have already alerted
them to the need to inquire beyond the face of the Sy’s TCT. Other circumstances that would
impel a reasonably cautious man to make such inquiry in dealing with the property are the almost
simultaneous transactions affecting the acquisition of the property that the petitioners were also
aware of and the material, undervaluation of the property in the deed of sale, e.i. the price in
consideration of the property of Php 1M each half when the market value is at least Php
14Mostensibly at the request of Sy to minimize her liabilities for Capital Gains Tax.
Case No. 45
Topic: Attributes and Limitations of Certificate of Title

Sampacu v. Lantud
FACTS: On September 14, 1984, respondent Hadji Serad Mingca Lantud, the plaintiff in the
lower court, filed an action to quiet title with damages with the Regional Trial Court (RTC) of
Lanao del Sur, Branch 8, Marawi City (trial court), against petitioner Datu Kiram Sampaco
(deceased), the defendant in the lower court, who has been substituted by his heirs, represented
by Hadji Soraya Sampaco-Macabando.
Respondent alleged in his Complaint that he is the owner in fee simple of a parcel of
residential lot located at Marinaut, Marawi City, with an area of 897 square meters covered by
Original Certificate of Title (OCT) No. P-658. On August 25, 1984, petitioner Datu Kiram
Sampaco, through his daughter Soraya Sampaco-Macabando with several armed men, forcibly
and unlawfully entered his property and destroyed the nursery buildings, cabbage seedlings and
other improvements therein worth P10,000.00. On August 30, 1984, Barangay Captain Hadji
Hassan Abato and his councilmen prepared and issued a decision in writing stating that petitioner
Datu Kiram Sampaco is the owner of the subject parcel of land. Respondent stated that the acts
of petitioner and the said decision of the Barangay Captain may cast a cloud over or otherwise
prejudice his title.Respondent stated that he and his predecessors-in-interest have been in open,
public and exclusive possession of the subject property. He prayed that the acts of petitioner and
the decision of Barangay Captain Hadji Hassan Abato and his councilmen be declared invalid,
and that petitioner be ordered to pay respondent damages in the amount of P10,000.00 and
attorneys fees.
In his Answer, defendant Datu Kiram Sampaco, petitioner herein, denied the material
allegations of the Complaint. Petitioner asserted that he and his predecessors-in-interest are the
ones who had been in open, public, continuous, and exclusive possession of the property in
dispute. Petitioner alleged that OCT No. P-658 was secured in violation of laws and through
fraud, deception and misrepresentation, considering that the subject parcel of land is a residential
lot and the title issued is a free patent. Moreover, respondent and his predecessors-in-interest had
never taken actual possession or occupied the land under litigation. On the contrary, petitioner
has all the evidence of actual possession and ownership of permanent improvements and other
plants on the land in dispute.
The trial court held that the issuance of respondents title, OCT No. P-658, was tainted
with fraud and irregularities and the title is, therefore, spurious; hence, it is null and void, and
without any probative value. On August 15, 2003, the Court of Appeals rendered a Decision
reversing the decision of the trial court. The Court of Appeals stated that the Torrens title has
three attributes: (1) a Torrens title is the best evidence of ownership over registered land and,
unless annulled in an appropriate proceeding, the title is conclusive on the issue of ownership;
(2) a Torrens title is incontrovertible and indefeasible upon the expiration of one year from the
date of the entry of the decree of registration; and (3) a Torrens title is not subject to collateral
attack.
ISSUE: Whether or not the Court of Appeals erred in sustaining the validity of OCT No.
P-658 and confirming respondent as owner of the property in dispute.
HELD: The Torrens title is conclusive evidence with respect to the ownership of the
land described therein, and other matters which can be litigated and decided in land registration
proceedings. Tax declarations and tax receipts cannot prevail over a certificate of title which is
an incontrovertible proof of ownership.[ An original certificate of title issued by the Register of
Deeds under an administrative proceeding is as indefeasible as a certificate of title issued under
judicial proceedings. However, the Court has ruled that indefeasibility of title does not attach to
titles secured by fraud and misrepresentation.
Fraud and misrepresentation, as grounds for cancellation of patent and annulment of title,
should never be presumed, but must be proved by clear and convincing evidence, mere
preponderance of evidence not being adequate. Fraud is a question of fact which must be proved.
the Court holds that the evidence on record is insufficient to prove that fraud was committed in
the issuance of respondents Torrens title. Hence, respondents Torrens title is a valid evidence of
his ownership of the land in dispute.
Respondents original certificate of title was issued on May 22, 1981, while the
counterclaim was filed by petitioner on October 15, 1984, which is clearly beyond the one-year
prescriptive period. In fine, the Court of Appeals did not err in confirming that respondent is the
owner of the parcel of land covered by OCT No. P-658.
Case No. 46
Topic: Attributes and Limitations of Certificate of Title

De Guzman v. Agbagala
FACTS: On November 18, 1987, respondent filed a civil case praying that the deed of
donation over five parcels of land executed by Carmen in favor of Madelene be nullified. An
amended complaint was filed on September 15, 1988 to include petitioner spouses Raymundo and
Perla de Guzman as one of the transferees of the properties.

Respondent claimed that the deed of donation was fake, which was confirmed by the handwriting
expert of the NBI, Rogelio G. Azores, who examined the document and compared it with several
documents bearing the signature of Carmen. He found that the purported signature of the late
Carmen on the deed of donation was forged.
Petitioners filed their answer claiming that they applied for a free patent over the subject
property. They were issued a free patent and on December 11, 1987, an Original Certificate of
Title (OCT) over the subject property was registered in their name. During the trial, they also
presented a tax declaration and realty tax receipts from 1985 to 1990 issued to them.
The RTC declared the deed of donation in favor of Madelene null and void ab initio,
canceled the deeds of sale executed by Madelene in favor of the defendants, and declared null and
void the OCT in the name of petitioners. On appeal, the CA affirmed the decision of the RTC,
hence, the petition.
Petitioners argue that at the time of the filing of the complaint on September 15, 1988, an
OCT had already been issued in their name. Thus this certificate of title can only be nullified in an
action directly attacking its validity.
Respondent counters that at the time the amended complaint was filed, the OCT which was
issued on December 11, 1987 was not yet indefeasible since less than one year had lapsed.
Furthermore, she asserts that the doctrine of indefeasibility does not apply if the free patent is null
and void ab initio.
ISSUE: Can a certificate of title only be nullified in an action directly attacking its validity?
RULING: As provided in sections 32 and 48 of PD 1529, a decree of registration or patent
and the certificate of title issued pursuant thereto may be attacked on the ground of falsification or
fraud within one year from the date of their issuance. Such an attack must be direct and not by a
collateral proceeding.
An action is deemed an attack on a title when the object of the action or proceeding is to
nullify the title and thus challenge the judgment pursuant to which the title was decreed. The attack
is direct when the object of the action is to annul or set aside such judgment, or enjoin its
enforcement. On the other hand, the attack is indirect or collateral when, in an action to obtain a
different relief, an attack on the judgment is nevertheless made as an incident thereof.
In the present case, the attack on the OCT was merely collateral because the action was
principally for the declaration of nullity of the deed of donation and the other deeds of conveyance
which followed.
However, the principle of indefeasibility does not apply when the patent and the title based
thereon are null and void. An action to declare the nullity of a void title does not prescribe and is
susceptible to direct, as well as to collateral, attack. The OCT in the case was registered on the
basis of a free patent which the RTC ruled was issued by the Director of Lands without authority.
The petitioners falsely claimed that the land was public land when in fact it was not as it was
private land previously owned by Carmen who inherited it from her parents. The Public Land law
applies only to lands of the public domain and the Director of Lands has no authority to grant free
patent to lands that have ceased to be public in character and have passed to private ownership.
This finding was affirmed by the CA, and there is no reason to reverse it.
Since the Director of Lands has no authority to grant a free patent over privately- owned
land, any title issued pursuant thereto is null and void. Therefore, although the OCT was merely
collaterally attacked, it was still correctly nullified because the free patent on which it was based
was null and void ab initio.
Case No. 47
Topic: Attributes and Limitations of Certificate of Title

Firaza vs Ugay
Facts: Spouses Ugay commenced a complaint for Quieting of Title alleging that they are
the registered owners of a lot and the tax declaration issued in Firaza’s name should be annulled
on the ground that it creates a cloud upon the respondents’ title. The petitioner set up the
affirmative defense that the respondents obtained their title through fraud and misrepresentation
perpetrated during the processing of their Free Patent Application before the Office of the CENRO.
On the basis of the said affirmative defense, the petitioner also filed a counterclaim praying for the
nullification of OCT issued to Spouses Ugay and reconveyance to him of ownership of the lot.
The RTC required the parties to submit their respective memorandum to which the respondent
complied. However, petitioner filed a Motion to Dispense with the Filing of the Petitioner’s
Memorandum reasoning that his affirmative defense cannot be proven adequately through a
written pleading.The RTC denied petitioner’s affirmative defense on the ground that the same can
be better ventilated along with the allegations of the complaint and answer in a full-blown
trial.During the trial Land Management Officer Tadem was presented a witness for respondents.
The petitioner’s counsel, however, propounded questions pertaining to the circumstances attending
the issuance by Tadem of a recommendation for the respondents’ Free Patent Application.
Respondents counsel objected to the questions. Hence, the RTC required the parties to file, as they
did so file, their respective position papers on whether the petitioner’s counterclaim constitutes a
direct or a collateral attack to the validity of the respondents’ title. The RTC issued an Order
disallowing any issue pertaining to the petitioner’s counterclaim which in turn was adjudged as a
direct attack to the validity of the respondents’ title; hence, prohibited. When his motion for
reconsideration was denied by the RTC in an Order, the petitioner sought recourse with the CA
via a special civil action for certiorari. The CA affirmed RTC’s decision but premised on the
different finding that the petitioner’s counterclaim was a collateral attack to the validity of the
respondent’s title.
Issue: Whether or not petitioner’s counterclaim constitutes collateral attack in respondents
title.
Ruling: No. Section 48 of Presidential Decree No. 1529 proscribes a collateral attack to a
certificate of title and allows only a direct attack. The attack is considered direct when the object
of an action is to annul or set aside such proceeding, or enjoin its enforcement. Conversely, an
attack is indirect or collateral when, in an action to obtain a different relief, an attack on the
proceeding is nevertheless made as an incident thereof. Such action to attack a certificate of title
may be an original action or a counterclaim, in which a certificate of title is assailed as void. The
CA erroneously classified the herein counterclaim as a collateral attack. On the other hand, the
RTC correctly adjudged the same as a direct attack to the respondents’ land title but mistakenly
declared it as a prohibited action. As such counterclaim, it involves a cause of action separate from
that alleged in the complaint; it has for its purpose the vindication of a right in as much as the
complaint similarly seeks the redress of one. As the plaintiff in his own counterclaim, the petitioner
is equally entitled to the opportunity granted the plaintiff in the original complaint, to establish his
cause of action and to prove the right he asserts.
Case No. 48
Topic: Attributes and Limitations of Certificate of Title

Casimiro Development Corporation v. Mateo


FACTS: In 1988, petitioner purchased from China Bank the land in question which was
previously sold by the mother of Mateo to Rodolfo Pe who in turn constituted a mortgage on the
property in favor of China Bank as security for a loan. China Bank foreclosed the mortgage and
consolidated its ownership of the property after Rodolfo failed to redeem. A TCT was issued in
the name of China Bank. In 1991, CDC brought an action for unlawful detatiner against the
respondent’s siblings. Respondent counters that CDC acquired the property from China Bank in
bad faith because it had actual knowledge of the possession of the property by the respondent and
his siblings.
ISSUE: Whether or not CDC was an innocent purchaser for value.
RULING: One who deals with property registered under the Torrens system need not go
beyond the certificate of title, but only has to rely on the certificate of title. He is charged with
notice only of such burdens and claims as are annotated on the title. China Bank’s TCT’s was a
clean title, that is, it was free from any lien or encumbrance, CDC had the right to rely, when it
purchased the property, solely upon the face of the certificate of title in the name of China Bank.
The respondent’s siblings’ possession did not translate to an adverse claim of ownership. They
even characterized their possession only as that of mere agricultural tenants. Under no law was
possession grounded on tenancy a status that might create a defect or inflict a law in the title of the
owner. CDC having paid the full and fair price of the land, was an innocent purchaser for value.
The TCT in the name of CDC was declared valid and subsisting.
Case No. 49
Topic: Attributes and Limitations of Certificate of Title

Nieto v. Municipality of Meycauayan


Facts: Anacleto Nieto was the registered owner of a parcel of land, consisting of 3,882
square meters, situated at Poblacion, Meycauayan, Bulacan and covered by TCT No. T-24.055
(M). The property is being used by respondent, Municipality of Meycauayan, Bulacan, which
constructed an extension of the public market therein.
Upon Anacletos death on July 26, 1993, his wife, Sixta P. Nieto, and their three children,
namely, Eulalio P. Nieto, Gaudencio Nieto and Corazon Nieto-Ignacio, herein petitioners, collated
all the documents pertaining to his estate. When petitioners failed to locate the owners duplicate
copy of TCT No. T-24.055 (M), they filed a petition for the issuance of a second owners copy with
the RTC, Malolos, Bulacan. In that case, petitioners discovered that the missing copy of the title
was in the possession of the respondent. Consequently, petitioners withdrew the petition and
demanded from respondent the return of property and the certificate of title.
Petitioners filed a complaint for recovery of possession and damages against respondent
alleging that the latter was in possession of the owner’s copy of TCT No. T-24.055 (M). They
prayed that respondent be ordered to surrender to them the owner’s copy of TCT No. T-24.055
(M), vacate the property, and pay them the rents thereon from 1966 until the date of the filing of
the complaint for the total of P1,716,000.00, and P10,000.00 a month thereafter, as well as
P300,000.00 as moral damages, and P100,000.00 as attorney’s fees.
Issues: Are lands covered by the Torrens System subject to prescription?
May the defense of imprescriptibility only be invoked by the registered owner to the exclusion of
his legitimate heirs?
Held: First Issue:No. An action to recover possession of a registered land never prescribes
in view of the provision of Section 44 of Act No. 496 to the effect that no title to registered land
in derogation of that of a registered owner shall be acquired by prescription or adverse possession.
It follows that an action by the registered owner to recover a real property registered under the
Torrens System does not prescribe.
Second Issue: It is well settled that the rule on imprescriptibility of registered lands not only
applies to the registered owner but extends to the heirs of the registered owner as well.
In Mateo v. Diaz, the Court held that prescription is unavailing not only against the
registered owner, but also against his hereditary successors because the latter step into the shoes
of the decedent by operation of law and are the continuation of the personality of their predecessor-
in-interest. Hence, petitioners, as heirs of Anacleto Nieto, the registered owner, cannot be barred
by prescription from claiming the property.
However, the Court held that while it is true that a Torrens title is indefeasible and
imprescriptible, the registered landowner may lose his right to recover possession of his registered
property by reason of laches.
Case No. 50
Topic: Attributes and Limitations of Certificate of Title

Heirs of Alejandra Delfin v. Rabadon13


FACTS: On October 19, 1993, respondents filed before the RTC an action to recover the
ownership and possession of the subject property from petitioners, seeking as well the payment of
damages based on their complaint and the testimonies of their witnesses during trial.
The RTC ruled that petitioners had the better right to the ownership and possession of the
subject property. It based its conclusion on the fact that the subject property was declared by
petitioners for taxation purposes and that they paid the realty taxes due thereon. It held that while
tax declarations and tax receipts are not incontrovertible evidence of ownership, they become proof
of ownership when accompanied by proof of actual possession such as petitioners’ continuous
declaration of the subject property for taxation purposes, their payments of the corresponding
taxes, and the construction of their respective houses thereon.
The CA reversed the RTC’s pronouncement, holding that respondents had the better right
of ownership and possession over the subject property. It observed that, apart from the self-serving
testimonies of some of the petitioners, the only evidence adduced by them in support of their claim
are mere copies of tax declarations and tax receipts over the subject property and a Report dated
July 14, 1993 of one Director Silverio G. Perez of the Department of Registration of the LRA
(LRA Report) to the effect that the property in question is covered by TCT No. 20910. The CA
stressed that tax declarations and tax receipts are not conclusive evidence of ownership or of the
right to possess the land when not supported by other evidence of actual possession which
remained wanting in this case.
PETITIONER. Petitioners countered that: (a) they inherited the subject property from
their predecessor-in-interest, Remegio, who bought the foregoing even before the second World
War; (b) the subject property was issued a certificate of title in the name of Remegio, however,
the said title was lost; (c) Alejandra inherited the subject property by virtue of an extra-judicial
settlement and after its execution, she and her children, petitioners Leopoldo, Francisco and
Marcelito Delfin, took over the possession of the same; and (d) the subject property had been
declared by them for taxation purposes and they paid the corresponding realty taxes due thereon.
By way of affirmative defense, petitioners further contended, inter alia, that respondents’ demands
were already barred by laches, given that they took about 55 years to file their complaint.
RESPONDENT. They alleged that, (a) the subject property was owned by their
predecessor-in-interest,8 Emiliana Bacalso (Emiliana), pursuant to Decree No. 98992;9 (b) while
the foregoing decree was lost during the last World War, its existence could still be shown by a
certification (LRA certification) issued by the Land Registration Authority (LRA), and a certified
copy from page 19 of the daybook of cadastral lots issued by the Register of Deeds (RD) of Cebu
City (daybook entry); (c) after Emiliana’s death, Genaro Rabadon took over the possession of the
subject property and upon his death, his children, herein respondents, took over its possession until
1988; (d) in 1989, they discovered that the said property was already in the possession of petitioner
Alejandra Delfin (Alejandra) and some of her children and their families already constructed their
houses thereon; and (e) when they confronted Alejandra, she claimed that petitioners’ predecessor-
in-interest, Remegio Navares (Remegio) previously bought the said property; however, when they
asked to see a copy of the deed of sale, she could not produce the same.
ISSUE: Are the respondents have the better right to the ownership and possession of the
subject property.
RULING: The petition is bereft of merit. The Court finds that the respondents have shown
a better right to the ownership and possession of the subject property.
It is an elemental rule that a decree of registration bars all claims and rights which arose or
may have existed prior to the decree of registration. By the issuance of the decree, the land is bound
and title thereto quieted, subject only to certain exceptions29 under the property registration
decree. Besides, tax declarations and tax receipts may only become the basis of a claim for
ownership when they are coupled with proof of actual possession of the property. In this case,
records are bereft of any showing that petitioners, or any of their predecessors-in-interest, have
been in actual possession of the subject property prior to 1989 as they claim. The tax declarations
and tax receipts are insufficient to prove their proffered theory that their predecessor-in-interest,
Remegio, was the lawful possessor and owner of the foregoing property even before the last World
War. On the contrary, aside from the LRA certification and daybook entry which prove the
existence of Decree No. 98992, respondents' possession of the subject property prior to petitioners'
entry in 1989 was attested to by one Marcelina Tabora who. As the CA notes, appears to be an
unbiased witness. All told, by sheer preponderance of evidence, respondents have shown a better
right to the ownership and possession of the subject property and hence, must be awarded the same.
Case No. 51
Topic: Remedies of an aggrieved party- Petition for Review

Heirs of Maximo Labanon v. Heirs of Constancio Labanon


FACTS: During the lifetime of Constancio Labanon, prior to the outbreak of WWII, he
settled upon a piece of alienable and disposable public agricultural land situated at Brgy. Lanao,
Kidapawan, Cotabato x x x. Constancio cultivated the said lot and introduced permanent
improvements that still exist up to the present. Being of very limited educational attainment, he
found it difficult to file his public land application over said lot. Constancio then asked his
brother, Maximo Labanon who was better educated to file the corresponding public land
application under the express agreement that they will divide the said lot as soon as it would be
feasible for them to do so. The offer was accepted by Maximo. During the time of the application
it was Constancio who continued to cultivate the said lot in order to comply with the cultivation
requirement set forth under Commonwealth Act 141, as amended, on Homestead applications.
After which, on June 6, 1941, due to industry of Constancio, Homestead Application No. 244742
(E-128802) of his brother Maximo was approved with Homestead Patent No. 67512. Eventually,
Original Certificate of Title No. P-14320 was issued by the Register of Deeds of Cotabato over
said lot in favor of Maximo Labanon.
On February 11, 1955, Maximo Labanon executed a document denominated as
Assignment of Rights and Ownership and docketed as Doc. No. 20; Page No. 49; Book No. V;
Series of 1955 of the Notarial Register of Atty. Florentino Kintanar. The document was executed
to safeguard the ownership and interest of his brother Constancio Labanon. On April 25, 1962,
Maximo Labanon executed a sworn statement reiterating his desire that his elder brother
Constancio, his heirs and assigns shall own the eastern portion of the Lot.
After the death of Constancio Labanon, his heirs executed an [e]xtra-judicial settlement
of estate with simultaneous sale over the aforesaid eastern portion of the lot in favor of Alberto
Makilang, the husband of Visitacion Labanon, one of the children of Constancio. Subsequently,
the parcel of land was declared for taxation purposes in the name of Alberto under TD No.
11593. However, in March 1991, the defendants heirs of Maximo Labanon namely, Alicia L.
Caniedo, Leopoldo Labanon, Roberto Nieto and Pancho Labanon, caused to be cancelled from
the records of the defendant Provincial Assessor of Cotabato the aforesaid TD No. 11593 and the
latter, without first verifying the legality of the basis for said cancellation, did cancel the same. x
x x Further, after discovering that the defendant-heirs of Maximo Labanon were taking steps to
deprive the heirs of Constancio Labanon of their ownership over the eastern portion of said lot,
the latter, thru Alberto Makilang, demanded the owners copy of the certificate of title covering
the aforesaid Lot to be surrendered to the Register of Deeds of Cotabato so that the ownership of
the heirs of Constancio may be fully effected but the defendants refused and still continue to
refuse to honor the trust agreement entered into by the deceased brothers. x x x
ISSUE: Are the Heirs of Constancio Labanon precluded from challenging the OCT No.
P-41320?
HELD: No. The principle of indefeasibility of a TCT is embodied in Section 32 of
Presidential Decree No. (PD) 1529, amending the Land Registration Act, which provides:

Section 32. Review of decree of registration; Innocent purchaser for


value. The decree of registration shall not be reopened or revised by reason
of absence, minority, or other disability of any person adversely affected
thereby, nor by any proceeding in any court for reversing judgments, subject,
however, to the right of any person, including the government and the
branches thereof, deprived of land or of any estate or interest therein by such
adjudication or confirmation of title obtained by actual fraud, to file in the
proper Court of First Instance a petition for reopening and review of the
decree of registration not later than one year from and after the date of the
entry of such decree of registration, but in no case shall such petition be
entertained by the court where an innocent purchaser for value has acquired
the land or an interest therein, whose rights may be prejudiced. Whenever the
phrase innocent purchaser for value or an equivalent phrase occurs in this
Decree, it shall be deemed to include an innocent lessee, mortgagee, or other
encumbrancer for value.

Upon the expiration of said period of one year, the decree of registration
and the certificate of title issued shall become incontrovertible. Any person
aggrieved by such decree of registration in any case may pursue his remedy
by action for damages against the applicant or any other persons responsible
for the fraud.

Contrary to petitioners interpretation, the aforequoted legal provision does not totally
deprive a party of any remedy to recover the property fraudulently registered in the name of
another. Section 32 of PD 1529 merely precludes the reopening of the registration proceedings for
titles covered by the Torrens System, but does not foreclose other remedies for the reconveyance
of the property to its rightful owner. As elaborated in Heirs of Clemente Ermac v. Heirs of Vicente
Ermac:

While it is true that Section 32 of PD 1529 provides that the decree of


registration becomes incontrovertible after a year, it does not altogether
deprive an aggrieved party of a remedy in law. The acceptability of the
Torrens System would be impaired, if it is utilized to perpetuate fraud against
the real owners.

A more succinct explanation is found in Vda. De Recinto v. Inciong, thus:

The mere possession of a certificate of title under the Torrens system does
not necessarily make the possessor a true owner of all the property described
therein for he does not by virtue of said certificate alone become the owner
of the land illegally included. It is evident from the records that the petitioner
owns the portion in question and therefore the area should be conveyed to
her. The remedy of the land owner whose property has been wrongfully or
erroneously registered in another's name is, after one year from the date of
the decree, not to set aside the decree, but, respecting the decree as
incontrovertible and no longer open to review, to bring an ordinary action in
the ordinary court of justice for reconveyance or, if the property has passed
into the hands of an innocent purchaser for value, for damages.

Undeniably, respondents are not precluded from recovering the eastern portion of Original
Certificate of Title (OCT) No. P-14320, with an area subject of the Assignment of Rights and
Ownership previously owned by their father, Constancio Labanon. The action for Recovery of
Ownership before the RTC is indeed the appropriate remedy.
Case No. 52
Topic: Remedies of an aggrieved party- Petition for Review

Tichangco v. Enriquez
FACTS: Sometime in March 1996, Renato Tichangco, in behalf of the homeowners’
association of Gagalangin and Sunog Apog (Tondo, Manila), petitioner, who are occupants of
various parcels of land in Gagalangin, Tondo, filed a land title verification request with the Land
Registration Authority. The verification request was prompted by an alleged claim of ownership
of a certain Manotok, private respondent, over the land which petitioners occupy, and which they
perceive as public land, being portions of the dried or filled bed of Estero de Maypajo and Sunog
Apog area. They further contend that the certificate of titles issued to private respondent are null
and void.
Private respondent’s claim on the other hand is based on accretion by the drying up of a
portion of the esteros adjoining their property, which they sought registration of these dried up
lands, and is now covered by certificates of title, which is the subject of the controversy.
ISSUE: Are the petitioners correct that the certificates of title issued to private respondent
should be declared null and void?
RULING: No. The fundamental purpose of the Land Registration Law (Act No. 496, now
PD 1529) is to finally settle title to real property in order to preempt any question on the legality
of the title -- except claims that were noted on the certificate itself at the time of registration or
those that arose subsequent thereto. Consequently, once the title is registered under the said law,
owners can rest secure on their ownership and possession.
The proceedings for the judicial registration of land under the Torrens system involve more
consequences than an ordinary action would. Once a decree of registration is made under the
Torrens system, and the reglementary period has passed within which the decree may be
questioned, the title is perfected and cannot be collaterally questioned later on.
After the registration is completed and finalized in the regular course, the rights of all
adverse claimants are foreclosed by the decree of registration. The government itself assumes the
burden of giving notice to all parties. The very purpose and intent of the law, however, would be
defeated by permitting persons to litigate again on the basis of the same adverse claims in the
registration proceedings, after they have already been given the opportunity to do so. For them to
raise the same questions anew would be to cast doubt again upon the validity of the registered title.
Even assuming that petitioners may still institute an action for the nullification of the
subject OCTs, the review of a decree of registration under Section 38 of Act No. 496 (Section 32
of Presidential Decree No. 1529) would prosper only upon proof that the registration was procured
through actual fraud. "The fraud must be actual and extrinsic, not merely constructive or intrinsic;
the evidence thereof must be clear, convincing and more than merely preponderant, because the
proceedings which are assailed as having been fraudulent are judicial proceedings which by law,
are presumed to have been fair and regular."
Actual fraud proceeds from an intentional deception perpetrated through the
misrepresentation or the concealment of a material fact. The fraud is extrinsic if it is employed to
deprive parties of their day in court and thus prevent them from asserting their right to the property
registered in the name of the applicant. The fraud is intrinsic if that which is alleged in the petition
to set aside the decree is the fraud involved in the same proceedings in which the parties seeking
relief have had ample opportunity to assert their right, to attack the document presented by the
applicant for registration, and to cross-examine the witnesses who have testified thereon. Inquiry
into this latter kind of fraud is barred after the judgment of the land registration court has become
final.
Petitioners fail to convince the Court that the facts they rely upon to justify a review of the
decree in question constitute actual extrinsic fraud.
Case No. 53
Topic: Remedies of an aggrieved party- Reconveyance

Torbela vs Rosario
Facts: The controversy began with a parcel of land, with an area of 374 sq. meters Lot
356-A, located in Urdaneta City, Pangasinan which was originally part of larger land measuring
749 sq. meters Lot 356 and covered by OCT 16676 in the name of Valeriano. Valeriano gave 374
sq m lot to his sister Marta Semilla, married to Eugenio Torbela. Upon the death of Spouses
Torbela, the land was adjudicated equally among their children, the Torbela sibilings. On
December 12, 1964, the Torbela siblings executed a Deed of Absolute Quitclaim over the land in
favour of Dr. Rosario. Accordingly, for and in consideration of P9.00 the land is transferred and
convey to Rosario. OCT 16676 in Valeriano’s name was partially cancelled and a TCT was issued
in favour of Rosario. Another Deed of Absolute Quitclaim was subsequently executed on
December 28, 1964, this time by Dr. Rosario, acknowledging that he only borrowed Lot No. 356-
A from the Torbela siblings and was already returning the same to the latter for P1.00. The
aforequoted Deed was notarized, but was not immediately annotated on TCT issued to Rosario.
Dr. Rosario obtained a loan from DBP secured by a mortgage constituted on Lot No. 356-A. The
mortgage was annotated in the TCT. Dr. Rosario used the proceeds of the loan for the construction
of improvements on Lot No. 356-A. On May 16, 1967 Cornelio executed an Affidavit of Adverse
Claim, on behalf of the Torbela siblings. They have annotated such adverse claim and the deed of
absolute claim on the TCT. The construction, however, was completed and Dr. Rosario was able
to pay the loan from DBP. Dr. Rosario acquired another loan from PNB and secured such loan
three parcel of lot incuding Lot 356-A. The annotation of adverse claim on the TCT was cancelled
by Dr. Rosario. Consequently, Dr. Rosario and his wife obtained third loan from Banco Filipino
which was secured by three lots including Lot 356-A. Because Banco Filipino paid the balance of
Dr. Rosarios loan from PNB, the mortgage on Lot No. 356-A in favor of PNB was cancelled. On
February 13, 1986, the Torbela siblings filed before the RTC a Complaint for recovery of
ownership and possession of Lot No. 356-A, plus damages, against the spouses Rosario. The
spouses Rosario afterwards failed to pay their loan from Banco Filipino and extrajudicially
foreclosed the mortgage on the lots including Lot 356-A. Banco Filipino was the lone bidder of
the forclosed properties and a certificate of sale was annotated on the TCT. Torbela siblings filed
before the RTC their Amended Complaint, impleading Banco Filipino as additional defendant and
praying that the spouses Rosario be ordered to redeem Lot No. 356-A from Banco Filipino. The
spouses Rosario instituted before the RTC a case for annulment of extrajudicial foreclosure and
damages, with prayer for a writ of preliminary injunction and temporary restraining order, against
Banco Filipino. The RTC declared that the real estatate mortgage over the Lot 356-A executed by
Dr. Rosario in favour of Banco Filipino is legal and valid. The Torbela siblings and Dr. Rosario
appealed the foregoing RTC judgment before the Court of Appeals. The CA affirmed the decision
of RTC.
Issue: Whether or not Torbela siblings may still recover the lot 356-A which was
mortgaged by Dr. Rosario.
Ruling: Under Article 2085 of the Civil Code, one of the essential requisites of the contract
of mortgage is that the mortgagor should be the absolute owner of the property to be mortgaged;
otherwise, the mortgage is considered null and void. However, an exception to this rule is the
doctrine of mortgagee in good faith. Under this doctrine, even if the mortgagor is not the owner of
the mortgaged property, the mortgage contract and any foreclosure sale arising therefrom are given
effect by reason of public policy. This principle is based on the rule that all persons dealing with
property covered by a Torrens Certificate of Title, as buyers or mortgagees, are not required to go
beyond what appears on the face of the title. However, The Court finds that Banco Filipino is not
a mortgagee in good faith. The notice of adverse claim was not validly cancelled, and the improper
cancellation should have been apparent to Banco Filipino and aroused suspicion in said bank of
some defect in Dr. Rosarios title. Under Section 110 of the Land Registration Act or Section 70 of
the Property Registration Decree, notice of adverse claim can only be cancelled after a party in
interest files a petition for cancellation before the RTC wherein the property is located, and the
RTC conducts a hearing and determines the said claim to be invalid or unmeritorious. No petition
for cancellation has been filed and no hearing has been conducted herein to determine the validity
or merit of the adverse claim of the Torbela siblings. Hence, the right of the Torbela siblings over
Lot No. 356-A is superior over that of Banco Filipino; and as the true owners of Lot No. 356-A,
the Torbela siblings are entitled to a reconveyance of said property even from Banco Filipino.
Case No. 54
Topic: Remedies of an aggrieved party- Reconveyance

Lorzano v. Tabayag Jr.


FACTS: The petitioner Lorzano and the respondent are two of the children of the late Juan
Tabayag (Tabayag) who died on June 2, 1992. Tabayag owned a parcel of land situated in Sto.
Domingo, Iriga City (subject property). Right after the burial of their father, the petitioner allegedly
requested from her siblings that she be allowed to take possession of and receive the income
generated by the subject property until after her eldest son could graduate from college. The
petitioner’s siblings acceded to the said request. After the petitioner’s eldest son finished college,
her siblings asked her to return to them the possession of the subject property so that they could
partition it among themselves. However, the petitioner refused to relinquish her possession of the
subject property claiming that she purchased the subject property from their father as evidenced
by a Deed of Absolute Sale of Real Property executed by the latter on May 25, 1992. The
respondent claimed that their father did not execute the said deed of sale. He pointed out that the
signature of their father appearing in the said deed of sale was a forgery as the same is markedly
different from the real signature of Tabayag. The respondent alleged that the petitioner purposely
forged the signature of Tabayag in the said deed of sale to deprive him and their other siblings of
their share in the subject property. He then averred that the subject property was already covered
by Original Certificate of Title (OCT) No. 1786 issued by the Register of Deeds of Iriga City on
January 9, 2001 registered under the name of the petitioner. OCT No. 1786 was issued pursuant to
Free Patent No. 051716 which was procured by the petitioner on June 24, 1996. RTC rendered a
decision declaring the supposed deed of sale void. The RTC opined that a cursory comparison
between the signature of Tabayag appearing on the said deed of sale and his signatures appearing
on other documents would clearly yield a conclusion that the former was indeed a forgery. CA
affirmed.
ISSUE: W/N reconveyance of the subject property in favor of the respondent is improper
considering that the property was registered under the petitioner’s name pursuant to a free patent
and only the Government, through the Office of the Solicitor General (OSG) could assail her title
thereto in an action for reversion
RULING: An action for reconveyance of the subject property to the heirs of the late
Tabayag on account of the fraud committed by the petitioner is proper in this case. Generally, a
free patent that was fraudulently acquired, and the certificate of title issued pursuant to the same,
may only be assailed by the government in an action for reversion pursuant to Section 101 of the
Public Land Act. A title emanating from a free patent which was secured through fraud does not
become indefeasible, precisely because the patent from whence the title sprung is itself void and
of no effect whatsoever. The rule is that only the government, through the OSG, upon the
recommendation of the Director of Lands, may bring an action assailing a certificate of title issued
pursuant to a fraudulently acquired free patent. However, the foregoing rule is not without an
exception. A recognized exception is that situation where plaintiff-claimant seeks direct
reconveyance from defendant public land unlawfully and in breach of trust titled by him, on the
principle of enforcement of a constructive trust. A private individual may bring an action for
reconveyance of a parcel of land even if the title thereof was issued through a free patent since
such action does not aim or purport to re-open the registration proceeding and set aside the decree
of registration, but only to show that the person who secured the registration of the questioned
property is not the real owner thereof. In Roco, et al. v. Gimeda, we stated that if a patent had
already been issued through fraud or mistake and has been registered, the remedy of a party who
has been injured by the fraudulent registration is an action for reconveyance.
Case No. 55
Topic: Remedies of an aggrieved party- Reversion

YUJUICO vs REPUBLIC
FACTS: In 1973, Fermina Castro filed an application for the registration and confirmation of her
title over a parcel of land located in Parañaque City, in the Pasig-Rizal Court of First Instance
(CFI). The application was opposed by the Office of the Solicitor General (OSG) on behalf of the
Director of Lands, and by Mercedes Dizon, a private party.
Trial court ruled in favor of Castro.
The Director of Lands and Mercedes Dizon did not appeal from the adverse decision of the
Pasig-Rizal CFI. Thus, the order for the issuance of a decree of registration became final, and a
Decree was issued by the Land Registration Commission (LRC). An Original Certificate of Title
was issued in the name of Fermina Castro by the Register of Deeds. Land was then sold to Jesus
Yujuico. The OCT of Castro was cancelled and a TCT was issued in Yujuico’s name over Lot1
while another TCT was issued in favor of herein co-petitioner Augusto Carpio.
Meanwhile, PD no. 1085 was issued and asserts that Land reclaimed in the foreshore and
offshore areas of Manila Bay became the properties of the Public Estates Authority (PEA), a
government corporation that undertook the reclamation of lands or the acquisition of reclaimed
lands. Thus, an OCT was issued in favor of PEA. The PEA also acquired ownership of other
parcels of land along the Manila Bay coast which were subsequently sold to the Manila Bay
Development Corporation (MBDC) The PEA undertook the construction of the Manila Coastal
Road. As this was being planned, Yujuico and Carpio discovered that a verification survey they
commissioned showed that the road directly overlapped their property, and that they owned a
portion of the land sold by the PEA to the MBDC. Yujuico and Carpio filed before the Parañaque
City Regional Trial Court (RTC), a complaint for the Removal of Cloud and Annulment of Title
with Damages.
Respondent’s Contention: First, since the subject land was still underwater, it could not
be registered in the name of Fermina Castro. Second, the land registration court did not have
jurisdiction to adjudicate inalienable lands, thus the decision adjudicating the subject parcel of land
to Fermina Castro was void. And third, the titles of Yujuico and Carpio, being derived from a void
title, were likewise void.
Trial Court ruled in favor of the petitioner and states that after 28 years without being
contested, the case had already become final and executory. The trial court also found that the
OSG had participated in the LRC case, and could have questioned the validity of the decision but
did not.
On appeal, reversed the decision of the lower court asserting that shores are properties of
the public domain intended for public use and, therefore, not registrable and their inclusion in a
certificate of title does not convert the same into properties of private individuals.
ISSUE: Whether or not the action of the Government for reversion is proper?
HELD: No. We maintain to agree with the findings of the court that the property of
Fermina Castro was registrable land, as based on the two (2) ocular inspections conducted on
March 22, 1974 by Lands Administrative Assistant Lazaro G. Berania and Lands Geodetic Engr.
Manuel Cervantes, finding ‘… the same no longer forms part of Manila Bay but is definitely solid
land which cannot be reached by water even in the highest of tides’. This Berania-Cervantes report
based on ocular inspections literally overturned the findings and recommendations of Land
Director Narciso V. Villapando dated November 15, 1973, and that of Director Ernesto C.
Mendiola dated December 1, 1977, and the fact that the Villapando-Mendiola reports were merely
based on projections in the cadastral map or table surveys.
The recognition of petitioners’ legal ownership of the land is further bolstered by the
categorical and unequivocal acknowledgment made by PEA in its September 30, 2003 letter where
it stated that: “Your ownership thereof was acknowledged by PEA when it did not object to your
membership in the CBP-IA Association, in which an owner of a piece of land in CBP-IA
automatically becomes a member thereof.” Section 26, Rule 130 provides that “the act, declaration
or omission of a party as to a relevant fact may be given in evidence against him.” The admissions
of PEA which is the real party-in-interest in this case on the nature of the land of Fermina Castro
are valid and binding on respondent Republic. Respondent’s claim that the disputed land is
underwater falls flat in the face of the admissions of PEA against its interests. Hence, res judicata
now effectively precludes the relitigation of the issue of registrability of petitioners’ lot.
In sum, the Court finds that the reversion case should be dismissed for lack of jurisdiction
on the part of the Parañaque RTC. Even if we treat said case as a petition for annulment of
judgment under Rule 47 of the 1997 Rules of Civil Procedure, the dismissal of the case
nevertheless has to be upheld because it is already barred by laches. Even if laches is disregarded,
still the suit is already precluded by res judicata in view of the peculiar facts and circumstances
obtaining therein.
Case No. 56
Topic: Registration under RA No. 8371 (Compare the Separate Opinions of Puno, Vitug,
Kapunan, Mendoza and Panganiban)

CRUZ V. SECRETARY OF ENVIRONMENT AND NATURAL RESOURCES


Facts: Petitioners Isagani Cruz and Cesar Europa in their capacity as citizens and taxpayers
assailed the constitutionality of R.A. No. 8371 which is also known as the Indigenous Peoples
Rights Act of 1997 (IPRA) on the ground that it amounts to the unlawful deprivation of the State’s
ownership over lands of the public domain and all other natural resources therein, by recognizing
the right of ownership of Indigenous Cultural Communities or Indigenous Peoples (ICCs/IPs) to
their ancestral domains and ancestral lands based on their native title. After the deep-seated
deliberation, the members of the court voted as follows: Seven Justices have voted to dismiss the
petition while the other 7 Justices ruled in favor of the petition. They failed to come up with the
necessary majority instead the votes were equally divided. Both parties initiated a redeliberation
but nothing happened since the voting remained the same. As a result, in virtue of Section 7, Rule
56 of the Rules of Court, the petitioned was dismissed.
Issue: Whether or not the Regalian Doctrine negates native title.
Ruling: The Regalian Doctrine does not negate native title to lands held in private
ownership since time immemorial. The Congress in its wisdom did not intend to have clashing of
laws. In this case, it also resolved the question on whether the existence of native title to land is
considered as an exception to the theory of Regalian. It has been clearly elucidated by the Supreme
Court that the Doctrine of native title is not an exception to the Regalian Doctrine. The Regalian
Doctrine and the Doctrine of the Native Title can exist side by side. The possession per se since
time immemorial has ripened to ownership. By having this IPRA, it serves as a way that removed
the barriers when it comes to the idea of reconciling the Regalian Doctrine and Doctrine of Native
Title such as for this instance wherein it is impossible to own land in Baguio or in the whole
Cordillera Administrative Region under PD 1529 since lands which are beyond 18 % slope cannot
be titled but this problem was cured. On the other hand, the NCIP does not give title but it merely
facilitates the recognition of ancestral lands. It is a recognition of something that is already present.
Moreover, registration may spawn if ownership of land is not acquired or proven by the seven
modes of acquiring ownership.
Separate Opinions on the IPRA Law’s Constitutionality:
Justice Puno; NO. Ancestral lands and ancestral domains are not part of the lands of the
public domain. They are private and belong to the ICCs/IPs. The classification of lands in the
public domain under Section 3, Article XII of the Constitution does not include ancestral lands nor
ancestral domains. The rights of ICCs/IPs to their ancestral domains and ancestral lands may be
acquired in two modes: (1) by native title over both ancestral lands and domains; or (2) by torrens
title under the Public Land Act and the Land Registration Act with respect to ancestral lands only.
Both modes presume or recognize the land as private and not public.
The right of ownership to ancestral domain under Section 7(a) involves “lands, bodies of
water traditionally and actually occupied by ICCs/IPs, sacred places, traditional hunting and
fishing grounds, and all improvements made by them at any time within the domains”, not “waters,
minerals, coal, petroleum, and other mineral oils, all forces of potential energy, fisheries, forests
or timber, wildlife, flora and fauna, and other natural resources” enumerated in Section 2, Article
XII of the Constitution. Ownership therefore of natural resources remain with the State.
Justice Vitug; YES. Sections 7 and 57 go beyond the context of the fundamental law and
virtually amount to an undue delegation, if not an unacceptable abdication, of State authority over
a significant area of the country and its patrimony.
Further, Art. 12, Sec 2 provides that all "lands of the public domain, waters, minerals, coal,
petroleum, and other mineral oils, all forces of potential energy, fisheries, forest or timber, wildlife,
flora and fauna, and other natural resources are owned by the State," and, with the exception of
agricultural lands, "shall not be alienated." It ordains that the "exploration, development, and
utilization of natural resources shall be under the full control and supervision of the State.
With the IPRA Law, it allows such natural resources into co-production, joint venture or
production sharing agreements, or into agreements with foreign owned corporation involving
technical or financial assistance for large-scale exploration, development and utilization. Thus, the
State loses full control and supervision over the public lands and natural resources covered by the
IPRA Law. It would go against the Constitution.
Justice Panganiban; I vote to partially GRANT the Petition and to DECLARE as
UNCONSTITUTIONAL Sections 3(a) and (b), 5, 6, 7(a) and (b), 8 and related provisions of RA
8371.
RA 8371, which defines the rights of indigenous cultural communities and indigenous
peoples, admittedly professes a laudable intent. It was primarily enacted pursuant to the state
policy enshrined in our Constitution to "recognize and promote the rights of indigenous cultural
communities within the framework of national unity and development." Though laudable and well-
meaning, this statute, however, has provisions that run directly afoul of our fundamental law from
which it claims origin and authority. More specifically, Sections 3(a) and (b), 5, 6, 7(a) and (b), 8
and other related provisions contravene the Regalian Doctrine -- the basic foundation of the State's
property regime.
Respondent NCIP claims that IPRA does not violate the Constitution, because it does not
grant ownership of public domains and natural resources to ICCs/IPs. Rather, it recognizes and
mandates respect for the rights of indigenous peoples over their ancestral lands and domains that
had never been lands of the public domain. I say, however, that such claim finds no legal support.
Nowhere in the Constitution is there a provision that exempts such lands and domains from its
coverage. Quite the contrary, it declares that all lands of the public domain and natural resources
are owned by the State; and with the exception of agricultural lands, all other natural resources
shall not be alienated. RA 8371 Violates the Inalienability of Natural Resources and of Public
Domains.
Under Section 3, Article XII of the Constitution, Filipino citizens may acquire no more
than 12 hectares of alienable public land, whether by purchase, homestead or grant. More than
that, but not exceeding 500 hectares, they may hold by lease only. RA 8371, however, speaks of
no area or term limits to ancestral lands and domains. In fact, by their mere definitions, they could
cover vast tracts of the nation's territory. The properties under the assailed law cover everything
held, occupied or possessed "by themselves or through their ancestors, communally or individually
since time immemorial." It also includes all "lands which may no longer be exclusively occupied
by [them] but from which they traditionally had access to for their subsistence and traditional
activities, particularly the home ranges of ICCs/IPs who are still nomadic and/or shifting
cultivators."
Justice Kapunan; YES. The provisions of the IPRA affirming the ownership by the
indigenous peoples of their ancestral lands and domains by virtue of native title do not diminish
the State’s ownership of lands of the public domain, because said ancestral lands and domains are
considered as private land, and never to have been part of the public domain, following the doctrine
laid down in Cariño vs. Insular Government. A proper reading of Cariño would show that the
doctrine enunciated therein applies only to lands which have always been considered as private,
and not to lands of the public domain, whether alienable or otherwise. A distinction must be made
between ownership of land under native title and ownership by acquisitive prescription against the
State. Ownership by virtue of native title presupposes that the land has been held by its possessor
and his predecessors-in-interest in the concept of an owner since time immemorial.
The constitutional provision vesting ownership over minerals, mineral lands and other
natural resources in the State is not violated by provisions of the IPRA which grant certain rights
to the indigenous peoples over the natural resources found within the ancestral domains, e.g., to
benefit from and share in the profits from the allocation and utilization of the same, as well as
priority rights in the harvesting, extraction, development or exploitation thereof. The State retains
full control over the exploration, development and utilization of natural resources even with the
grant of said rights to the indigenous peoples, through the imposition of requirements and
conditions for the utilization of natural resources under existing laws, such as the Small-Scale
Mining Act of 1991 and the Philippine Mining Act of 1995. Moreover, the rights granted to
indigenous peoples for the utilization of natural resources within their ancestral domains merely
amplify what has been earlier granted to them under the aforesaid laws.
It also bears stressing that the grant of priority rights does not preclude the State from
undertaking activities, or entering into co-production, joint venture or production-sharing
agreements with private entities, to utilize the natural resources which may be located within the
ancestral domains. There is no intention, as between the State and the indigenous peoples, to create
a hierarchy of values; rather, the object is to balance the interests of the State for national
development and those of the indigenous peoples.
Justice Mendoza; NO. Petitioners do not complain of any injury as a result of the
application of the statute to them. The exercise of jurisdiction where there is no genuine
controversy is not to show timidity but respect for the judgment of a coequal department of
government whose acts, unless shown to be clearly repugnant to the fundamental law, are
presumed to be valid. The polestar of constitutional adjudication was set forth by Justice Laurel in
the Angara case when he said that "this power of judicial review is limited to actual cases and
controversies to be exercised after full opportunity of argument by the parties, and limited further
to the constitutional question raised or the very lis mota, presented." For the exercise of this power
is legitimate only in the last resort, and as a necessity in the determination of real, earnest, and vital
controversy between individuals. Until, therefore, an actual case is brought to test the
constitutionality of the IPRA, the presumption of constitutionality, which inheres in every statute,
must be accorded to it.
Case No. 57
Topic: Restrictions on Alienation and Encumbrance of lands acquired through Homestead
and Free Patents

Filinvest Land v. Backy


Facts: Respondents were grantees of agricultural public lands located in Tambler, General
Santos City through Homestead and Fee patents sometime in 1986 and 1991 which are covered by
Original Certificates of Title issued by the Register of Deeds of General Santos City.
Negotiations were made by petitioner. Eventually, a Deed of Conditional Sale of the above-
enumerated properties in favor of petitioner Filinvest Land, Inc. was executed. Upon its execution,
respondents were asked to deliver to petitioner the original owner's duplicate copy of the
certificates of title of their respective properties.
A few days after the execution of the aforestated deeds and the delivery of the
corresponding documents to petitioner, respondents came to know that the sale of their properties
was null and void, because it was done within the period that they were not allowed to do so and
that the sale did not have the approval of the Secretary of the Department of Environment and
Natural Resources (DENR) prompting them to file a case for the declaration of nullity of the deeds
of conditional and absolute sale of the questioned properties and the grant of right of way with the
RTC, Las Piñas, Branch 253.
On the other hand, petitioner claims that sometime in 1995, the representative of Hadji
Ngilay approached petitioner to propose the sale of a portion of his properties. Thereafter,
representatives of petitioner flew to General Santos City from Manila to conduct an ocular
inspection of the subject properties. Petitioner was willing to purchase the properties but seeing
that some of the properties were registered as land grants through homestead patents.
RTC’s Ruling: The RTC ruled in favor of Filinvest Land, Inc. and upheld the sale of all
the properties in litigation. It found that the sale of those properties whose original certificates of
title were issued by virtue of the 1986 Patents was valid, considering that the prohibitory period
ended in 1991, or way before the transaction took place.
As to those patents awarded in 1991, the same court opined that since those properties were
the subject of a deed of conditional sale, compliance with those conditions is necessary for there
to be a perfected contract between the parties.
CA’s Ruling: While the CA upheld the validity of the sale of the properties the patents of
which were awarded in 1986, including the corresponding grant of right of way for the same lots,
it nullified the disposition of those properties granted through patents in 1991 and the right of way
on the same properties.
The CA ruled that the contract of sale between the parties was a perfected contract, hence,
the parties entered into a prohibited conveyance of a homestead within the prohibitive period of
five years from the issuance of the patent
Petitioner’s Contentions: A conditional sale involving the 1991 patents did not violate
the prohibition against alienation of homesteads under the public land act since no actual transfer
or disposition was perfected until all the conditions of the deed are fulfilled.
Registration is the operative act that conveys or disposes rights in real property. Being
unregistered, the deed of conditional sale did not convey or dispose of the 1991 homesteads or any
rights therein in violation of the public land act.
Assuming the nullity of the sale of the 1991 patents, the honorable court of appeals should
have ordered respondents as a matter of law to return to petitioners what they have received.
Respondent’s Arguments: The Honorable Court of Appeals did not err in holding that
the Deed of Conditional Sale and Deed of Absolute Sale for the properties covered by the 1991
Patents, as well as the Right of Way Agreement thereto is null and void for the simplest reason
that the said transactions were volatile of the Public Land Act.
Issue: Is the conditional sale of the subject lands void for these being covered by 1991
Patents?
Held: Yes. The Public Land Act. It bears stressing that the law was enacted to give the
homesteader or patentee every chance to preserve for himself and his family the land that the State
had gratuitously given to him as a reward for his labour in cleaning and cultivating it. Its basic
objective, as the Court had occasion to stress, is to promote public policy that is to provide home
and decent living for destitute, aimed at providing a class of independent small landholders which
is the bulwark of peace and order. Hence, any act which would have the effect of removing the
property subject of the patent from the hands of a grantee will be struck down for being violative
of the law.
In the present case, the negotiations for the purchase of the properties covered by the patents issued
in 1991 were made in 1995 and, eventually, an undated Deed of Conditional Sale was executed.
On October 28, 1995, respondents received the downpayment of P14,000.000.00 for the properties
covered by the patents issued in 1991. Applying the five-year prohibition, the properties covered
by the patent issued on November 24, 1991 could only be alienated after November 24, 1996.
Therefore, the sale, having been consummated on October 28, 1995, or within the five-year
prohibition, is as ruled by the CA, void.
Case No. 58
Topic: Restrictions on Alienation and Encumbrance of lands acquired through Homestead
and Free Patents

Abelgas v. Comia, G.R. No. 174715, October 11, 2012


FACTS: Respondents were grantees of agricultural public lands located in Tambler,
General Santos City through Homestead and Fee patents sometime in 1986 and 1991.
Negotiations were made by petitioner, represented by Lina de Guzman-Ferrer with the
patriarch of the Ngilays, Hadji Gulam Ngilay sometime in 1995. Eventually, a Deed of Conditional
Sale of the above- enumerated properties in favor of petitioner Filinvest Land, Inc. was executed.
Upon its execution, respondents were asked to deliver to petitioner the original owner's duplicate
copy of the certificates of title of their respective properties. Respondents received the
downpayment for the properties. A few days after the execution of the aforestated deeds and the
delivery of the corresponding documents to petitioner, respondents came to know that the sale of
their properties was null and void, because it was done within the period that they were not allowed
to do so and that the sale did not have the approval of the Secretary of the Department of
Environment and Natural Resources (DENR). Thereafter, representatives of petitioner flew to
General Santos City from Manila to conduct an ocular inspection of the subject properties.
Petitioner was willing to purchase the properties but seeing that some of the properties were
registered as land grants through homestead patents.
LOWER COURT’S RULING: RTC. The RTC ruled in favor of Filinvest Land, Inc. and
upheld the sale of all the properties in litigation. It found that the sale of those properties whose
original certificates of title were issued by virtue of the 1986 Patents was valid, considering that
the prohibitory period ended in 1991, or way before the transaction took place. Furthermore, the
RTC also upheld the grant of right of way as it adjudged that the right of way agreement showed
that the right of way was granted to provide access from the highway to the properties to be
purchased.
COURT OF APPEALS. Respondents elevated the case to the CA in which the latter
modified the judgment of the RTC. While the CA upheld the validity of the sale of the properties
the patents of which were awarded in 1986, including the corresponding grant of right of way for
the same lots, it nullified the disposition of those properties granted through patents in 1991 and
the right of way on the same properties. As to the "1991 Patents," the CA ruled that the contract
of sale between the parties was a perfected contract, hence, the parties entered into a prohibited
conveyance of a homestead within the prohibitive period of five years from the issuance of the
patent.
ISSUE: Is a conditional sale involving the 1991 patents violative of the prohibition against
alienation of homesteads under the public land act?
RULING: Yes. The five-year prohibitory period following the issuance of the homestead
patent is provided under Section 118 of Commonwealth Act No. 141, as amended by
Commonwealth Act No. 456, otherwise known as the Public Land Act. It bears stressing that the
law was enacted to give the homesteader or patentee every chance to preserve for himself and his
family the land that the State had gratuitously given to him as a reward for his labour in cleaning
and cultivating it. Its basic objective, as the Court had occasion to stress, is to promote public
policy that is to provide home and decent living for destitute, aimed at providing a class of
independent small landholders which is the bulwark of peace and order. Hence, any act which
would have the effect of removing the property subject of the patent from the hands of a grantee
will be struck down for being violative of the law.
Petitioner argues that the correct formulation of the issue is not whether there was a
perfected contract between the parties during the period of prohibition, but whether by such deed
of conditional sale there was "alienation or encumbrance" within the contemplation of the law.
This is wrong. The prohibition does not distinguish between consummated and executory sale. The
conditional sale entered into by the parties is still a conveyance of the homestead patent. And, even
assuming that the disputed sale was not yet perfected or consummated, still, the transaction cannot
be validated. The prohibition of the law on the sale or encumbrance of the homestead within five
years after the grant is mandatory. The purpose of the law is to promote a definite policy, i.e., "to
preserve and keep in the family of the homesteader that portion of the public land which the State
has gratuitously given to him." Thus, the law does not distinguish between executory and
consummated sales. Where the sale of a homestead was perfected within the prohibitory period of
five years; the fact that the formal deed of sale was executed after the expiration of the staid period
did not and could not legalize a contract that was void from its inception.
Case No. 59
Topic: Restrictions on Alienation and Encumbrance of lands acquired through Homestead
and Free Patents

Binayug v. Ugaddan
FACTS: At the crux of this controversy are two parcels of land covered by an OCT in the
name of Gerardo Ugaddan, husband of respondent Basilia Lacambra and father of the other
respondents Eugenio, Norberto, Pedro, Angelina, Tereso, Dominga, and Geronima, all bearing the
surname Ugaddan. Gerardo azquired title over the subject properties through the grant of
Homestead Patent in his favor. Upon Gerardo’s death, respondents discovered that the OCT had
been cancelled. The records of the Registry of Deeds show that Gerardo, with the consent of his
wife Basilia, sold the subject properties to Juan Binayug. As a result of the sale, the OCT in
Gerardo’s name was cancelled and a TCT in Juan’s name was issued. Juan was the father of
petitioner Alejandro Bunayug and the subject properties passed on to him and his wife Ana
Ugaddan Binayug upon Juan’s death. After conducting their own investigation, respondents filed
a complaint “for declaration of nullity of title, annulment of instrument, and declaration of
ownership with damages” against petitioners. Respondents averred that the purported sale between
Gerardo and Juan was prohibited under the Public Land Act; and that the Absolute Deed of Sale
between Gerardo (with Basilia’s consent) and Juan was forged. Hence, respondents asserted that
the Absolute Deed of Sale was forged and that they fraudulently obtained the TCT. The RTC
declared the TCT in name of Juan null and void.
ISSUE: Whether or not Section 118 of the Public Land Act is applicable in this case.
HELD: Section 118 of the Public Land Act, as amended, reads that “[e]xcept in favor of
the Government or any of its branches, units, or institutions, or legally constituted banking
corporations, lands acquired under free patent or homestead provisions shall not be subject to
encumbrance or alienation from the date of the approval of the application and for a term of five
years from and after the date of issuance of the patent or grant x x x.” The provisions of law are
clear and explicit. A contract which purports to alienate, transfer, convey, or encumber any
homestead within the prohibitory period of five years from the date of the issuance of the patent is
void from its execution. In a number of cases, this Court has held that such provision is mandatory.
In the present case, it is settled that Homestead Patent No. V-6269 was issued to Gerardo
on January 12, 1951 and the Absolute Deed of Sale between Gerardo and Juan was executed on
July 10, 1951, after a lapse of only six months. Irrefragably, the alienation of the subject properties
took place within the five-year prohibitory period under Section 118 of the Public Land Act, as
amended; and as such, the sale by Gerardo to Juan is null and void right from the very start. As a
void contract, the Absolute Deed of Sale dated July 10, 1951 produces no legal effect whatsoever
in accordance with the principle “quod nullum est nullum producit effectum,”23 thus, it could not
have transferred title to the subject properties from Gerardo to Juan and there could be no basis for
the issuance of TCT No. T-106394 in Juan’s name. A void contract is also not susceptible of
ratification, and the action for the declaration of the absolute nullity of such a contract is
imprescriptible.
Case No. 60
Topic: Restrictions on Alienation and Encumbrance of lands acquired through Homestead
and Free Patents

Flores v. Bagaoisan
FACTS: On December 20, 1976, petitioners, together with their mother Luisa Viernes,
executed a Deed of Confirmation and Quitclaim in favor of Vicente T. Lazo. Through this
document, petitioners agreed to "sell, cede, convey, grant, and transfer by way of QUITCLAIM"
the subject property to Lazo. Thereafter, respondent, Marciano Bagaoisan, bought the subject
property from Lazo, as evidenced by a Deed of Absolute Sale dated February 20, 1977.
On June 21, 1996, respondent filed an action for ownership, quieting of title, partition and
damages against petitioners, praying that he be declared as the true owner of the subject property
and that the entire property covered by the OCT be partitioned among them.
In answer, petitioners stated that they did not relinquish ownership or possession of the
land to Lazo. While admitting that they executed the Deed of Confirmation and Quitclaim in favor
of Lazo, petitioners claimed that they were misled into signing the same, with Lazo taking
advantage of their lack of education. They insist that the Deed of Confirmation and Quitclaim is
void as its contents were not fully explained to them, and that it violates Section 118 of the Public
Land Act (Commonwealth Act No. 141), which prohibits the alienation of lands acquired through
a homestead patent.
The RTC ruled in favor of the respondent which was affirmed by the CA, hence, the
petition.
ISSUE: Can the subject property, which was issued pursuant to a Homestead Patent given
on November 12, 1973, be conveyed to the respondent through the Deed of Confirmation and
Quitclaim?
RULING: No. The deed is void for violating the five-year prohibitory period against
alienation of lands acquired through homestead patent as provided under Section 118 of the Public
Land Act. The use of the words "confirmation" and "quitclaim" in the title of the document was
an obvious attempt to circumvent the prohibition imposed by law, as the effect would still be the
alienation or conveyance of the property. To validate such an arrangement would be to throw the
door open to all possible fraudulent subterfuges and schemes that persons interested in land given
to a homesteader may devise to circumvent and defeat the legal provisions prohibiting their
alienation within five years from the issuance of the patent.
It bears stressing that the law was enacted to give the homesteader or patentee every chance
to preserve for himself and his family the land that the State had gratuitously given to him as a
reward for his labor in cleaning and cultivating it. Its basic objective is to promote public policy,
that is to provide home and decent living for destitutes, aimed at providing a class of independent
small landholders which is the bulwark of peace and order. Hence, any act which would have the
effect of removing the property subject of the patent from the hands of a grantee will be struck
down for being violative of the law.
The conveyance of a homestead before the expiration of the five-year prohibitory period
following the issuance of the homestead patent is null and void and cannot be enforced, for it is
not within the competence of any citizen to barter away what public policy by law seeks to
preserve. There is, therefore, no doubt that the Deed of Confirmation and Quitclaim, which was
executed three years after the homestead patent was issued, is void and cannot be enforced.
Case No. 61
Topic: Restrictions on Alienation and Encumbrance of lands acquired through Homestead
and Free Patents

Metropolitan Bank vs. Viray


Facts: On 7 July 1979, Rico Shipping, Inc., represented by its President, Erlinda Viray-
Jarque, together with respondent Edgardo D. Viray, in their own personal capacity and as solidary
obligors, obtained two separate loans from petitioner Metropolitan Bank and Trust Company in
the total amount of P250,000. The debtors executed a promissory note promising to pay in four
semi-annual installments of P62,500 starting on 23 January 1980, with 15% interest and 2% credit
evaluation and supervision fee per annum. The two loans were subsequently renewed and secured
by one promissory note. Under the note, the debtors made a total payment of P134,054 leaving a
balance of P115,946 which remained unpaid despite demands by MBTC. Another promissory note
was executed and the debtors obtained a loan from MBTC an amount of P50,000 payable on
November 2, 1981. Again, the debtors failed to pay on the due date. MBTC filed a complaint for
sum of money against the debtors with the RTC which ruled in favor of MBTC. Meanwhile, on
29 December 1982, the government issued Free Patents in favor of Viray over three parcels of
land. The OCTs containing the free patents were registered with the Registry of Deeds of Cagayan
de Oro City on 18 January 1983. Written in the OCT was the land is subject to provisions of CA
141 which provide that except in favor of the Government or any of its branches, units or
institutions, the land thereby acquired shall be inalienable and shall not be subject to encumbrance
for a period of five years from the date of this patent, and shall not be liable for the satisfaction of
any debt contracted prior to the expiration of said period. The RTC, however, issued a writ of
execution over the lots and sold the lots at public auction on favour of the MBTC as the winning
bidder. Viray filed an action for annulment of sale against the sheriff and MBTC with the RTC.
Viray sought the declaration of nullity of the execution sale, the sheriffs certificate of sale, the
sheriffs deed of final conveyance and the TCT's issued by the Register of Deeds.The RTC declared
that the sale was valid. VIray filed an appeal to the CA. The CA reversed the decision of the RTC
and the sale was considered null and void since it was made during the five years prohibition
written in the OCT.
Issue: Whether or not the auction sale falls within the five years prohibition laid in Sec.
118 of CA 141.
Ruling: Yes. Sec. 118 of CA 141 states that “Except in favor of the Government or any of
its branches, units, or instruction, lands acquired under free patent or homestead provisions shall
not be subject to encumbrance or alienation from the date of the approval of the application and
for a term of five years from and after the date of issuance of the patent and grant, nor shall they
become liable to the satisfaction of any debt contracted prior to the expiration of said period, but
the improvements or crops on the land may be mortgaged or pledged to qualified persons,
associations, or corporations.” The Court ruled that. The law clearly provides that lands which
have been acquired under free patent or homestead shall not be encumbered or alienated within
five years from the date of issuance of the patent or be liable for the satisfaction of any debt
contracted prior to the expiration of the period. And secondly, while it is true that the debt in this
case was contracted prior to the five-year prohibitory period, the same is of no consequence. It
must be emphasized that the main purpose in the grant of a free patent or homestead is to preserve
and keep in the family of the homesteader that portion of public land which the State has given to
him so he may have a place to live with his family and become a happy citizen and a useful member
of the society. The provision that nor shall they become liable to the satisfaction of any debt
contracted prior to the expiration of the five-year period is mandatory and any sale made in
violation of such provision is void and produces no effect whatsoever, just like what transpired in
this case. Clearly, it is not within the competence of any citizen to barter away what public policy
by law seeks to preserve.
Case No. 62
Topic: Restrictions on Alienation and Encumbrance of lands acquired through Homestead
and Free Patents

Kings Properties v. Galido


FACTS: On 18 April 1966, the heirs of Domingo Eniceo, namely Rufina Eniceo and Maria
Eniceo, were awarded with Homestead Patent consisting of four parcels of land located in San
IsidroOn 10 September 1973, a deed of sale covering the Antipolo property was executed between
Rufina Eniceo and Maria Eniceo as vendors and respondent as vendee. Rufina Eniceo and Maria
Eniceo sold the Antipolo property to respondent for P250,000. A certain Carmen Aldana delivered
the owners duplicate copy to respondent. Petitioner alleges that sometime in February 1995,
Bolinas came to the office of Alberto Tronio Jr.petitioners general manager, and offered to sell the
Antipolo property. During an on-site inspection, Tronio saw a house and ascertained that the
occupants were Bolinas relatives. Tronio also went to the Registry of Deeds to verify the records
on file. Tronio ascertained that OCT No. 535 was clean and had no lien and encumbrances. After
the necessary verification, petitioner decided to buy the Antipolo property.
On 16 January 1996, respondent filed a civil complaint with the trial court against the
Eniceo heirs and petitioner. Respondent prayed for the cancellation of the certificates of title issued
in favor of petitioner, and the registration of the deed of sale and issuance of a new transfer
certificate of title in favor of respondent.
ISSUE: Whether or not A grantee or homesteader is prohibited from alienating to a private
individual a land grant
RULING: A grantee or homesteader is prohibited from alienating to a private individual
a land grant within five years from the time that the patent or grant is issued. A violation of this
prohibition renders a sale void. This, however, expires on the fifth year. From then on until the
next 20 years, the land grant may be alienated provided the Secretary of Agriculture and Natural
Resources approves the alienation. The Secretary is required to approve the alienation unless there
are “constitutional and legal grounds” to deny the approval. In this case, there are no apparent or
legal grounds for the Secretary to disapprove the sale of the Subject Land.
The failure to secure the approval of the Secretary does not ipso factomake a sale void. The absence
of approval by the Secretary does not a sale made after the expiration of the 5-year period, for in
such event the requirement of Section 118 of the Public Land Act becomes merely directory or a
formality. The approval may be secured later, producing the effect of ratifying and adopting the
transaction as if the sale had been previously authorized

You might also like